Anda di halaman 1dari 195

KUMPULAN SOAL

ASPD SMP
BAHASA INDONESIA
2021
PERSIAPAN DAN PEMANTAPAN
ASESMEN STANDARISASI PENDIDIKAN DAERAH
MKKS SMP/MTs PROVINSI DIY
TAHUN PELAJARAN 2020/2021

Mata Pelajaran : Bahasa Indonesia


Kelas : IX (Sembilan) UTAMA
Hari dan Tanggal : Senin, 22 Maret 2021
Pukul : 08.00 – 09.30 (90 Menit)

PETUNJUK UMUM
1. Isikan identitas Anda dengan benar
2. Tersedia waktu 90 menit untuk mengerjakan paket tes tersebut.
3. Jumlah soal sebanyak 40 butir, pada setiap butir soal terdapat 4 (empat)
pilihan jawaban.
4. Periksa dan bacalah soal-soal sebelum Anda menjawabnya dengan cermat.
5. Periksalah pekerjaan Anda sebelum dikirim / submit
6. Berdoalah sebelum mengerjakan

PETUNJUK KHUSUS
Jawablah dengan menghitamkan bulatan ( ) huruf A, B, C, atau D pada lembar jawab
yang tersedia!

1. Cermatilah teks berikut!


Menurut dr. Yuniantika, tenaga medis Gadjah Mada Medical Center, upaya
pengendalian penularan koronavirus melalui Pengetatan Terbatas Kegiatan
Masyarakat (PTKM) bisa berhasil jika masyarakat berperan aktif dengan menaati
imbauan dan meminimalkan aktivitas di luar rumah.

Makna kata aktivitas pada teks tersebut adalah ….


A. keberadaan
B. keinginan
C. kegiatan
D. kerjaan

2. Cermati teks berikut!


Penularan koronavirus dapat melalui droplet yang dikeluarkan pada saat seseorang
batuk atau bicara. Penularan terjadi ketika percikan terhirup orang lain yang ada di
sekitar. Oleh karena itu, masker dikenakan untuk melindungi seseorang dari droplet
yang dikeluarkan oleh orang lain. Bahkan, untuk meyakinkan betapa pentingnya
masker pada era pandemi ini, ada slogan yang berbunyi “Maskerku Melindungiku
dan Dirimu”.

Tujuan penulisan teks tersebut adalah ….


A. memberitahu agar melindungi orang lain dengan memberikan masker
B. mengajak orang lain untuk membelikan masker orang di sekitarnya
C. menyampaikan pentingnya melindungi diri dari droplet orang lain
D. mengajak semua orang agar menggunakan masker

pp_aspd Utama Bahasa Indonesia 1


mkks_smp/mts_provinsi_diy
3. Cermati teks berikut!
Ada beberapa kelompok masyarakat yang diprioritaskan mendapat vaksin COVID-19
karena jumlah vaksin yang tersedia masih terbatas. Kelompok tersebut antara lain
tenaga kesehatan yang memiliki risiko tinggi untuk terinfeksi dan menularkan
COVID-19. Kelompok lain yang juga mendapat prioritas, yaitu orang dengan
pekerjaan risiko tinggi tertular dan menularkan COVID-19 karena tidak dapat
melakukan jaga jarak secara efektif, seperti anggota TNI/Polri, aparat hukum, dan
petugas pelayanan publik lainnya.

Ide pokok paragraf tersebut adalah ….


A. keterbatasan sasaran pemberian vaksin
B. prioritas pemberian vaksin koronavirus
C. penyakit penyerta yang berisiko
D. risiko pemberian vaksin

4. Cermati teks berikut!


COVID-19 bisa menyerang siapa saja tanpa memandang status, dari rakyat
biasa hingga pejabat. Meskipun semua bisa terjangkit virus ini, respon tubuh
tiap-tiap orang berbeda. Ada yang reaksi atau gejalanya ringan, sedang, dan
parah hingga memerlukan perawatan khusus dan harus berada di ruang HCU.
Namun, ada juga orang yang tidak menunjukkan gejala apa pun. OTG (Orang
Tanpa Gejala) dapat dikatakan suatu keberuntungan bagi dirinya, tetapi bisa
membahayakan orang di sekitarnya.

Pernyataan yang sesuai dengan teks tersebut adalah …


A. Koronavirus menyerang seseorang yang mengabaikan protokol kesehatan.
B. Semua orang memiliki reaksi sama dan memerlukan perawatan.
C. Orang tanpa gejala (OTG) tidak mempan serangan koronavirus.
D. Orang tanpa gejala (OTG) membahayakan orang di sekitarnya.

5. Cermati teks berikut!


(1) Aktivitas Gunung Merapi terlihat landai dan sudah mulai tenang sehingga
ratusan pengungsi Gunung Merapi dari Desa Balerante dan Tegalmulyo pulang. (2)
Koordinator Posko Pengungsian Desa Balerante Kecamatan Kemalang, Jainu,
mengatakan bahwa warga sudah siap berkemas sejak pukul 07.00 WIB. (3) Evakuasi
pertama adalah logistik dan dilanjutkan pelepasan warga oleh BPBD, pemerintah desa,
dan seluruh relawan pukul 10.30 WIB. (4) Jainu mengatakan bahwa warga sadar status
Gunung Merapi masih siaga sehingga mereka tetap menjaga kewaspadaan. (5) Setelah
evakuasi logistik dan warga, ternak milik warga juga akan dipulangkan secara bertahap.

Kalimat yang menyatakan hubungan sebab-akibat pada teks tersebut ditandai


nomor ....
A. (1) dan (2)
B. (1) dan (4)
C. (2) dan (3)
D. (2) dan (5)

pp_aspd Utama Bahasa Indonesia 2


mkks_smp/mts_provinsi_diy
6. Cermati poster berikut!

Kalimat yang mewakili isi poster tersebut adalah ...


A. Masyarakat diberi tahu bahwa suasana ruang isolasi seperti penjara.
B. Masyarakat diajak untuk tinggal di rumah saja agar terhindar dari COVID-19.
C. Masyarakat diberi tahu agar menjauh dari ruang isolasi penderita COVID-19.
D. Masyarakat diajak menjaga jarak ketika berada diruang isolasi.

7. Cermati teks dan pernyataan berikut!


Teks
Listyo Sigit Prabowo resmi dilantik sebagai Kepala Kepolisian Negara Republik Indonesia
(Kapolri) oleh Presiden Joko Widodo pada Rabu (27/1/2021) di Istana Kepresidenan,
Jakarta, pukul 09.30 WIB. Seusai dilantik, Sigit mengungkapkan akan segera menjalankan
program yang sebelumnya telah disampaikan dalam uji kelayakan dan kepatutan (fit and
proper test) di Komisi III DPR RI. Sigit mengungkapkan keinginannya untuk meniadakan
tilang lalu lintas di jalan. Dia ingin nantinya Polantas yang bertugas di jalan hanya fokus
menjalankan tugas mengatur lalu lintas. Sementara itu, penilangan tetap ada, tetapi
dilakukan secara automasi melalui electronic traffic law enforcement (ETLE). "Yang kami
hindarkan adalah interaksi anggota dengan masyarakat yang menimbulkan potensi
penyalahgunaan kewenangan," kata Sigit.
Pernyataan
(1) Pelanggaran peraturan lalu lintas oleh masyarakat meningkat karena tilang
ditiadakan.
(2) Kenyamanan masyarakat dalam berlalu lintas meningkat karena Polantas yang
bertugas di jalan lebih fokus menjalankan tugas mengatur lalu lintas.
(3) Kepercayaan masyarakat terhadap polisi meningkat karena berkurangnya potensi
polisi dalam menyalahgunakan kewenangan.
(4) Sebagian masyarakat merasa dirugikan dengan program tersebut karena tidak bisa
menjalin hubungan yang baik dengan polisi.

Situasi yang kemungkinan muncul dalam masyarakat berkaitan dengan isi teks
tersebut ditandai dengan nomor ....
A. (1) dan (2)
B. (1) dan (3)
C. (2) dan (3)
D. (3) dan (4)

pp_aspd Utama Bahasa Indonesia 3


mkks_smp/mts_provinsi_diy
8. Cermati teks berikut!
Walaupun dance challenge TikTok terkesan sederhana dan tidak
membutuhkan banyak energi, tren yang satu ini juga baik untuk kesehatan, baik
secara fisik maupun mental. Secara fisik, dance challenge membuat orang-orang
yang semula “mager” dan termasuk kaum “rebahan” akan bangkit dan menari.
Ketika video banyak ditonton, kita pun termotivasi untuk
membuat challenge lainnya.
Selain itu, tren yang satu ini merupakan salah satu sumber hiburan yang mudah
diakses selama masa pandemi. Kehadiran konten dance challenge dan
semacamnya menjadi sebuah penyegaran di tengah gempuran informasi
seputar COVID-19 yang sering membuat kita gelisah.

Ringkasan isi teks tersebut yang tepat adalah ...


A. TikTok yang terkenal dengan dance challenge membuat orang-orang yang
semula “mager” dan termasuk kaum “rebahan” bangkit dan menari.
B. Dance challenge TikTok menjadi penyegaran di tengah gempuran informasi
seputar COVID-19 yang sering membuat kita gelisah.
C. Walaupun terkesan sederhana dan tidak membutuhkan banyak energi, Dance
challenge Tiktok baik untuk kesehatan secara fisik maupun mental.
D. Selain bermanfaat untuk kesehatan fisik maupun mental, dance
challenge TikTok juga merupakan hiburan yang mudah diakses selama masa
pandemi.

9. Cermati teks berikut!


Badan Meteorologi, Klimatologi, dan Geofisika (BMKG) Stasiun Klimatologi Sleman
memperkirakan cuaca ekstrem pada puncak musim hujan tahun 2021 di wilayah DIY
akan mulai pada akhir Januari hingga pertengahan Februari. Prakiraan potensi
cuaca ekstrem tersebut menunjukkan bahwa kondisi dinamika atmosfer tidak stabil
dalam beberapa hari ke depan. Hal itu dapat meningkatkan potensi pertumbuhan
awan hujan di wilayah Yogyakarta. Selain itu, kondisi tersebut juga disebabkan oleh
munculnya pusat tekanan rendah di Australia bagian utara dan munculnya sirkulasi
siklonik di barat Sumatera Utara dan timur Kalimantan.

Simpulan isi teks tersebut adalah ...


A. Cuaca ekstrem terjadi di DIY sepanjang bulan Januari sampai dengan Februari
2021.
B. Diperkirakan pertengahan Februari adalah puncak musim penghujan di wilayah
DIY.
C. Masyarakat DIY hendaknya waspada terhadap kemungkinan terjadinya cuaca
ekstrem.
D. Perkiraan potensi cuaca ekstrem yang terjadi di DIY dikeluarkan oleh BMKG
Sleman.

10. Cermati teks berikut!


(1) Jeruk merupakan salah satu buah yang sangat populer. (2) Jeruk hampir dapat
ditemui di berbagai belahan dunia. (3) Jeruk terdiri atas berbagai jenis, seperti jeruk
manis, jeruk nipis, lemon, dan jeruk bali. (4) Jeruk memiliki lebih dari 170 fitokimia
dan 60 flavonoid.

pp_aspd Utama Bahasa Indonesia 4


mkks_smp/mts_provinsi_diy
Kalimat yang berupa fakta ditandai dengan nomor ….
A. (1) dan (2)
B. (1) dan (3)
C. (2) dan (3)
D. (3) dan (4)

11. Cermati kedua teks berikut!


Teks 1 Teks 2
Yogyakarta terdiri atas lima kabupaten/ Tidak lengkap kayaknya jika kamu
kota dengan variasi budaya dan kondisi berkunjung ke Yogyakarta, tapi tidak
wilayah yang beragam. Masing-masing berwisata ziarah di wilayah Imogiri. Ya,
kabupaten kota di Yogyakarta kecamatan yang terletak di Kabupaten
mempunyai kekhasan budaya dan Bantul ini, menyimpan banyak potensi
tradisi, serta kondisi geografis yang wisata yang unik dan cocok bagi kamu
berbeda. Ada wilayah kota dengan yang suka belajar sejarah. Dari tempat ini,
fasilitas penunjang yang modern, ada kita dapat melihat betapa tangguh dan
pedesaan dengan ragam budaya eloknya raja dan kerajaan di Jawa,
tradisional yang terjaga, perbukitan khususnya, pada masa lampau.
yang sejuk, dan wilayah pantai yang
indah.

Perbedaan bahasa kedua kutipan teks laporan tersebut adalah menggunakan ….

Teks 1 Teks 2
A. kalimat aktif kalimat pasif
B. ragam bahasa formal ragam bahasa nonformal
C. kata ganti orang kedua kata ganti orang pertama
D. kalimat majemuk kalimat tunggal

12. Cermati kedua teks berikut!


Teks 1 Teks 2
Hujan di sebagian besar wilayah Matahari telah meninggi ketika kami
Yogyakarta menyebabkan bencana tiba di lokasi kunjungan. Pada pukul
alam di beberapa lokasi. Banjir dan 11.00 kami beramai-ramai mengikuti
tanah longsor melanda beberapa pemandu yang menjelaskan objek.
wilayah di Kulon Progo, Gunungkidul, Hal-hal penting kami catat sebisanya
dan Bantul. Wilayah tersebut terkena sebagai bahan pembuatan laporan di
dampak paling parah karena secara sekolah. Satu jam kemudian, kami
geografis sebagian wilayah berupa berteduh di bawah pepohonan untuk
perbukitan. melepas lelah.

Perbedaan pola penyajian kedua teks tersebut menggunakan urutan ….


Teks 1 Teks 2
A. sebab-akibat waktu
B. waktu sebab-akibat
C. ruang waktu
D. peristiwa ruang

pp_aspd Utama Bahasa Indonesia 5


mkks_smp/mts_provinsi_diy
13. Cermati kutipan cerita berikut!
(1) Suli melirik jam dinding di kamarnya. Paling lambat tiga puluh menit lagi tugas
dari Bu Tutik harus sudah dikirim. (2) Akhirnya, kurang dua menit dari batas waktu
pengumpulan, tugas sudah terkirim ke surel Bu Tutik. Suli mengembuskan napas
lega. (3) Sebenarnya, Bu Tutik tidak akan marah jika ada yang sedikit terlambat
dalam mengumpulkan tugas. (4) Namun, bukan kebiasaan Suli untuk menunda-
nunda pekerjaan. Apalagi itu tugas dari sekolah yang pengiriman hasil kerjanya
dibatasi.

Bukti bahwa Suli selalu tepat waktu terdapat pada kalimat nomor ….
A. (1) dan (2)
B. (1) dan (3)
C. (2) dan (4)
D. (3) dan (4)

14. Cermati kutipan cerita berikut!


Sehabis mandi Rasti disibukkan dengan jerawat yang berjajar di dahinya.
Dibersihkannya wajahnya dengan hati-hati. Dioleskannya krim khusus pemberian
temannya, yang konon manjur itu, di dahinya. Sudah dua hari ini ia memakainya.
Dilihatnya cermin di depannya. Ia bingung, bagaimana mesti menyamarkan jerawat
itu? Dicobanya berbagai cara agar jerawat itu tak nampak bagai pulau-pulau di
lautan. Lalu dilihatnya jam mejanya. Ia terkejut.
Rasti mengetuk pintu ruang pertemuan. Samar-samar didengarnya pertemuan
sudah dimulai. Hatinya berdebar-debar, takut dimarahi bosnya. Apa alasannya nanti?
Apakah harus jujur kalau sibuk mengurusi jerawat? Ia merasa jerawat telah
mangacaukan hidupnya.

Peristiwa yang terdapat dalam kutipan cerita tersebut adalah …


A. Wajah Rasti penuh dengan jerawat dan hidupnya kacau.
B. Rasti terlambat datang ke pertemuan dan dimarahi bosnya.
C. Jerawat di wajah Rasti membandel dan Rasti bingung mengatasinya.
D. Karena sibuk mengurusi jerawat di wajahnya, Rasti terlambat datang ke
pertemuan.

15. Cermati kutipan cerita berikut!

Saat libur kenaikan kelas, ayah memintaku tetap belajar. Bukan belajar dengan
membaca buku pelajaran, tetapi membaca buku-buku yang lain. Ayah memintaku
membaca buku-buku cerita, seperti novel, biografi, dan kumpulan cerpen.
Ayah paling senang kalau aku membaca riwayat hidup orang-orang hebat. Kata
ayah, bukan tentang kehebatannya yang mesti diingat. Yang mesti direnungkan
adalah kesulitan-kesulitan yang dihadapi. Kesulitan itu pupuk keberhasilan. Semua
kesusahan itu menjadi lambaran kebahagian. Itulah caranya belajar tentang
kehidupan.

Alasan ayah meminta aku membaca buku cerita adalah untuk ….


A. belajar tentang kehidupan
B. mendapatkan kebahagiaan
C. mengetahui kesulitan diri sendiri
D. memecahkan kesulitan orang lain

pp_aspd Utama Bahasa Indonesia 6


mkks_smp/mts_provinsi_diy
16. Cermati kutipan cerita berikut!
Sum termenung. Tinggal dua hari lagi harus bayar kontrakan, tapi uang masih kurang
dua ratus ribu. Ke mana harus mencarinya? Pinjam sama Bu Jantra lagi tak mungkin.
Minggu kemarin dia sudah diberi uang bonus plus uang untuk beli obat anaknya. Ga
tahu diri namanya. Suaminya, seorang sopir bus, belum tentu bawa pulang uang. Jalan
makin padat. Ribuan motor memenuhi jalanan. Sum memegangi kepalanya yang terasa
pening.

Latar suasana kutipan cerita tersebut adalah ….


A. panik
B. sedih
C. murung
D. bingung

17. Cermati kutipan fabel berikut!


Suatu hari, ketika mereka sedang bercanda, datanglah Mongki dengan gayanya
yang angkuh.
“Hai, Teman-teman, sedang apa kalian?” tanya Mongki dengan senyum
congkaknya.
“Hai juga, Mongki!” jawab Kiki Tupai datar.
Mongki, monyet berekor panjang, memang dikenal sombong di hutan itu. Ia selalu
bercerita tentang kehebatannya. Ia merasa paling tampan dan selalu meremehkan
hewan lain. Itu sebabnya, ia dijauhi penghuni hutan.

Cara pengarang menggambarkan watak tokoh Mongki melalui….


A. tingkah laku tokoh
B. cara bicara tokoh
C. pendapat tokoh lain
D. diceritakan langsung penulis

18. Cermati kutipan cerpen berikut!


Sekuat tenaga Tom mempertahankan Wilson ketika ombak besar menerpa. Perahu
kayunya terombang-ambing sampai akhirnya hancur berkeping-keping. Tom
bertahan di atas papan sebesar tubuhnya. Wilson, bola tenis sahabatnya, terlepas
dan hanyut di tengah badai lautan. Tom menangis sekeras-kerasnya sambil
memanggil-manggil Wilson.

Kutipan cerpen tersebut merupakan tahapan alur….


A. pengenalan
B. klimaks
C. peleraian
D. penyelesaian

19. Cermati kutipan cerpen berikut!


“Bagaimana bisa kau tidak tahu? Sudah jelas kau yang meminjamnya kemarin!”
Langkahku terhenti saat suara Syifa terdengar jelas dari dalam kelas. Tanpa
bersuara aku mengintip dari celah jendela. “Lalu kau menuduh aku yang menyobek
buku catatanmu? Aku mengembalikannya dalam keadaan baik-baik saja!” Ternyata
Syifa sedang berbicara dengan Rena. Situasi di dalam kelas terasa panas,
membuatku malas untuk masuk ke kelas.

pp_aspd Utama Bahasa Indonesia 7


mkks_smp/mts_provinsi_diy
Makna simbol panas pada kutipan cerpen tersebut adalah ....
A. menggelisahkan
B. menegangkan
C. menakutkan
D. mengerikan

20. Cermati kutipan fabel berikut!


Saat kura-kura sedang memanen buah-buah segar dari kebunnya, kera
memperhatikan dari kejauhan. Kera melihat buah-buah segar itu dengan mata
berbinar. Kera mendekati kura-kura dan berkata, “Kura-kura, di desa seberang ada
kebun buah yang luas. Di sana kau bisa memanen buah sebanyak yang kau mau.
Aku akan mengantarmu ke sana.” Kura-kura tergiur dan sepakat pergi bersama
Kera ke desa seberang. Di perjalanan, Kera berusaha keras membuat perangkap
agar Kura-kura tak bisa kembali ke kebunnya. Tiba-tiba, Kura-kura terjebak di salah
satu perangkap yang dibuat oleh Kera. “Hei Kera, bantu aku!” Kera mengejek Kura-
kura dan berlari kembali ke arah kebun milik Kura-kura. Kera sudah tak sabar ingin
menguras habis buah milik Kura-kura, tapi tiba-tiba ia terjerat perangkap buatannya
sendiri. Kura-kura yang melihatnya pun tertawa.

Amanat dalam kutipan fabel tersebut adalah....


A. teman harus saling menolong, jangan serakah
B. jangan suka berbohong, jangan sembrono
C. jangan sembrono, jangan mudah percaya
D. jangan serakah, jangan mudah percaya

21. Cermati kutipan cerita berikut!

Pedagang garam tersebut memiliki seekor keledai yang digunakan untuk


mengangkut garam ke kota terdekat. Dalam hati, ia sangat menyayangi keledai
tersebut sampai makanan dan tempat tinggal keledai selalu disediakan. Keledai
sudah dianggap keluarga dan menjadi teman hidup satu-satunya pedagang garam
tersebut. Akan tetapi, keledai tidak puas dengan perlakuan pedagang garam.

Sudut pandang kutipan cerita tersebut adalah …


A. orang ketiga serbatahu
B. orang ketiga pengamat
C. orang pertama pelaku utama
D. orang pertama pelaku sampingan

pp_aspd Utama Bahasa Indonesia 8


mkks_smp/mts_provinsi_diy
22. Cermati kutipan cerita berikut!

Ketiga anak itu terkejut melihat ibunya. Wajahnya memang wajah ibu mereka,
tapi badannya sungguh mengerikan. Badannya penuh sisik dan tidak berkaki. Si Ibu
memiliki ekor sama persis seperti ikan.
Si Bungsu menangis keras melihat ibunya. Ia bahkan menolak untuk disusui. Si
Sulung marah.
"Kau bukan Ibu kami, kau pasti ikan yang mencelakai Ibu kami. Ibu, Ibu, di mana
Ibu?" teriak si Sulung.
"Percayalah, Nak! Aku ini ibumu. Ibu berubah seperti ini karena bertekad untuk
tinggal di laut. Si Ibu mencoba menjelaskan. Namun, ketiga anaknya bergeming.
Mereka justru meninggalkan ibunya dan pulang.

Simpulan akibat konflik kutipan cerita tersebut adalah …


A. Wajah Ibu mengerikan.
B. Si Sulung tidak mau ditinggal.
C. Ketiga anaknya pergi.
D. Si Bungsu ketakutan melihat si Ibu.

23. Cermati kedua teks berikut!


Teks 1 Teks 2
Ani anak yang pandai. Selain pandai, ia Suatu hari ia tak hadir ke sekolah.
juga suka menari. Di sekolah ada Sepulang sekolah, aku ke rumahnya. Tapi
ekstrakurikuler menari, tentu Ani salah rumahnya kosong. Aku sangat bingung
satu anggotanya. Hari ini sepulang kenapa hari ini Dinda tak ada. Biasanya
sekolah, Ani ada latihan menari dengan kalau ia mau pergi, selalu memberi
ketiga temannya untuk persiapan tahuku, tapi kali ini tidak. Aku bingung
lomba lusa. sekali.

Perbedaan pola pengembangan kedua kutipan teks tersebut diawali dengan …


Teks 1 Teks 2
A. pemunculan masalah pendeskripsian waktu
B. pengenalan tokoh pendeskripsian tempat
C. pendeskripsian tempat pengenalan tokoh
D. pengenalan tokoh pemunculan masalah

24. Bacalah kedua teks cerita berikut!


Teks 1 Teks 2
Seekor domba tinggal di tepian hutan Seperti bentuk jari manis yang anggun,
bersama ketujuh anaknya. Setiap hari belimbing putih sering dipesan kurcaci
domba tersebut keluar untuk mencari wanita untuk mempercantik wajah dan
makanan di hutan. Sebelum keluar tubuh supaya tetap segar dan penuh
rumah, sang Domba memberi tahu pesona. Nah, belimbing biru cabang
anak-anaknya supaya berjaga-jaga. Ia kelingking kecil dan agak rapuh. Belimbing
berkata,”Serigala itu hewan yang biru berkhasiat menyembuhkan penyakit
sangat licik dan jahat. Apabila kalian lupa. Semua kurcaci yang pelupa di hutan
mendengar suara lolongan dan melihat Morin, langsung pulih ingatannya ketika
empat kuku hitamnya, jangan pernah memakan belimbing biru.
membuka pintu ini!”

pp_aspd Utama Bahasa Indonesia 9


mkks_smp/mts_provinsi_diy
Perbedaan penggunaan bahasa kedua kutipan teks tersebut adalah …
Teks 1 Teks 2
A. mengandung makna konotasi mengandung makna lugas
B. menggunakan kata sandang tidak menggunakan kata sandang
C. semua kalimat tunggal semua kalimat majemuk
D. tidak terdapat bahasa daerah terdapat bahasa daerah

25. Cermati teks berikut!


Ibu pernah mengatakan kepadaku bahwa tekad Ayah untuk menjadi pelukis
mumpuni begitu besar. Meski begitu, aku sering mendengar orang-orang
mengatakan bahwa Ayah tidak akan pernah berhasil menjadi pelukis. Bukti dari
pernyataan itu, setiap kali ayah mengadakan pameran, belum pernah sekali pun
lukisannya terjual.
“Ayahmu pantang menyerah,” sahut Ibu ketika aku bertanya tentang kapan ayah
mulai merintis dunia lukisnya.
“Menurut Ibu gimana?” tanyaku.
“Maksudmu?”
Aku ingin tahu pendapat Ibu perihal perjuangan Ayah. Dalam hati aku penasaran.

Komentar yang tepat terhadap isi teks cerita tersebut adalah...


A. Nasib pelukis akan ditentukan oleh keberhasilannya dalam mengadakan
pameran.
B. Melukis tidak bisa dijadikan sebuah profesi andalan bagi seorang kepala rumah
tangga.
C. Sikap Ayah yang gigih dan pantang menyerah dalam mewujudkan cita-cita perlu
diteladani.
D. Usaha yang dilakukan Ayah untuk menjadi pelukis mumpuni tidak mungkin
dapat tercapai.

26. Cermati teks berikut!


Pembangunan dalam rangka reparasi beberapa kawasan di Kota Yogyakarta telah
dilakukan. Gubernur DIY, Sri Sultan Hamengku Buwono X, meresmikan empat
kawasan hasil pembangunan Kota Yogyakarta pada Jumat, 18 Desember 2020.
Karena pandemi COVID-19, peresmian dilakukan secara virtual dari Gedhong
Pracimasana, sentral Kepatihan, Yogyakarta.

Perbaikan penggunaan istilah pada teks tersebut adalah ....


A. reparasi diganti revitalisasi, sentral di ganti kompleks
B. reparasi diganti reformasi, sentral di ganti grha
C. reparasi diganti revitalisasi, virtual diganti live
D. virtual diganti live, sentral di ganti grha

27. Cermati paragraf rumpang berikut!


Gadjah Mada Electronic Nose atau yang sering disebut GeNose adalah sebuah
alat pendeteksi koronavirus yang dibuat oleh para ahli dari Universitas
Gadjah Mada. Alat ini dapat mendeteksi keberadaan COVID-19 melalui
embusan napas hanya dalam kurun waktu 80 detik. […]. Selanjutnya, ada
sebuah sensor pada alat tersebut yang akan mendeteksi volatille organic
compound (VOC).

pp_aspd Utama Bahasa Indonesia 10


mkks_smp/mts_provinsi_diy
Kalimat yang tepat untuk melengkapi bagian yang rumpang adalah …
A. Pasien cukup mengembuskan napas ke sebuah kantong khusus yang
terhubung langsung dengan GeNose.
B. Pasien cukup menghirup udara dari sebuah kantong khusus yang
terhubung langsung dengan GeNose.
C. Pasien cukup menghirup udara selama 18 detik dari kantong yang
terhubung langsung dengan GeNose.
D. Pasien cukup menahan napas selama satu menit dalam keadaan hidung
terhubung langsung dengan GeNose.

28. Cermati petunjuk berikut!

Cara Membuat Disinfektan dari Cairan Pemutih


(1) Siapkan botol kaca, cairan pemutih, dan air bersih!
(2) Cairan disinfektan siap digunakan!
(3) Tambahkan air bersih lalu tutup botol kaca dengan rapat!
(4) Tuang cairan pemutih atau cairan karbol secara hati-hati ke dalam botol kaca
terlebih dahulu!
(5) Kocok botol kaca secara perlahan agar cairan pemutih atau cairan karbol dapat
tercampur sempurna dengan air!
(6) Bagikan larutan cairan tersebut ke dalam botol semprot yang lebih kecil agar
mudah digunakan!

Urutan petunjuk yang tepat adalah ….


A. (1),(4),(5),(6),(3),(2)
B. (1),(4),(3),(5),(6),(2)
C. (1),(4),(5),(3),(6),(2)
D. (1),(4),(3),(6),(5),(2)

29. Cermati kalimat berikut!

Wakil Menteri Kesehatan, dr. Dante Saksono, mengungkapkan bahwa ada dua
kasus varian korona B117 yang terdeteksi dibulan Maret

Variasi kata terdeteksi adalah ….


A. diperkirakan
B. dipastikan
C. ditemukan
D. diumumkan

30. Cermati teks berikut!

(1) Kecelakaan lalu lintas bisa terjadi pada jenis kendaraan apapun, baik kendaraan
laut, kendaraan udara, dan kendaraan darat. (2) Kecelakaan tersebut meninggalkan
bekas yang buruk secara mental dan spiritual. (3) Berita tentang kecelakaan akibat
daripada mengabaikan peraturan lalu lintas hampir tidak pernah absen dari media
massa di Indonesia. (4) Hampir setiap hari media massa mengabarkan peristiwa
kecelakaaan.

pp_aspd Utama Bahasa Indonesia 11


mkks_smp/mts_provinsi_diy
Kalimat tidak efektif dalam paragraf tersebut ditandai nomor ….
A. (1) dan (2)
B. (1) dan (3)
C. (2) dan (4)
D. (3) dan (4)

31. Cermati kalimat-kalimat berikut!


(1) Penjelasan apoteker belum difahami oleh pasien.
(2) Selama pandemi COVID-19, kita disarankan untuk beraktivitas di rumah.
(3) Sirene merupakan bagian dari sistim peringatan dini tsunami.
(4) Zifa mengolesi tangan dan kakinya dengan balsam.

Kata tidak baku ditunjukkan kalimat nomor ….


A. (1) dan (3)
B. (2) dan (3)
C. (2) dan (4)
D. (3) dan (4)

32. Cermati teks berikut!

Lingkungan hidup sangat […]1 kehidupan manusia. Lingkungan hidup ini mencakup
benda hidup dan benda mati. Jika […]2 dengan baik, lingkungan hidup dapat
menciptakan masyarakat yang sehat.

Kata berimbuhan yang tepat untuk melengkapi paragraf tersebut adalah ….


A. (1) berpengaruh, (2) terpelihara
B. (1) berpengaruh, (2) memelihara
C. (1) memengaruhi, (2) terpelihara
D. (1) memengaruhi, (2) memelihara

33. Cermati kalimat berikut!

Sejak pandemi COVID-19, peserta didik melaksanakan pembelajaran jarak jauh


ke rumah.

Alasan ketidaktepatan penggunaan kata depan ke pada kalimat tersebut karena


menunjuk ….
A. tempat
B. asal
C. posisi
D. tujuan

34. Cermati teks berikut!

(1) Aktivitas wisata mulai menggeliat di masa new normal pandemi COVID-19. (2)
Hal ini ditandai dengan adanya berbagai perubahan. (3) Harus ada perbaikan
ekonomi di segala sektor. (4) Harapannya, pelaku industri pariwisata dapat
menerapkan regulasi yang telah ditetapkan.

pp_aspd Utama Bahasa Indonesia 12


mkks_smp/mts_provinsi_diy
Perbaikan kalimat (3) agar menjadi paragraf padu adalah …
A. Pemerintah daerah berhak mengeluarkan regulasi demi keamanan dan
kenyamanan setiap wisatawan yang datang.
B. Perubahan tersebut meliputi tujuan wisata, kegiatan wisata, dan pedoman
keselamatan berwisata.
C. Setiap wisatawan yang datang harus mematuhi protokol kesehatan agar tetap
aman selama berlibur.
D. Aktivitas wisatawan mengalami perubahan untuk melindungi industri pariwisata
yang terancam.

35. Cermati kalimat berikut!

Banyak beberapa warga mengungsi di barak yang disediakan pemerintah daerah.

Kalimat tersebut tidak efektif karena ….


A. pemborosan kata
B. tidak memiliki subjek
C. mengandung subjek ganda
D. tidak terdapat predikat

36. Cermati kalimat-kalimat berikut!

(1) Pelaksanaan PJJ akan tetap dilanjutkan, karena tingginya angka penularan
virus COVID-19.
(2) “Kita tidak boleh membuka masker saat berbicara dengan orang lain,” kata
Andin.
(3) Ia membeli beras; telur; minyak goreng; gula, dan sabun mandi sebagai
persiapan menghadapi PPKM.
(4) Meski mengalami erupsi, Gunung Merapi masih berada pada level siaga.

Kalimat dengan tanda baca yang tepat terdapat pada nomor….


A. (1) dan (2)
B. (2) dan (3)
C. (2) dan (4)
D. (3) dan (4)

37. Cermati kalimat berikut!

“Hari ini kita akan bergotong-royong untuk melakukan: penyemprotan disinfektan di


rumah-rumah warga, tempat ibadah, dan pos ronda,” kata Pak Tegar.

Perbaikan penggunaan tanda baca yang tepat pada kalimat tersebut adalah ...
A. “Hari ini kita akan bergotong-royong untuk melakukan: penyemprotan disinfektan
di rumah-rumah warga, tempat ibadah dan pos ronda,” kata Pak Tegar.
B. “Hari ini kita akan bergotong royong untuk melakukan: penyemprotan disinfektan
di rumah-rumah warga, tempat ibadah dan pos ronda,” kata Pak Tegar.
C. “Hari ini kita akan bergotong royong untuk melakukan penyemprotan disinfektan
di rumah-rumah warga, tempat ibadah, dan pos ronda,” kata Pak Tegar.
D. “Hari ini kita akan bergotong-royong untuk melakukan penyemprotan disinfektan
di rumah-rumah warga, tempat ibadah, dan pos ronda!” kata Pak Tegar.

pp_aspd Utama Bahasa Indonesia 13


mkks_smp/mts_provinsi_diy
38. Cermati kalimat berikut!

Dokter rumah sakit Bakti Pertiwi mengatakan bahwa penyakit yang bukan
disebabkan oleh infeksi kuman termasuk penyakit kronis, seperti strok dan
diabetes.

Perbaikan penulisan ejaan yang tepat pada kalimat tersebut adalah ...
A. Dokter rumah Sakit Bakti Pertiwi mengatakan bahwa penyakit yang bukan
disebabkan oleh infeksi kuman termasuk penyakit kronis, seperti strok dan
diabetes.
B. Dokter rumah sakit Bakti Pertiwi mengatakan bahwa penyakit yang bukan
disebabkan oleh infeksi kuman termasuk penyakit kronis, seperti stroke dan
diabetes.
C. Dokter Rumah Sakit Bakti Pertiwi mengatakan bahwa penyakit yang bukan
disebabkan oleh infeksi kuman termasuk penyakit kronis, seperti stroke dan
diabetes.
D. Dokter Rumah sakit Bakti Pertiwi mengatakan bahwa penyakit yang bukan
disebabkan oleh infeksi kuman termasuk penyakit kronis, seperti stroke dan
diabetes.

39. Cermati kalimat berikut!

Sistem kekebalan tubuh memproduksi anti bodi yang mampu mengenali virus jika
menyerang tubuh.

Penulisan anti bodi pada teks tersebut tidak tepat karena....


A. anti diadaptasi dari bahasa asing sehingga harus ditulis serangkai dengan kata
yang mengikutinya.
B. anti merupakan imbuhan asing sehingga harus ditulis serangkai dengan kata
yang mengikutinya.
C. anti merupakan unsur serapan sehingga harus ditulis serangkai dengan kata
yang mengikutinya.
D. anti merupakan bentuk terikat sehingga harus ditulis serangkai dengan kata
yang mengikutinya.

40. Cermati tabel berikut!

Tabel Informasi Nilai Gizi Margarin “Saya Suka”

Informasi Nilai Gizi Jumlah


lemak tidak jenuh tunggal 5 gr
lemak tidak jenuh ganda 2 gr
omega-3 260 mg
omega-6 1360 mg
garam 120 mg
lemak jenuh 4 gr

pp_aspd Utama Bahasa Indonesia 14


mkks_smp/mts_provinsi_diy
Paragraf yang sesuai dengan tabel tersebut adalah ...
A. Kandungan gizi yang terdapat pada margarin “Saya Suka” cukup beragam.
Kandungan gizi lemak tidak jenuh ganda lebih banyak daripada lemak tidak
jenuh tunggal.
B. Kandungan gizi yang terdapat pada margarin “Saya Suka” cukup beragam.
Kandungan gizi yang paling banyak, yaitu lemak jenuh disusul lemak tidak jenuh
tunggal.
C. Kandungan gizi yang terdapat pada margarin “Saya Suka” cukup beragam.
Kandungan gizi yang terbanyak, yaitu lemak tidak jenuh ganda.
D. Kandungan gizi yang terdapat pada margarin “Saya Suka” cukup beragam.
Kandungan omega-6 lebih banyak daripada omega-3.

pp_aspd Utama Bahasa Indonesia 15


mkks_smp/mts_provinsi_diy
SIAP APSD PAKET 1
1. Cermati teks berikut!
Pandemi Covid-19 merupakan realita yang menggemparkan umat manusia di seluruh
muka bumi, termasuk bangsa Indonesia yang sejak awal Maret 2020 telah terpapar wabah
ini. Dalam penyebarannya yang cukup cepat, wabah ini telah mengubah tatanan
kehidupan manusia dalam berbagai sektor, mulai dari masalah ekonomi, kesehatan,
sosial, agama, politik hingga dunia pendidikan. Wabah ini telah menyita pikiran seluruh
lapisan masyarakat untuk bersama-sama berpartisipasi dalam upaya tanggap darurat
bencana..

Makna kata bercetak miring pada teks tersebut adalah....


A. penyakit langka yang belum pernah muncul sebelumnya
B. wabah yang berjangkit secara serempak di mana-mana
C penyakit yang membahayakan kehidupan manusia
D. penyakit yang kemunculannya rutin setiap tahun

2. Cermati teks berikut!

Prakiraan cuaca Jakarta hari ini, Selasa (19/1/2021), sebagian akan diguyur hujan
dengan intensitas berbeda. Dikutip dari informasi cuaca Badan Meteorologi Klimatologi
dan Geofisika (BMKG), pada pagi ini hanya Jakarta Selatan dan Jakarta Timur yang
diprediksi berawan. Sisanya diprediksi hujan dengan intensitas ringan.
Berbeda, siang nanti, kedua wilayah tersebut justru diperkirakan hujan dengan
intensitas ringan. Sisanya cerah berawan dan malam hari, keseluruhan Jakarta diprediksi
diguyur hujan dengan intensitas ringan hingga sedang.

Pernyataan yang sesuai dengan isi teks tersebut adalah...


A. Pada hari Selasa, tanggal 19 Januari 2021, seluruh Jakarta akan diguyur hujan dengan
intensitas tinggi.
B. Informasi cuaca Badan Meteorologi Klimatologi dan Geofisika (BMKG) pada Selasa,
19 Januari 2021 pagi Jakarta Selatan dan Jakarta Timur yang diprediksi hujan deras.
C. Jakarta Barat, Jakarta Pusat, dan Jakarta Utara pada Selasa, 19 Januari 2021 pagi
diprediksi hujan dengan intensitas ringan.
D. Selasa, tanggal 19 januari 2021 siang, diperkirakan wilayah Jakarta Timur dan Jakarta
Selatan berawan.

3. Cermati teks berikut!

(1) "Kenal di mana sama Jasmine?"


(2) "Di bus Trans Jakarta, Tante. Waktu itu saya mau ke kampus. Jasmine nggak kebagian
tempat duduk. Saya berdiri, mempersilakan Jasmine duduk."
Dia anak kuliahan juga! Sok-soknya cari perhatian putriku hanya dengan sebuah kursi!
"Wah, kamu baik sekali..."
"Tapi Jasminenya nolak, Tante."
"Menolak? Kenapa?"
"Kursinya diberikan buat seorang Ibu yang juga berdiri. Saya jadi malu, Tante."
Aku menahan tawa! Apa aku bilang! Mancing perhatian putriku hanya dengan sebuah kursi?
(3) Hah! Kursi bus kota lagi! (4) Lalu aku ambil masker di rak buku depan kamarku.

1
Latar tempat dan bukti latar pada kutipan cerita tersebut adalah …
A. di rumah, bukti latar kalimat (1)
B. di bus, bukti latar kalimat (2)
C. di bus, bukti latar kalimat (3)
D. di rumah, bukti latar kalimat (4)

4. Cermati teks berikut!

Suatu hari seekor unta betina dan anaknya sedang bercengkrama. Bertanya anak kepada
ibunya, “Ibu, bolehkan aku bertanya sesuatu?”
“Tentu, anakku! Adakah sesuatu yang mengganggumu?” tanya ibunya.
Si anak bertanya, “Ibu, mengapa kita memiliki punuk?”
“Oh, itu karena kita adalah hewan gurun, kita memerlukan punuk untuk menyimpan air
sebagai cadangan ketika ada di padang pasir.”
“Oh, begitu, lalu kenapa kaki kita panjang dan bulat?”
“Itu untuk mempermudah kita berjalan di padang pasir. Dengan kaki seperti itu kita bisa
bergerak dengan lebih baik dibanding hewan lainnya.”
“Lalu mengapa kita memiliki bulu mata yang panjang? Sangat mengganggu penglihatan,
ibu”.
Sang ibu menjawab dengan tegas dan bangga bahwa jawabannya bisa memuaskan
keingintahuan anaknya, “Anakku, bulu mata yang panjang itu untuk melindungi mata kita
dari debu dan pasir. Bagaimana anakku, apakah masih ada yang ingin kau ketahui?”
Si anak terdiam sementara sambil berpikir sesuatu.

Komentar yang tepat terhadap isi teks cerita tersebut adalah...


A. Anak unta termasuk anak yang cerewet dan rewel.
B. Pertanyaan anak unta menunjukkan kebodohannya.
C. Semangat anak unta untuk mengetahui sesuatu sangat tinggi.
D. Pertanyaan anak unta tidak semuanya harus dijawab sang ibu.

5. Cermati teks berikut!

(1) Matahari belum naik, tetapi langkahnya harus segera sampai di pasar untuk belanja
bahan masak pagi ini. (2) Apalagi di tengah masa WFH seperti ini, Nisa wajib ke pasar
karena anak-anak dan suaminya berada di rumah. (3) Beli makanan jadi rasanya lebih boros
sehingga dia memilih memasak sendiri. (4) Nisa memilih sayur dan lauk untuk masak hari
ini.
Bukti Nisa berwatak hemat terdapat pada kalimat nomor....
A. (l)
B. (2)
C. (3)
D. (4)

2
6. Cermati teks berikut!

Ah! Sedang PSBB begini, siapa yang bertamu! Kubuka pintu sambil mengenakan masker.
"Selamat malam, Om." Tamu tak diundang itu berdiri sekitar dua meter dari pintu.
Aku mendelik. Rambut gondrong, celana jeans belel. Wajah tertutup masker!
"Jasmine ada, Om?" Santai saja suaranya dan tetap menjaga jarak.
Napasku ngos-ngosan. Tak pernah kurasakan perasaan ini sebelumnya. Setelah 17 tahun
kami hidup tanpa ada orang lain, tiba-tiba ada anak gondrong ingin bertemu dengan
Jasmine! Putriku, yang kujaga dan kurawat dengan telaten! Saat pandemi Covid-19 pula!
"Siapa, ya?"

Makna simbol rambut gondrong, celana jeans belel pada cerita tersebut adalah....
A. pemuda yang malas
B. pemuda ketinggalan zaman
C. pemuda kurang sopan santun
D. pemuda yang tidak berpendidikan

7. Cermati teks petunjuk berikut!


Cara Membuat Tongseng kepala Kambing Sederhana
(1) Rebus kepala kambing dengan menambahkan rempah salam, sereh, daun jeruk dan
lengkuas, sampai empuk.
(2) […]
(3) Ulek halus semua bumbu yang telah disediakan.
(4) Tumis bumbu, daun salam, sereh, dan lengkuas hingga harum lalu masukan santan.
(5) Masukan kepala kambing, beri gula jawa, garam, kecap, dan kaldu bubuk.
(6) Masukan tomat dan tongseng siap di hidangkan

Kalimat yang tepat untuk melengkapi bagian rumpang petunjuk tersebut adalah...
A. Masak terus hingga kepala kambing terlihat matang.
B. Masukkan garam dan gula secukupnya ke dalam air rebusan.
C. Potong kepala kambing yang sudah direbus dan buang tulangnya.
D. Siapkan bahan-bahan dan bumbu-bumbu yang disediakan sebelumnya.

8. Cermati petunjuk berikut!

Petunjuk Masuk Rumah Setelah Bepergian Saat Pandemi Virus Korona


(1) Mandi hingga bersih!
(2) Ganti pakaian yang baru!
(3) Lepas sepatu di pintu, sebelum masuk ke rumah!
(4) Setelah itu, cuci tangan dengan sabun dan air mengalir sampai bersih!
(5) Semprotkan cairan disinfektan pada barang yang dibawa bepergian!
(6) Jangan menyentuh barang apa pun dan jangan langsung beristirahat!

Urutan petunjuk yang tepat adalah....


A. (3), (4), (5), (6), (1), (2)
B. (3), (4), (5), (6), (2), (1)
C. (4), (3), (5), (6), (1), (2)
D. (5), (4), (2), (3), (6), (1)

3
9. Cermati kedua teks berikut!

Teks 1
Studi yang dilakukan oleh Eye Disease Prevalence Research Group (2004) memperkirakan
bahwa pada tahun 2020 jumlah penderita penyakit mata dan kebutaan di dunia akan
mencapai 55 juta jiwa. Hal ini disebabkan penyakit mata dan kebutaan akan meningkat
terutama bagi mereka yang telah berumur di atas 65 tahun. Orang yang berumur 80 tahun
ke atas yang merupakan 8% dari total penduduk, mengalami kebutaan sebanyak 69%.

Teks 2
Seiring dengan meningkatnya perhatian dan pengetahuan pengaruh gizi terhadap
kesehatan, khususnya kesehatan mata menyebabkan pesatnya pertumbuhan pasar terhadap
produk-produk kesehatan, makanan obat, makanan kesehatan, obat kesehatan, suplemen
makanan, suplemen kesehatan, obat herbal, obat alami mata. Bahkan, sebagian besar
produk-produk untuk kesehatan mata yang dipasarkan sekarang berbentuk suplemen,
makanan obat, makanan kesehatan, obat kesehatan, suplemen makanan,
suplemen kesehatan, obat herbal, obat alami.

Perbedaan pola penyajian kedua teks tersebut adalah….


A. Teks 1 urutan sebab-akibat
Teks 2 urutan penambahan
B. Teks 1 urutan peristiwa
Teks 2 urutan waktu
C. Teks 1 urutan sebab-akibat
Teks 2 urutan waktu
D. Teks 1 urutan peristiwa
Teks 2 urutan penambahan

10. Cermati kedua teks berikut!

Teks 1
Jika ada spesies burung yang disebut-sebut menjadi inspirasi dari lambang negara Indonesia,
elang jawa (nisaetus bartelsi) atau javan hawk-eagle merupakan sosok paling meyakinkan
dari sang Garuda. Sosoknya yang gagah dengan jambul panjang di bagian belakang kepala
membuat elang jawa kerap diidentikkan dengan garuda. Garuda dalam kitab Adiparwa
diceritakan merupakan burung gagah berani yang dijadikan kendaraan Dewa Wisnu.

Teks 2
Tuhan tidak perlu diteriakkan karena Dia hadir dalam bisikan-bisikan lirih di sudut sepi
ruang hati. Dia sulit ditemui dalam gegap gempitanya cacian dan hinaan. Tuhan akan hadir
ketika di hati dan pikiran hanya muncul ketulusan dan kepasrahan. Tuhan itu Maha Pengasih
dan Maha Penyayang, bukan dewa perang yang mengajarkan permusuhan dan
kesombongan.

Perbedaan penggunaan bahasa kedua teks tersebut adalah….


A. Teks 1 menggunakan kalimat majemuk
Teks 2 menggunakan kalimat tunggal
B. Teks 1 menggunakan kata sandang
Teks 2 tidak menggunakan kata sandang

4
C. Teks 1 menggunakan kata kata teknis
Teks 2 menggunakan kata kata daerah
D. Teks 1 menggunakan kata ganti
Teks 2 tidak menggunakan kata ganti

11. Cermati kutipan berikut!

Potensi bahaya dari aktivitas […] Gunung Merapi saat ini berupa guguran lava dan awan
panas pada sektor selatan-barat daya meliputi Sungai Boyong, Bedog, Krasak, Bebeng,
dan Putih sejauh 5 km.

lstilah yang tepat untuk melengkapi bagian rumpang kalimat tersebut adalah ....
A. seismograf
B. mitigasi
C. tektonik
D. vulkanik

12. Cermati kalimat berikut!


1.
2. Tidak ada diskriminasi dalam lingkungan kami yang beragam.

Variasi yang dapat menggantikan kata bercetak miring pada kalimat tersebut adalah....
A. persamaan
B. pembedaan
C. perilaku
D. tindak

13. Cermati kalimat berikut!

Hadirin, pada kesempatan ini, perkenankan aku berpidato mengenai perjuangan


Kartini!

Penggunaan kata pada kalimat tersebut tidak tepat karena ....


A. menggunakan kata ganti yang tidak sesuai
B. ketidakhematan menggunakan kata
C. menggunakan kata kerja pasif
D. salah penempatan kata sapaan

14. Cermati kalimat berikut!

Masyarakat harus saling bantu-membantu dalam mengatasi penyebaran virus korona.

Kalimat tersebut tidak efektif karena....


A. mengandung kalimat yang bermakna ganda
B. penggunaan kata tugas yang tidak tepat
C. ketidaklengkapan struktur kalimat
D. penggunaan kata yang berlebih

5
15. Cermati paragraf berikut!
(1) Pesisir Barat merupakan salah satu kabupaten yang ada di Provinsi Lampung
yang menyimpan surga destinasi wisata. (2) Provinsi hasil pemekaran dari kabupaten
Lampung Barat ini merupakan surga bagi para pecinta destinasi wisata pantai. (3) Ikan-
ikan segar banyak terdapat di pasar ikan kota Krui yang didapatkan oleh para nelayan
setempat. (4) Banyak turis domestik maupun mancanegara yang mengunjungi pantai-
pantai yang ada di kabupaten ini untuk menikmati keindahan panorama alamnya ataupun
hendak berselancar.
Paragraf tersebut tidak padu karena...
A. Kalimat (l) membahas destinasi wisata Pesisir Barat, sementara kalimat lainnya
membahas tentang keindahan panorama.
B. Kalimat (2) membahas Pesisir Barat yang merupakan surga bagi pecinta wisata,
sementara kalimat lainnya membahas keindahan alam.
C. Kalimat (3) membahas tentang ikan segar yang dijual di pasar, sementara kalimat
lainnya membahas destinasi wisata di Pesisir Barat.
D. Kalimat (4) membahas tentang turis yang mengunjungi pantai, sementara kalimat
lainnya membahas tentang keindahan alam.

16. Cermati teks rumpang berikut!

Lutein merupakan suatu kelompok senyawa karotenoid berwarna kuning yang banyak
terdapat pada sayuran berwarna hijau. Lutein terdapat di macula dan dipercaya dapat
[…] mata dari [ ...] oksidatif oleh sinar radiasi ultraviolet.

Kata bentukan yang tepat untuk melengkapi paragraf yang rurnpang tersebut adalah....
A. dilindungi, kerusakan
B. melindungi, kerusakan
C. melindungi, perusakan
D. dilindungi, perusakan

17. Cermatilah teks berikut!


Janda bolong merupakan salah satu tanaman hias yang tren selama pandemi Covid-19.
Warna dan bentuk daun tanaman ini menarik masyarakat sehingga janda bolong menjadi
primadona. Tanaman ini merupakan anggota suku araceae atau talas-talasan dari marga
monstera. Nama janda bolong berasal dari bahasa Jawa ron phodo bolong yang berarti
daun yang berlubang-lubang. Jika diucapkan secara singkat, maka terdengar seperti ron
dho bolong . Pengucapan ron dho terdengar hampir sama dengan rondho yang berarti
‘janda’. Nama tersebut menjadi langsung populer di pasar tanaman hias ‘janda bolong’.
Ide pokok paragraf tersebut adalah....
A. klasifikasi tanaman hias janda bolong
B. daya tarik tanaman hias janda bolong
C. keindahan tanaman hias janda bolong
D. tanaman hias yang tren saat pandemi Covid-19

6
18. Bacalah teks berikut!
Virus korona merupakan famili virus yang menyebabkan penyakit mulai dari flu
biasa hingga penyakit yang lebih berat seperti Middle East Respiratory
Syndrome dan Severe Acute Respiratory Syndrome. Pada 11 Februari 2020, WHO
mengumumkan nama virus korona jenis baru tersebut adalah korona virus disease
2019 (disingkat menjadi Covid-19).
Masa inkubasi dapat bervariasi antarpasien, yaitu 2-14 hari setelah terpapar
virus berdasarkan periode inkubasi yang ditunjukkan sebelumnya pada virus
MERS. Masa inkubasi 24 hari telah diamati dalam penelitian terbaru. WHO
mengatakan periode inkubasi yang panjang dapat mencerminkan paparan
ganda koronavirus.

Ringkasan yang tepat sesuai teks tersebut adalah ...


A. Virus korona merupakan famili virus yang menyebabkan penyakit mulai dari flu
biasa hingga penyakit yang lebih berat dengan masa inkubasi dapat bervariasi
antarpasien, yaitu 2-14 hari setelah terpapar virus.
B. Virus korona merupakan famili virus yang menyebabkan penyakit mulai dari flu
biasa hingga penyakit yang lebih berat seperti Middle East Respiratory
Syndrome dan Severe Acute Respiratory Syndrome
C. Pada 11 Februari 2020, WHO mengumumkan nama virus korona jenis baru tersebut
adalah korona virus disease 2019 (disingkat menjadi Covid-19).
D. WHO mengatakan periode inkubasi yang panjang dapat mencerminkan paparan
ganda koronavirus.

19. Cermati teks berikut!

Setiap anak adalah anugerah individu yang unik yang seyogianya berkembang secara
alamiah, sesuai dengan fitrah, usia, psikis, fisik, minat, kepekaan mereka untuk
mempelajari dan menguasai sesuatu yang membutuhkan waktu serta melewati suatu
proses. Untuk itu, diperlukan suatu cara agar anak merasa nyaman dan tidak terbebani
dalam belajar. Misalnya, dapat menggunakan cara belajar secara aktif-positif (active
learning), belajar yang menarik (attractive learning), dan belajar yang menyenangkan
(joyful learning). Itulah sebabnya, dibutuhkan inovasi dalam pendidikan.

Simpulan isi teks tersebut adalah...


A. Anak adalah anugerah yang unik dan harus kita syukuri.
B. Setiap anak adalah anugerah individu yang unik dan alamiah.
C. Inovasi pendidikan perlu agar anak bisa belajar dengan senang.
D. Setiap anak berhak berkembang alamiah, tanpa terbebani dalam belajar.

20. Cermati teks ulasan berikut!


Novel yang berjudul My Idiot Brother karya Agnes Davonar ini memunyai cerita
yang menarik. Kisah suka duka Angel yang mempunyai kakak mengidap penyakit
down sindrom atau idiot dikisahkan dengan lancar. Di samping itu sampul bukunya
juga menarik membuat orang tergiur untuk membacanya. Akan tetapi, di dalam
novel tersebut ada sedikit kata-kata yang agak mengganggu karena berkonotasi
kurang sopan menurut kebudayaan kita.

7
Kekurangan atau kelemahan buku yang terdapat dalam teks ulasan tersebut adalah....
A. kisah novel tentang orang yang mengidap down sindrom
B. sampul buku novel yang membuat pembaca tergiur
C. terdapat beberapa kata yang berkonotasi kurang sopan
D. penggunaan bahasa dalam penceritaan tokoh

21. Cermati teks berikut!

(1) "Ditahan ya, Mak, sakitnya. Kalau tidak diobati, kapan Amak sembuhnya?" bujuk
anak bungsunya. Amak memberengut.
Fatimah meninggalkan Amak di kamar setelah kehabisan akal membujuk sang ibu. Ia
temui kakak-kakaknya yang menunggu di ruang tamu.
"Bagaimana?" tanya kakaknya yang nomor dua, Fitri namanya.
"Amak tidak mau." Fatimah, Fitri, dan empat anak Amak yang lain menatap ke arah
yang entah. (2) Mata mereka basah. Hati mereka gelisah.
"Amak tidak peduli padamu, Amak inginkan Bujang."
Terdengar suara dari kamar. Semua mata saling berpandangan.
(3) Bujang anak kesayangan. Amak memanggilnya `Bujang Sayang' sejak ia dilahirkan,
meski Ayah sudah memberinya nama Rizal. Hari itu Bujang tidak bersama saudaranya
yang lain. Ia di Pekanbaru sejak tiga bulan kemarin. Persis sebelum Amak terbaring
lemah dan berita itu sampai ke rumah.
(4) "Mak, ini Murni. Uda Bujang sudah seminggu tidak pulang. Terakhir ia kirim pesan.
Katanya Uda Bujang sudah mengurus perpisahan dengan Murni di pengadilan." Suara
itu penuh isak dan tangis yang membeludak. Semua yang ada di kamar terpaku bagai
disambar petir.

Bagian klimaks pada kutipan cerita tersebut ditunjukkan pada kalimat nomor ....
A. (1)
B. (2)
C. (3)
D. (4)

22. Cermati teks berikut!


Hiduplah seekor rusa pada zaman dahulu. la sangat sombong dan pemarah. Sering ia
meremehkan kemampuan hewan lain.
Pada suatu hari si Rusa berjalan-jalan di pinggir danau. la bertemu dengan kura-kura
yang terlihat hanya mondar-mandir saja.
"Kura-kura, apa yang sedang Engkau lakukan di sini?"
"Aku sedang mencari sumber penghidupan,” jawab si Kura-kura.
Si Rusa tiba-tiba marah mendengar jawaban si Kura-kura. "Jangan berlagak Engkau,
hei Kura- kura! Engkau hanya mondar-mandir saja namun berlagak tengah mencari
sumber penghidupan!"
Si Kura-kura berusaha menjelaskan, namun si Rusa tetap marah. Bahkan, si Rusa
mengancam akan menginjak tubuh si Kura-kura. Si Kura-kura yang jengkel akhirnya
menantang untuk mengadu kekuatan betis kaki.
Si Rusa sangat marah mendengar tantangan si Kura-kura untuk mengadu betis. la pun
meminta agar si Kura-kura menendang betisnya terlebih dahulu. "Tendanglah
sekeras-kerasnya, semampu yang Engkau bisa lakukan!"

8
Penyebab konflik pada kutipan cerpen tersebut yang paling tepat adalah ...
A. Rusa meremehkan kura-kura.
B. Kura-kura jengkel dan menantang rusa.
C. Rusa marah mendengar jawaban kura-kura.
D. Rusa mengancam akan menginjak kura-kura.

23. Cermati teks cerita berikut!


Bermalam-malam cerita nenek terngiang di telinga. Lalu kuceritakan kepada
teman-teman apa yang kudengar dari nenek. Teman-teman percaya lalu menceritakan
kepada orang tua dan saudaranya.
“Bagaimana kalau kita tangkap kunang-kunang lalu kita masukkan ke botol?” usul
Salim waktu itu.
“Untuk apa?” tanyaku dan beberapa teman lain
“Supaya kita bisa melihat air mata bidadari”jawab Salim
Malam itu kami berlima dengan mudah menangkap puluhan kunang-kunang,
langsung kami masukkan ke dalam botol. Siang hari pulang sekolah kami ingin melihat
air mata bidadari. Gagal. Kami kecewa. Ternyata kunang-kunang itu mati!
“Aku ingin menangkap kunang-kunang lagi, Pak! Teriakku membuyarkan lamunan
ayah.
“Jangan. Tidak boleh, Nuri!” jawab ayahku.

Akibat konflik pada kutipan cerita tersebut yang paling tepat adalah ...
A. Kunang-kunang dalam botol mati.
B. Nuri ingin menangkap kunang-kunang lagi, tetapi tidak boleh.
C. Ayah Nuri buyar lamunannya karena mendengar teriakan Nuri.
D. Nuri dan teman-temannya kecewa karena gagal melihat air mata bidadari.

24. Cermati teks cerita berikut!


Diterapkannya sistem pembelajaran jarak jauh di dunia pendidikan adalah salah
satu cara agar pembelajaran tetap berlangsung.
Dengan pembelajaran jarak jauh, proses belajar mengajar menjadi lebih mudah
dan dapat menghemat waktu dan tenaga. Hal yang masih dirasakan yaitu kesulitan dalam
sarana smartphone dan pembelian kuota karena perekonomian yang tidak mendukung
serta jangkauan sinyal internet demi kelancaran mengikuti pembelajaran jarak jauh.
Sebagai anak bangsa, tentu tidak ingin pendidikan berhenti dan pelajar merupakan
aset bangsa dalam melanjutkan estafet pembangunan ke depan. Covid-19 bukanlah
penghalang dalam pendidikan. Meskipun belajar secara daring, tetapi tidak mematahkan
semangat dalam belajar dan meraih impian.
Simpulan pendapat pro yang tepat pada teks tersebut adalah...
A. Pembelajaran jarak jauh lebih menghemat waktu.
B. Di beberapa tempat mengalami gangguan sinyal.
C. Kesulitan sarana smartphone masih terasa.
D. Pelajar merupakan aset bangsa.

9
25. Cermati kalimat berikut!

Saya harus melegalisir fotokopi ijazah dan memfotokopi rapor untuk diserahkan
kepada wali kelas.

Kata tidak baku pada kalimat tersebut adalah....


A. melegalisir
B. fotokopi
C. ijazah
D. rapor

26. Cermati kalimat berikut!

Sebelum meninggal, Dadang sempat berbisik sesuatu kepada adiknya.

Perbaikan kata bentukan pada kalimat tersebut adalah....


A. dibisiki
B. membisikkan
C. membisik
D. bisik-bisik

27. Cermati kalimat berikut!


Meski nantinya dosis kedua vaksin telah diberikan, ini tidak membuat pasien kebal
sepenuhnya terhadap Covid-19.

Perbaikan kalimat tersebut adalah...


A. Meski dosis kedua vaksin telah diberikan, tidak membuat pasien kebal sepenuhnya
terhadap Covid-19.
B. Meski nantinya dosis kedua vaksin diberikan, ini tidak membuat kebal sepenuhnya
terhadap Covid-19.
C. Meski dosis kedua vaksin telah diberikan, ini tidak membuat kebal sepenuhnya
terhadap Covid-19.
D. Meski dosis kedua telah diberikan, vaksin tidak membuat pasien kebal terhadap
Covid-19.

28. Cermati teks berikut!


(1) Namun, berbeda dengan garuda yang disanjung-sanjung dan menjadi lambang
negara, nasib elang jawa justru miris. (2) Elang jawa memiliki ciri khas berupa jambul
di atas kepala dengan panjang 12 cm. (3) Bird Life International memperkirakan
populasi elang jawa saat ini hanya tersisa sekitar 300 – 500 individu dewasa. (4) Karena
itulah, pemerintah Indonesia menetapkan elang jawa sebagai spesies yang dilindungi.

Perbaikan kalimat (2) agar teks tersebut padu adalah...


A. Burung endemis Jawa ini masuk kategori terancam punah dengan status genting
(endangered).
B. Burung yang memiliki nama ilmiah Spizaetus Bartelsi itu mirip dengan elang
brontok.

10
C. Habitat burung ini hanya sebatas berada di wilayah hutan primer dan daerah
peralihan.
D. Elang jawa dapat ditemukan di hampir seluruh hutan di lereng gunung Pulau
Jawa.

29. Cermati kalimat berikut!

Kurban bencana erupsi Merapi diimbau segera berkumpul dan akan diefakuasi ke
tempat yang aman.

Perbaikan penulisan kata yang tidak tepat pada kalimat tersebut adalah....
A. korban, dievakuasi
B. eurupsi, dihimbau
C. korban, eurupsi
D. dihimbau, dievakuasi

30. Cermati kalimat berikut!

Prof. Dr. Yusril I Mahendra, SH lahir di Lalang Manggar Belitung Timur.

Perbaikan penggunaan tanda baca pada kaJimat tersebut adalah...


A. Prof. Dr. Yusril I. Mahendra, S.H lahir di Lalang Manggar Belitung Timur.
B. Prof. Dr. Yusril I. Mahendra, S.H. lahir di Lalang, Manggar, Belitung Timur.
C. Prof. Dr. Yusril I. Mahendra, S.H. lahir di Lalang, Manggar, Belitung Timur.
D. Prof. Dr. Yusril I. Mahendra, S.H., lahir di Lalang, Manggar, Belitung, Timur.

31. Cermati kalimat berikut!

Anak SD penjual koran itu berkata Tiap hari saya mengaji dan Saya tidak mau
mencuri meskipun saya butuh uang Bang

Perbaikan penulisan kalimat langsung tersebut adalah…


A. Anak SD penjual koran itu berkata, “Tiap hari saya mengaji, dan saya tidak mau
mencuri meskipun saya butuh uang, Bang”
B. Anak SD penjual koran itu berkata, “Tiap hari saya mengaji dan saya tidak mau
mencuri meskipun saya butuh uang, Bang.”
C. Anak SD penjual koran itu berkata, : “Tiap hari saya mengaji dan saya tidak mau
mencuri meskipun saya butuh uang Bang.”
D. “Anak SD penjual koran itu berkata, Tiap hari saya mengaji dan saya tidak mau
mencuri meskipun saya butuh uang Bang.”

11
32. Cermati kalimat berikut!

“Berilah hamba-Mu kesehatan , dan jauhkan aku dari virus korona!” kata Gozali.

Penggunaan tanda baca pada kalimat tersebut tidak tepat karena...


A. Tanda petik (“…”) tidak digunakan untuk mengapit petikan langsung.
B. Tanda titik (.) seharusnya tidak digunakan pada akhir kalimat langsung.
C. Tanda hubung (-) tidak digunakan untuk merangkai kata dengan kata ganti
Tuhan.
D. Tanda koma (, ) tidak digunakan pada kalimat majemuk setara yang menggunakan
konjungsi dan.

33. Cermati kalimat-kalimat berikut!

(1) Banjir merupakan suatu peristiwa yang terjadi saat air dalam jumlah besar
menggenangi sebagian wilayah daratan dalam waktu tertentu.
(2) Faktor tersebut adalah curah hujan yang tinggi atau kerusakan alam..
(3) Musibah banjir memang sangat merugikan.
(4) Banjir terjadi disebabkan oleh beberapa faktor.
(5) Biasanya tempat yang terkena banjir adalah pemukiman padat sepanjang aliran
sungai.

Kalimat-kalimat tersebut akan menjadi teks laporan yang padu dengan urutan....
A. (1), (4), (2), (5), (3)
B. (1), (2), (5), (4), (3)
C. (4), (5), (2), (3), (1)
D. (5), (2), (1), (4), (3)

34. Cermati kalimat-kalimat berikut!

(1) Ia berulang kali menepuk-nepuk pipi adiknya agar segera sadar.


(2) Bagi anak-anak perlu mendapat pengawasan agar aman dalam bermain.
(3) Hari Sumpah Pemuda jatuh pada tanggal 28 Oktober.
(4) Rapat itu membahas hasil USDA.

Kalimat tidak efektif terdapat pada nomor ....


A. (1) dan (2)
B. (2) dan (4)
C. (2) dan (3)
D. (3) dan (4)

12
35. Cermati kalimat berikut!

Zafika : 2007. Lebih Asyik Tanpa UAN. Yogyakarta; LKiS

Perbaikan penggunaan tanda baca yang tepat pada kalimat tersebut adalah....
A. Zafika. 2007. Lebih Asyik Tanpa UAN. Yogyakarta; LKiS.
B. Zafika. 2007: Lebih Asyik Tanpa UAN. Yogyakarta: LKiS.
C. Zafika. 2007, Lebih Asyik Tanpa UAN. Yogyakarta. LKiS.
D. Zafika. 2007. Lebih Asyik Tanpa UAN. Yogyakarta: LKiS.

36. Cermatilah teks berikut!

Ketakutan membuatku tak berdaya. Tubuhku lemas. Dengan paksa kami digiring menuju
rumah kayu dan disidang di sebuah meja kecil. Lelaki berdasi menginterogasi kami berdua.
“Siapa Kalian?”
“Apa maumu?”
“Siapa yang menyuruhmu ke sini?”
Lidahku kelu. Mulutku terkunci. Aku sangat takut. Kucoba melirik Andrian. Dia tak
ubahnya dengan aku. Takut. Kami diam. Lelaki itu semakin geram. Beberapa ancaman
ditujukan ke kami jika tidak menjawab pertanyaannya.
Aku sadar, apa pun alasanku mereka tidak akan percaya. Tampaknya mereka
menyembunyikan sesuatu yang tidak boleh diketahui orang.
“Aku Safitri, Pak. Warga Kampung Becici. Saya terjebak karena mencari daun pegagan
untuk penelitian, Pak.” Aku mencoba berdalih mengapa sampai ke puncak bukit. Aku
ceritakan bahwa aku adalah cucu almarhum Mbah Seno. Ternyata ketika mendengar nama
kakekku, mereka saling berpandangan. Ketegangan di wajahnya mulai turun. Intonasi
kalimatnya tidak lagi tinggi. Aku tidak mengerti. Tapi, setidaknya aku sedikit lega. Mereka
saling berbisik. Sesekali melihat wajah kami. Akhirnya, kami berdua diperkenankan
meninggalkan lokasi Puncak Becici setelah sederet sumpah serapah ditimpakan kepada
kami. Semua peralatan kami diminta. HP, tustel, dan alat tulis yang sempat kami bawa.

Tokoh utama pada kutipan teks cerita tersebut adalah....


A. Safitri
B. Andrian
C. Mbah Seno
D. Lelaki berdasi

37. Cermati paragraf berikut!

Mbak Di takut amarah Ibu Nyai jatuh kepadanya, yang bisa membuat berkah seorang
guru berkurang. Dengan raut wajah panik, Mbak Di menyerahkan amplop putih yang
berisi uang kepada Mbah Muh, berharap segera diterima.
"Emm, ngapunten, Mbah Muh. Saya sudah diamanahi Ibu Nyai. Mohon
diterima, nggih," pinta Mbak Di dengan sedikit terbata-bata.
"Lah, apalagi ini. Enggak usah, Ndhuk. Saya melakukan ini semua untuk kebaikan dan
ketulusan Mbah Yai menuntun Mbah Muh mendapatkan jalan yang lebih baik."

13
Amanat pada kutipan cerita tersebut adalah...
A. Seharusnya kita mengasihi orang yang kekurangan.
B. Kewajiban membalas budi meskipun tidak diminta.
C. Menyelesaikan kewajiban secepat mungkin.
D. Menghargai pilihan sikap orang lain.

38. Cermati kutipan cerpen berikut!

(1) Menjelang beduk magrib, aku duduk-duduk di teras Masjid Raya. (2) Kalian tahu
tidak, Masjid Raya yang ltu lho, ya warna genting biru? (3) Nama lengkapnya Masjid
Raya Pondok Indah. Bila kalian berdiri di halaman Mal Pondok Indah tampak
menaranya. (4) Sekiranya kalian ke Jakarta, main-mainlah ke sana. Mampir salat. Siapa
tahu ketemu aku.

Bukti latar waktu kutipan cerpen tersebut terdapat pada kalimat nomor....
A. (1)
B. (2)
C. (3)
D. (4)

39. Cermati kedua teks berikut!

Teks 1
Satu jam berlalu, kami terdiam dan terhanyut dalam angan masing-masing. Persoalan
yang kami hadapi saat ini benar-benar di luar batas kemampuan. Tapi aku yakin bahwa
Tuhan tidak memberi cobaan melebihi kemampuan hamba-Nya. Namun, Ranti Istriku,
tetap ngotot dan terus-menerus mendesakku. Kami masih tetap terdiam, aku berdiri dan
berjalan ke sana ke mari. “Jangan mondar-mandir, Mas, cepat cari jalan keluar,
kepalaku rasanya mau copot, lagi pula persoalan ini tidak bisa menunggu, harus segera
cari pinjaman karena kalau hujan turun dengan lebat takutmya rumah ini akan rubuh.”

Teks 2
Aku menjauhi trotoar, berjalan beberapa langkah dengan wajah tengadah. Lalu dari
tengah jalan seraya mengatupkan kedua tangan agar membentuk corong di sekitar
mulut, aku berteriak sekeras-keras: “ Lihat saja, aku bisa lebih dari kalian. Aku bisa,
aku pasti bisa!!” Semua orang melihatku, tapi aku tidak peduli. Aku lebih memilih
melanjutkan berjalan menuju rumah dengan mata berkaca-kaca, sambil dalam hati
memohon, “Ibu, tolong bantu aku.” Dengan perlahan-lahan, aku berbisik pada diriku
sendiri.

Perbedaan pola pengembangan kedua teks tersebut adalah ….


A. Teks 1 mendeskripsikan aksi tokoh
Teks 2 menampilkan lokasi kejadian
B. Teks 1 menampilkan lokasi kejadian
Teks 2 pemunculan masalah
C. Teks 1 pemunculan masalah
Teks 2 menampilkan aksi tokoh
D. Teks 1 mendeskripsikan aksi tokoh
Teks 2 pemunculan masalah

14
40. Bacalah kedua teks berikut!

Teks 1
Seorang gadis kecil yang tertidur pulas di kamar yang begitu luas untuk anak kecil
seukurannya. Sang mentari mulai tersenyum menyapa dunia pagi ini ditemani nyanyian
burung terbang ke sana ke mari. Sinar mentari memancarkan cahaya masuk melalui
jendela kecil samping tempat tidur. Udara dingin pun terasa sampai ke dalam tulang-
tulang.
“Kiki, bangun, Nak, sudah siang dan saatnya berangkat ke sekolah!” bisik Ibu ke
telinga Kiki.

Teks 2
Di kaki lembah desa itu hidup salah satu keluarga ayah, ibu, dua orang anak, laki-laki
dan perempuan. Kehidupan keluarga ini sangat sederhana. Ayah dan Ibu setiap hari
bekerja di ladang. Orang tua ini ikhlas bekerja dengan tanggung jawab demi keluarga dan
anak-anaknya. Anak pertama mempunyai sifat sedikit keras kepala, sedangkan anak yang
kedua mempunyai sifat sebaliknya.
Perbedaan penggunaan bahasa kedua teks tersebut adalah…
A. Teks 1 menggunakan makna kias, teks 2 menggunakan bahasa lugas.
B. Teks 1 menggunakan majas, teks 2 menggunakan ungkapan.
C. Teks 1 menggunakan ungkapan, teks 2 menggunakan dialog.
D. Teks 1 menggunakan kata ganti, teks 2 menggunakan majas.

15
Kunci Jawaban TO ASPD Bahasa Indonesia
1. B 21. D
2. C 22. C
3. D 23. B
4. C 24. A
5. C 25. A
6. C 26. B
7. C 27. D
8. A 28. A
9. A 29. A
10. B 30. B
11. D 31. B
12. B 32. D
13. A 33. A
14. D 34. A
15. C 35. D
16. B 36. A
17. D 37. B
18. A 38. A
19. D 39. C
20. C 40. B

16
SOAL ASPD
MKKS KABUPATEN BANTUL
PAKET 2

1. Cermati teks berikut!

Wabah yang sekarang menjadi tren pembicaraan adalah virus korona yang berasal dari
Wuhan, China. Sepanjang sejarah, kehidupan manusia telah dipengaruhi oleh adanya
berbagai macam penyakit menular. Tentunya, krisis Covid-19 saat ini tidak akan menjadi
yang terakhir. Selain korona, bumi kita pernah dilanda penyakit lain, seperti pes, kolera,
HIV, SARS, ebola, flu burung.

Makna kata bercetak miring pada teks tersebut adalah....


A. penyakit yang penularannya melalui kontak langsung dengan penderita
B. penyakit menular yang berjangkit dengan cepat, di daerah yang luas
C. penyakit yang melanda daerah tertentu pada kelompok tertentu
D. penyakit yang terjadi karena unsur pembawaan

2. Cermati teks berikut!


Badan Meteorologi Klimatologi dan Geofisika (BMKG) mencatat, sebagian besar
wilayah telah memasuki musim hujan. Puncak musim hujan akan terjadi pada Januari dan
Februari 2021. BMKG pun meminta masyarakat mewaspadai cuaca ekstrem.
"Saat ini tercatat sebagian besar wilayah Indonesia, yaitu 94 persen dari 342 zona
musim telah memasuki musim hujan," kata Kepala BMKG, Dwikorita Karnawati, dalam
keterangan tulis, Selasa 19 Januari 2021.Sebagian besar wilayah yang sudah memasuki
puncak musim hujan tersebut terutama Jawa, Bali, Sulawesi Selatan, hingga Nusa
Tenggara.

Pernyataan yang sesuai dengan isi teks tersebut adalah...


A. Semua wilayah di Indonesia sudah memasuki musim hujan.
B. Cuaca ekstrem terjadi di wilayah Indonesia sebesar 94 persen.
C. Bulan Januari dan Februari 2021 musim hujan akan mencapai puncaknya.
D. Sulawesi Selatan baru akan memasuki puncak musim hujan pada bulan Februari.

3. Cermati teks berikut!


"Mbah Muh, monggo diminum dulu tehnya," kataku menghidangkan segelas teh dan
kerupuk dalam stoples di atas meja kecil, dekat tempat cuci piring. Aku balas mengangguk,
lalu beranjak pergi. (1) Kulihat dari sudut dapur pondok kerumun santri saling menyeka air
mata dengan punggung tangan. (2) Ada juga yang saling berpelukan meminta dikuatkan.
Saat kuamati lebih teliti, tatapan mereka ke arah Mbah Muh yang sibuk mencuci piring. (3)
Tadi aku sempat mengintip tangan Mbah Muh sedikit pucat karena terlalu lama terendam
air. (4) Jemarinya yang telah keriput jadi tampak layu. (5) Sejak acara 40 hari wafatnya
Mbah Yai, Mbah Muh telah bekerja keras menunaikan tugasnya mencuci piring. (6) Di sela-
sela bekerja, beliau selalu menyempatkan sejenak membaca tahlil di makam Mbah Yai.

Latar suasana pada kutipan cerita tersebut adalah …


A. (1) dan (2)
B. (2) dan (3)
C. (4) dan (5)
D. (5) dan (6)

1|Page
4. Cermati teks berikut!
“Anak muda, perbuatanmu menolong adalah cerminan hatimu yang baik. Tetapi, mengapa
demi menolong seekor kepiting, engkau membiarkan capit kepiting melukaimu hingga sobek
seperti itu?”
“Paman, seekor kepiting memang menggunakan capitnya untuk memegang benda. Dan
saya sedang melatih mengembangkan rasa belas kasih. Maka, saya tidak mempermasalahkan
jari tangan ini terluka asalkan bisa menolong nyawa mahluk lain, walaupun itu hanya seekor
kepiting,” jawab si pertapa muda.dengan kepuasan hati karena telah melatih sikap belas
kasihnya dengan baik.
Mendengar jawaban si pertapa muda, kemudian orang tua itu memungut sebuah ranting.
Ia lantas mengulurkan ranting ke arah kepiting yang terlihat kembali melawan arus sungai.
Segera, si kepiting menangkap ranting itu dengan capitnya.”
“Lihat, Anak muda. Melatih mengembangkan sikap belas kasih memang baik, tetapi harus
pula disertai dengan kebijaksanaan. Bila tujuan kita baik, yakni untuk menolong mahluk lain,
tidak harus dengan cara mengorbankan diri sendiri. Ranting pun bisa kita manfaatkan, bukan?”

Komentar yang tepat terhadap isi teks cerita tersebut adalah...


A. Orang tua tidak perlu menunjukkan kepintaranya supaya yang muda belajar sendiri.
B. Pertapa muda lebih baik tidak usah menolong jika mencelakakan diri sendiri.
C. Melakukan kebaikan atau menolong harus disertai dengan pengetahuan.
D. Menolong perlu juga mempertimbangkan keuntungan dan kerugian.

5. Cermati teks berikut!


(1) Nisa kurang menyukai makanan pedas, apalagi sambal. (2) Perutnya selalu protes jika
mengonsumsi makanan yang bercabai. (3) Namun, untuk memenuhi keinginan suaminya,
Nisa terpaksa membuatnya setiap hari. (4) Meski rasanya tak seenak buatan ibunya, Adi
selalu menagih untuk dibuatkan. (5) Mungkin rasanya sudah mendekati.

Bukti Nisa berwatak setia terdapat pada kalimat nomor....


A. (l)
B. (2)
C. (3)
D. (4)

6. Cermati teks berikut!


"Jasmine memang seperti ayahnya. Selalu memikirkan orang lain ketimbang dirinya."
Kalimat ini sengaja dikeraskan. Aku hampir saja terbatuk.
"Papa?" istriku mengingatkan. "Katanya mau menyegerakan shalat?"
Aku dongkol juga. "Pakai maskernya!" Aku sodorkan kepada istriku. Lalu bergegas masuk
ke ruangan dalam. Tujuanku sekarang ke kamar Jasmine. Shalat Isya bisa ditunda sebentar.
Urusan Jasmine matahariku dan anak sialan ini harus disegerakan juga!

Makna simbol kata matahari yang tepat pada cerita tersebut adalah....
A. penyemangat
B. anak satu-satunya
C. anak kesayangan
D. sumber penghasilan

2|Page
7. Cermati teks petunjuk berikut!

Cara Membuat Masakan Sayur Lodeh Betawi


(1) Tumis bumbu halusnya sampai mengeluarkan bau yang wangi!
(2) Masukan daging tetelan dan masaklah sampai daging tetelan berubah warnanya!
(3) Beri air secukupnya dan masak kembali sampai daging tetelannya empuk!
(4) […]
(5) Tambahkan santan kental dan masak kembali lagi sampai mendidih!
(6) Setelah sayur lodeh Betawi matang, taburkan bawang goreng di atasnya!

Kalimat yang tepat untuk melengkapi bagian rumpang petunjuk tersebut adalah...
A. Masukan bumbu-bumbu dan aduk supaya merata!
B. Setelah tetelan dirasa empuk beri gula secukupnya!
C. Apabila air masakan berkurang, tambahkan lagi secukupnya!
D. Masukan sayurannya dan masak kembali sampai airnya mendidih!

8. Cermati petunjuk berikut!

Cara Menggunakan Sarung Tangan Plastik


(1) Pilihlah sarung tangan yang utuh setelah tangan bersih!
(2) Cuci tangan dengan air mengalir sebelum memakai sarung tangan!
(3) Buanglah sarung tangan kemudian cuci tangan hingga bersih!
(4) Hindari menyentuh wajah selama memakai sarung tangan!
(5) Setelah selesai digunakan, lepaskan sarung tangan dan pastikan tidak
menyentuh kulit!
(6) Pakailah dengan hati-hati agar tidak sobek!

Urutan petunjuk yang tepat adalah....


A. (2), (1), (6), (4), (5), (3)
B. (1), (2), (6), (4), (5), (3)
C. (6), (2), (1), (4), (5), (3)
D. (6), (1), (2), (4), (5), (3)

9. Cermati kedua teks berikut!

Teks 1
Pendidikan moral sudah semestinya diterapkan kembali dalam kegiatan pembelajaran
dewasa ini. Anak-anak zaman sekarang sudah semakin jauh dari nilai moralitas. Hal ini bisa
dilihat dari maraknya kenakalan remaja dan pergaulan bebas yang mereka lakukan. Untuk
itu, pendidikan moral harus kembali diterapkan di dalam proses belajar mengajar anak agar
mereka menjadi anak yang bermoral baik.

Teks 2
Pada tahun yang sama juga dilakukan penelitian oleh Neuringer dan kolega, menunjukkan
kemampuan lutein dan zeaxanthin dalam meningkatkan kesehatan mata. Sedangkan
pengujian pada hewan yang disponsori oleh DSM Nutritional Products Swiss,
menunjukkan bahwa suplementasi atau penambahan lutein dan zeaxanthin meningkatkan
aktivitas antioksidan di dalam darah.

3|Page
Perbedaan pola penyajian kedua teks tersebut adalah….
A. Teks 1 menggunakan pola sebab akibat
Teks 2 menggunakan pola perbandingan
B. Teks 1 menggunakan pola perbandingan
Teks 2 menggunakan pola penambahan
C. Teks 1 menggunakan pola sebab akibat
Teks 2 menggunakan pola penambahan
D. Teks 1 menggunakan pola penambahan
Teks 2 menggunkakan pola sebab-akibat

10. Cermati kedua teks berikut!

Teks 1
(1) Julius Caesar, negarawan dan jenderal perang yang membawa Roma pada kejayaannya,
ternyata punya kesukaan mengoleksi buku. (2) Ketika menaklukkan Mesir, dia memboyong
40 ribu gulungan papirus dari Perpustakaan Aleksandria. (3) Kita tahu bahwa perpustakaan
ini mempunyai koleksi 700 ribu buku. (4) Namun sayang, belum sempat diangkut ke Roma,
semua papyrus itu ikut ludes terbakar saat Caesar membakar Pelabuhan Aleksandria untuk
melindungi tentaranya dari serangan penduduk asli Makedonia.

Teks 2
Perang Jawa pecah pada Rabu, 20 Juli 1825. Awal perang panjang dan melelahkan ini
ditandai penyerbuan dan pembakaran kediaman Diponegoro di Tegalrejo oleh pasukan
gabungan Belanda-Keraton. Salah satu pemicunya, rencana pelebaran jalan yang
memangkas pagar timur kediaman sang Pangeran. Seorang serdadu Belanda mengisahkan
bahwa sang Pangeran lolos dengan mengendarai kudanya.

Perbedaan penggunaan bahasa kedua teks laporan tersebut adalah…

A. Teks 1 tidak menggunakan kalimat majemuk, teks 2 menggunakan kalimat majemuk.


B. Teks 1 menggunakan kata-kata teknis, teks 2 tidak menggunakan kata kata biasa.
C. Teks 1 tidak menggunakan kata sandang, teks 2 menggunakan kata sandang.
D. Teks 1 tidak menggunakan kata ganti , teks 2 menggunakan kata ganti.

11. Cermati kutipan berikut!


Forum Komunikasi Pimpinan Daerah (Forkopimda) Kabupaten Sidoarjo, Jawa Timur,
mengajak masyarakat di kabupaten setempat supaya tidak takut menjalani […] Covid-19
sebagai upaya mencegah penyebaran virus korona jenis baru.

lstilah yang tepat untuk melengkapi bagian rumpang kalimat tersebut adalah ....
A. isolasi
B. karantina
C. sosialisasi
D. vaksinasi

4|Page
12. Cermati kalimat berikut!
Para mahasiswa melakukan autopsi untuk mempelajari organ tubuh manusia.
Variasi yang dapat menggantikan kata bercetak miring pada kalimat tersebut adalah....
A. pemotongan
B. pembedahan
C. pemenggalan
D. pengirisan

13. Cermati kalimat berikut!


“Ayolah, Mutia, daripada melamun, ikutlah dengan kita pergi makan mi ayam!”

Penggunaan kata pada kalimat tersebut tidak tepat karena ....


A. menggunakan kata ganti yang tidak sesuai
B. ketidakhematan menggunakan kata
C. menggunakan kata tidak baku
D. penempatan kata seru yang keliru

14. Cermati kalimat berikut!


Gunung Merapi sering berkali-kali mengeluarkan lava pijar dan awan panas.

Kalimat tersebut tidak efektif karena....


A. mengandung kalimat yang bermakna ganda
B. penggunaan kata tugas yang tidak tepat
C. ketidaklengkapan struktur kalimat
D. penggunaan kata yang berlebih

15. Cermati paragraf berikut!


(1) Nugroho sangat menyukai anime onepiece. (2) Tak hanya menyukai alur ceritanya, ia
juga sangat suka dengan lagu-lagu yang menjadi Original Soundtrack (OST) karya
Eichiro Odha ini. (3) Nugroho juga sangat menyukai buah anggur dan pisang. (5) Ia
mengaku sangat mengagumi beberapa tokoh yang ada dalam cerita anime ini diantaranya
ialah Monkey D. Luffy, Potgas D. Ace, Akagami No Shanks, Trafalgar Law dan lainnya.

Paragraf tersebut tidak padu karena...


A. Kalimat (l) membahas tentang anime onepiece, sementara kalimat lainnya membahas
tentang tokoh-tokoh dalam anime.
B. Kalimat (2) membahas tentang alur cerita dan lagu OST, sementara kalimat lainnya
membahas tokoh-tokoh dalam anime.
C. Kalimat (3) membahas tentang kesukaan Nugroho makan buah, sementara kalimat
lainnya membahas tentang kesukaan terhadap cerita anime onepiece.
D. Kalimat (4) membahas tentang tokoh dalam anime, sementara kalimat lainnya
membahas cerita anime.

5|Page
16. Cermati teks rumpang berikut!
Macula merupakan daerah kecil yang […] jutaan sel yang membantu […] penglihatan yang
tajam untuk membaca atau melihat objek dengan jelas.

Kata bentukan yang tepat untuk melengkapi paragraf rumpang tersebut adalah....
A. menghasilkan, mengandung
B. mengandung, menghasilkan
C. mengandung, menyusun
D. merupakan, pembuatan

17. Cermatilah teks berikut!


Ada beberapa ciri khusus bunga anggrek yang menjadikannya istimewa sebagai tanaman
hias. Anggrek memiliki bunga yang berwarna cerah dan bentuk yang unik. Ciri khusus
ini berguna untuk menarik perhatian organisme lain untuk membantu proses
penyerbukan. Di samping itu, masa berbunganya cukup lama. Bunga anggrek tersusun
majemuk, muncul dari tangkai bunga yang memanjang atau dari ketiak daun. Bunga
anggrek berbentuk simetri bilateral.

Ide pokok paragraf tersebut adalah....


A. ciri khusus bunga anggrek
B. keunikan bentuk bunga anggrek
C. kecerahan warna bunga anggrek
D. kemajemukan susunan bunga anggrek

18. Bacalah teks berikut!


Penularan Covid-19 dari orang ke orang diperkirakan terjadi melalui droplet. Ketika
orang yang terinfeksi batuk atau bersin, mirip dengan bagaimana influenza dan patogen
pernapasan lainnya,virus dapat terhirup ke dalam paru-paru. Penularan Covid-19 dapat
juga terjadi dengan menyentuh permukaan atau objek yang memiliki virus di atasnya dan
kemudian orang tersebut menyentuh mulut, hidung, atau mungkin mata mereka sendiri.
Antibiotik tidak bekerja melawan virus. Antibiotik digunakan hanya untuk melawan
bakteri. Antibiotik tidak boleh digunakan sebagai sarana pencegahan atau pengobatan
infeksi virus. Namun, antibiotik akan diberikan karena infeksi sekunder bakteri mungkin
terjadi saat pasien tersebut dirawat di rumah sakit.

Ringkasan yang tepat sesuai teks tersebut adalah ...


A. Penularan Covid-19 dari orang ke orang diperkirakan terjadi melalui droplet dan
sentuhan permukaan objek, sementara antibiotik tidak bekerja melawan virus tetapi
bakteri.
B. Penularan Covid-19 dari orang ke orang diperkirakan terjadi melalui droplet ketika
orang yang terinfeksi batuk atau bersin, mirip dengan bagaimana influenza dan
patogen pernapasan lainnya yang terhirup ke dalam paru-paru
C. Penularan Covid-19 dapat juga terjadi dengan menyentuh permukaan atau objek yang
memiliki virus di atasnya dan kemudian orang tersebut menyentuh mulut, hidung, atau
mungkin mata mereka sendiri.
D. Antibiotik tidak bekerja melawan virus tetapi antibiotik digunakan hanya untuk
melawan bakteri dan tidak boleh digunakan sebagai sarana pencegahan atau
pengobatan infeksi virus.

6|Page
19. Cermati teks berikut!
Pengertian inovasi pendidikan dapat juga diartikan sebagai metode pendidikan yang
dianjurkan bagi anak usia dini. Metode pendidikan seharusnya merangsang kecerdasan
mejemuk anak balita, karena pada usia ini mereka sedang berada di masa keemasan (golden
age). Kecerdasan Majemuk (Multiple Intelligences) yang diprakarsai oleh Dr. Howard
Gardner, guru besar bidang pendidikan di Harvard University ini terdiri atas delapan
kecerdasan: bahasa atau linguistik, logis dan matematis, spasial (tilik ruang), kinestetik
(jasmani), musikal, interpersonal, intrapersonal, dan naturalis.

Simpulan isi teks tersebut adalah...


A. Inovasi pendidikan merupakan metode pendidikan anak usia dini.
B. Usia emas anak balita harus dimanfaatkan dengan dirangsang sebaik mungkin.
C. Kecerdasan majemuk meliputi delapan jenis kecerdasan mental yang kompleks.
D. Inovasi pendidikan bagi anak usia dini harus mampu merangsang kecerdasan majemuk.

20. Cermati teks ulasan berikut!


Film “Hijrah Cinta” yang dirilis pada 24 Juli 2014 lalu ini mampu memaksa sang istri,
Pipik, mengeluarkan air mata. Semua kenangan dan kisah cinta bersama almarhum
suaminya, ustaz Jefri Al Bukhori alias Uje, bagaikan terulang. Walaupun hanya sebuah
film, Hijrah Cinta mampu menyampaikan lika-liku perjalanan kehidupan Uje dalam
mencari Tuhan. Karakter Uje yang diperankan apik oleh Alfie yang memiliki kemiripan
perawakan dan bentuk wajah hingga suara yang berpotensi membuat penonton makin
merasa tersentuh.

Kekurangan teks ulasan buku tersebut adalah....


A. tidak menjelaskan kekurangan film
B. tidak mengungkapkan isi cerita film
C. kurang menjelaskan garis besar film
D. kurang mengemukakan pemeran film

21. Cermati teks berikut!


(1) "To, setelah kamu selesaikan urusan wisudamu dan lain-lain segeralah kembali ke desa,
bantu Bapak bertani di sawah kita. Bapak dan ibumu sudah tua. Rasanya sudah tidak
sanggup lagi bekerja seperti dulu. Mbakyu-mbakyumu sudah diboyong suami masing-
masing dan tak mungkin kembali ke desa.
(2) Hanya kamu harapan Bapak. Garaplah sawah kita dan beberapa kebun itu. Tinggallah
di sini, Nak!" Demikian ayahnya memelas memohon kepada anak bungsunya itu untuk
kembali ke desanya.
(3) Pikirannya bimbang, campur aduk antara kasihan kepada ayahnya dan obsesi dirinya
menjadi bagian dari dinamika eksekutif di kota besar yang sering dia lihat di televisi-
televisi itu, clubbing, pergaulan komunitas, canda tawa dengan gadis-gadis cantik
manajer menengah perusahaan-perusahaan start up, fashion, dan segala kecanggihan
bahasa gaul membuatnya seperti laron terhadap lampu, terobsesi.
(4) Sementara desanya di Sleman sana dengan hamparan sawah, kebun-kebun, dan sungai-
sungai mengalir terasa sangat membosankan.

7|Page
Bagian konflik pada kutipan cerita tersebut ditunjukkan pada kalimat nomor ....
A. (1)
B. (2)
C. (3)
D. (4)

22. Cermati teks berikut!


Sampai suatu hari, harimau bernama Harim, membuat keributan di rumah Pena. Pena
yang melihat Harim mengacau di rumahnya sangat kasihan pada orang tuanya. Dia harus
segera mengambil tindakan. Pena berusaha mengalihkan perhatia Harim
"He. Harim, keluarlah, kalau kamu jantan kejarlah aku, “ teriak Pena.
Mendengar teriakan itu Harim marah karena ditantang. Harim keluar dan mengejar
Pena yang telah berlari cukup jauh. Sambil berlari Pena berusaha mencari ide untuk
membuat jera Harim. Tidak terasa mereka telah sampai di tengah hutan. Ketika melihat
sumur tua di tengah hutan, Pena pun mendapat ide. la sangat yakin kalau harimau yang
kelihatannya parkasa dan menakutkan belum tentu punya otak cemerlang. Pena berhenti di
tepi sumur.
"Sekarang kamu mau kemana, ha?” kata Harim memamerkan giginya.
"Tunggu dulu Harim! Jika mau memangsaku, kau harus kalahkan dulu temanku yang
hendak menantangmu. Temanku itu bersembunyi dalam sana." Kata Pena sambil menunjuk
pada sumur tua itu.
Kemudian Harim mendekati sumur dan ia segera menunjukkan giginya yang runcing
Tapi alangkah kagetnya Harim, karena hewan yang ada dalam sumur itu mengikuti
gerakannya dengan sangat mirip.

Penyebab konflik pada kutipan cerpen tersebut yang paling tepat adalah ...
A. Harim membuat keributan di rumah Pena.
B. Teman Pena menantang Harim.
C. Harim berusaha mengejar Pena.
D. Pena menantang Harim.

23. Cermatilah teks berikut!


Setiap kali hendak menulis namanya sendiri, tangannya selalu keseleo dan menulis
kata “babi”. Ia jadi dongkol sekali. Ia telah mengunjungi seorang ahi ilmu jiwa, tetapi
tidak mendapatkan hasil yang ia inginkan. Ia juga sudah datang ke depan seorang ulama,
tetapi ia hanya dinasihati supaya beristirahat. Padahal, ia yakin benar bahwa mungkin
sekali ia sedang berubah untuk menjadi gila.
Akhirnya ia datang ke dokter bedah.
“Dokter,” ujarnya dengan terharu, “saya sudah memutuskan untuk berpisah dengan
tangan ini. Ideologi kami tidak sama lagi. Daripada saya bosok dan diganggu terus, lebih
baik saya putuskan sekarang. Syaraf saya tak kuat lagi untuk menerima
pemberontakannya. Saya minta dokter sudi memotong tangan ini.”

Akibat konflik pada kutipan cerita tersebut yang paling tepat adalah ...
A. Tokoh merasa dongkol.
B. Tokoh mendatangi seorang ahli ilmu jiwa.
C. Tokoh mendapatkan nasihat dari ulama agar beristirahat.
D. Tokoh meminta dokter bedah untuk memotong tangannya.

8|Page
24. Cermati teks cerita berikut!
Berdasarkan proses pembentukan antibodi, seseorang masih bisa terpapar Covid-
19 bahkan setelah divaksin. Kemungkinan itu bisa terjadi jika saat hari penyuntikan, orang
tersebut sedang dalam masa inkubasi.
Bisa saja terdapat seseorang yang saat dilakukan vaksinasi sudah terinfeksi
sebelum disuntik atau terpapar dalam masa inkubasi, tapi tidak diketahui kemudian
divaksin. Setelah beberapa hari akan muncul gejala sehingga saat diperiksa positif.
Sebetulnya ini bukan karena vaksin. Tetapi karena memang sudah terpapar sebelumnya
atau masa inkubasi pada saat vaksinasi.
Jika positif korona setelah disuntikkan vaksin, tidak akan mengalami sakit yang
parah. Vaksinasi tidak akan menjamin 100 persen kebal dari paparan virus korona.

Simpulan pendapat pro yang tepat pada teks tersebut adalah...


A. Vaksinasi bisa gagal karena sebelum divaksin seseorang dalam masa inkubasi.
B. Seseorang yang telah divaksinasi tidak mungkin terinfeksi virus korona.
C. Vaksinasi membuat seseorang kebal terhadap paparan virus korona.
D. Antibodi membuat seseorang tidak akan terpapar Covid-19.

25. Cermati kalimat berikut!


Kami diajari berfikir rasional, logis serta kreatif dalam ekstrakurikuler jurnalistik yang
diadakan di sekolah kami.

Kata tidak baku pada kalimat tersebut adalah....


A. kreatif
B. berfikir
C. rasional
D. ektrakurikuler

26. Cermati kalimat berikut!

Kamu harus mau bermaafan kesalahan sahabat karibmu.

Perbaikan kata bentukan pada kalimat tersebut adalah....


A. mempermaafkan
B. memaafkan
C. bermaaf-maafan
D. dimaafkan

27. Cermati kalimat berikut!


Pengertian inovasi pendidikan dapat juga diartikan sebagai metode pendidikan yang
dianjurkan bagi anak usia dini.

Perbaikan kalimat tersebut adalah...


A. Inovasi pendidikan diartikan sebagai metode yang dianjurkan bagi anak usia dini.
B. Inovasi pendidikan diartikan sebagai metode pendidikan yang dianjurkan bagi anak
usia dini.
C. Pengertian inovasi dapat juga diartikan sebagai metode pendidikan yang dianjurkan
bagi anak usia dini.
D. Pengertian inovasi pendidikan diartikan sebagai metode pendidikan yang dianjurkan
bagi anak usia dini.

9|Page
28. Cermati teks berikut!
(1) Lain lagi dengan cenderawasih paradigalla ekor-panjang (Paradigalla carunculata)
yang punya ciri-ciri khusus. (2) Cenderawasih termasuk burung cendrawasih besar dengan
panjang sekitar 37 cm. (3) Keindahan bulu burung satwa ini memang sudah mendunia sejak
dulu. (4) Inilah jenis yang paling sederhana dalam keluarga paradisaeidae, dengan warna
warni bulunya yang terbatas.

Perbaikan kalimat (3) agar teks tersebut padu adalah...


A. Burung ini termasuk burung yang dilindungi di Indonesia.
B. Bulu tubuhnya berwarna hitam dengan ekor panjang dan runcing.
C. Orang papua memanfaatkan bulu-bulunya untuk pembuatan mahkota.
D. Kerajaan Sriwijaya pernah mempersembahkan cenderawasih kepada Kaisar China.

29. Cermati kalimat berikut!


Tenaga kesehatan yang telah mendapatkan izin dipersilakan antri untuk memperoleh
faksin.

Perbaikan penulisan kata yang tidak tepat pada kalimat tersebut adalah....
A. antre, vaksin
B. ijin, vaksin
C. ijin, antre
D. dipersilahkan, vaksin

30. Cermati kalimat berikut!


Tubuh yang sehat dan ideal dapat dicapai melalui makanan yang bergizi seimbang,
olahraga yang tepat dan proporsional, istirahat yang cukup dan berkualitas.

Perbaikan penggunaan tanda baca pada kalimat tersebut adalah...


A. Tubuh yang sehat dan ideal dapat dicapai melalui: makanan yang bergizi seimbang;
olahraga yang tepat dan proporsional; istirahat yang cukup dan berkualitas.
B. Tubuh yang sehat dan ideal dapat dicapai melalui: makanan yang bergizi seimbang,
olahraga yang tepat dan proporsional, istirahat yang cukup dan berkualitas.
C. Tubuh yang sehat dan ideal dapat dicapai melalui makanan yang bergizi seimbang:
olahraga yang tepat dan proporsional: istirahat yang cukup dan berkualitas.
D. Tubuh yang sehat dan ideal dapat dicapai melalui makanan yang bergizi seimbang;
olahraga yang tepat dan proporsional; istirahat yang cukup dan berkualitas.

31. Cermati kalimat berikut!


Kita memerlukan peralatan gambar spidol pensil cat tembok dan kuas.

Tanda baca yang tepat untuk melengkapi kalimat tersebut adalah...


A. Kita memerlukan peralatan gambar, spidol, pensil, cat tembok dan kuas.
B. Kita memerlukan peralatan gambar: spidol, pensil, cat tembok, dan kuas.
C. Kita memerlukan peralatan gambar; spidol, pensil, cat tembok, dan kuas.
D. Kita memerlukan peralatan gambar, spidol, pensil, cat tembok, dan kuas.

10 | P a g e
32. Cermati kalimat berikut!
Ketika Fani tidur, Febiola membuat agar-agar, rujak, dan asinan-bogor.

Penggunaan tanda baca pada kalimat tersebut tidak tepat karena...


A. Tanda koma (,) tidak digunakan setelah anak kalimat yang mendahului induk
kalimat.
B. Tanda koma (,) tidak digunakan pada rincian sebelum konjungsi dan.
C. Tanda hubung (-) seharusnya tidak digunakan pada kata ulang.
D. Tanda hubung (-) tidak digunakan sebelum kata bogor.

33. Cermati kalimat-kalimat berikut!


(1) Faktor utama penyebab banjir adalah ulah manusia itu sendiri.
(2) Hal ini menjadi penyebab terjadinya banjir.
(3) Pembuangan sampah ke sungai mengakibatkan terhambatnya aliran sungai dan air
pun meluap.
(4) Kegiatan seperti penebangan pohon secara liar menyebabkan berkurangnya daerah
resapan air hujan.
(5) Penyebab lain dari banjir adalah membuang sampah sembarangan, terutama di
sungai.

Kalimat-kalimat tersebut akan menjadi teks laporan yang padu dengan urutan ...
A. (1), (4), (2), (5), (3)
B. (1), (2), (5), (4), (3)
C. (4), (5), (2), (3), (1)
D. (5), (2), (1), (4), (3)

34. Cermati kalimat-kalimat berikut!


(1) Pekerjaan pokoknya bergulat dengan buku-buku.
(2) Mahasiswa universitas yang terkenal itu tertangkap tangan sedang menjadi joki ujian.
(3) Pada HUT ke-75 Republik Indonesia kemarin, ia mendapat hadiah mobil dari bank.
(4) Ia mengaku tidak memiliki apa pun selain uang Rp300.000,00.

Kalimat efektif terdapat pada nomor ....


A. (1) dan (3)
B. (2) dan (4)
C. (2) dan (3)
D. (3) dan (4)

35. Cermati kalimat berikut!

Damono, Sapardi Djoko : 2005. Mantra Orang Jawa. Yogyakarta; Indonesia Tera.

Perbaikan penggunaan tanda baca yang tepat pada kalimat tersebut adalah...
A. Damono, Sapardi Djoko. 2005. Mantra Orang Jawa. Yogyakarta; Indonesia Tera.
B. Damono, Sapardi Djoko, 2005. Mantra Orang Jawa,Yogyakarta: Indonesia Tera.
C. Damono, Sapardi Djoko. 2005. Mantra Orang Jawa. Yogyakarta: Indonesia Tera.
D. Damono, Sapardi Djoko, 2005. Mantra Orang Jawa. Yogyakarta, Indonesia Tera.

11 | P a g e
36. Cermatilah teks berikut!
Kata ibuku, pagi adalah hari yang paling dinantikan banyak orang di dunia. Tapi,
aku dan teman-temanku di sini sepakat, kami benci pagi. Karena di waktu pagi, kami
harus berpisah dengan ibu-ibu kami. Berpisah dengan orang yang paling kami sayangi.
Teman-temanku selalu menantikan waktu sore tiba. Atau, kata Mbak Ratih, waktu
senja. Tapi, kami lebih suka menyebut sore saja. Kami tak begitu sering mendengar
orang mengucapkan kata senja. Di waktu pergantian cerah dan gelap itulah, kami
bersuka ria. Karena, ada sepasang tangan cantik yang mengendong kami dan membawa
kami kembali ke rumah.
Kadang, saking tak sabarnya menunggu dijemput ibu, aku menangis. Seperti sore
ini. Mbak Ratih pun selalu tahu apa yang harus dilakukan. Ia memberiku mainan dan
permen agar air mataku tak jatuh lagi.
Tapi, aku tetap menangis. Aku berjanji pada diriku sendiri akan menghentikan
tangisanku jika ibu sudah menjemputku. Kupandangi terus pintu ruangan yang
berwarna-warni dan ada berbagai lukisan-lukisan dan gambar-gambar lucu itu. Ibu
belum juga datang.

Tokoh utama pada kutipan teks cerita tersebut adalah....


A. ibu
B. Ratih
C. aku
D. temanku

37. Cermati paragraf berikut!


"Kenal di mana sama Jasmine?"
"Di bus Trans Jakarta, Tante. Waktu itu saya mau ke kampus. Jasmine nggak kebagian
tempat duduk. Saya berdiri, mempersilakan Jasmine duduk."
"Wah, kamu baik sekali..."
"Tapi Jasminenya nolak, Tante."
"Menolak? Kenapa?"
"Kursinya diberikan buat seorang Ibu yang juga berdiri. Saya jadi malu, Tante."
Aku menahan tawa! Apa aku bilang! Mancing perhatian putriku hanya dengan sebuah
kursi? Hah! Kursi bus kota lagi!

Amanat pada kutipan cerita tersebut adalah...


A. menghormati gadis yang disukai.
B. memiliki rasa malu dalam setiap pergaulan.
C. memberikan duduk kepada gadis yang disukai.
D. mengutamakan dan menghormati orang yang lebih tua.

38. Cermati kutipan cerpen berikut!


(1) Lencho adalah seorang pekerja keras yang bekerja seperti binatang di ladang, tapi
dia masih bisa menulis. (2) Pada hari berikutnya, ketika dini hari, setelah meyakinkan
dirinya bahwa masih ada zat yang melindungi, ia mulai menulis sepucuk surat yang akan
dibawanya sendiri ke kota dan dimasukkan ke pos.
(3) Itu tidak lain adalah surat kepada Tuhan.
“Tuhan,” tulisnya, “kalau Kau tidak menolongku, aku dan keluargaku akan kelaparan
tahun ini. (4) Aku membutuhkan seratus peso untuk menanami kembali ladangku dan
untuk kebutuhan hidup sampai saatnya panen nanti, karena badai es.”

12 | P a g e
Bukti latar waktu kutipan cerpen tersebut terdapat pada kalimat nomor....
A. (1)
B. (2)
C. (3)
D. (4)

39. Cermati paragraf berikut!


Teks 1
Masalahnya Tri tidak mau kembali ke desa dan menggarap sawah, meskipun sebenarnya
dia tidak akan macul seperti Pak Karto dan Mas Ipung, para penyawahnya. Mereka yang
akan memacul dan sudah punya grup penggarap sendiri yang sudah lebih dari 10 tahun
bekerja untuk ayah Tri. Seperti juga ayahnya, nanti Tri hanya akan mejadi pengawas, atur
ini itu dan seterusnya.
Teks 2
Dan tibalah mereka di RSJ pusat kota, avanza itu diparkir tepat di depan pintu masuk, Joe
digiring layaknya tahanan. Begitu sampai di dalam, semua orang terkejut, dengan wajah
‘inikah dajjal yang terkutuk itu’ Joe melirik marah ke sekeliling, seperti banteng
menghadap matador, kedua tangannya yang diborgol bergetar, dokter-dokter yang
menggiringnya mulai cemas, anak itu tepat seperti apa yang dikatakan orangtuanya yaitu
pelawan.

Perbedaan pola pengembangan kedua teks tersebut adalah….


A. teks 1menampilkan lokasi kejadian
teks 2 pemunculan masalah
B. teks 1 menampilkan lokasi kejadian
teks 2 menampilkan aksi tokoh
C. teks 1 pemunculan masalah
teks 2 menampilkan lokasi kejadian
D. teks 1 mendeskripsikan aksi tokoh
teks 2 pemunculan masalah

40. Bacalah kedua teks berikut!


Teks 1
Di suatu sore ditemani hujan rintik-rintik, Kiki sedang menunggu ayah pulang dari kantor.
Rasanya hari ini Kiki kangen dengan ayah. Sudah beberapa jam menunggu di teras rumah,
tapi beliau tak kunjung datang. “Tak seperti biasanya”, kata ibu. Kecemasan juga tampak
di wajah ibu, tetapi ibu berusaha menyembunyikan kecemasannya.

Teks 2
Aku mengendarai motor pamanku menuju sebuah bukit di daerah Lembang. Sewaktu
liburan, aku sering pergi ke tempat itu hanya untuk menikmati warna jingga di sore hari.
Tempat itu kuketahui dari Leony. Dia sering ke sana sendirian sore-sore. Duduk sendirian,
dengar musik sambil berbaring, baca novel favoritnya dan melihat anak kecil main
layangan. Dia sangat suka hal itu. Bahkan dapat aku katakan itu hobinya. Terdengar aneh.

13 | P a g e
Perbedaan bahasa pada kedua teks tersebut adalah…
A. teks 1 menggunakan bahasa lugas
teks 2 menggunakan kata kias
B. teks 1 menggunakan majas
teks 2 menggunakan bahasa lugas
C. teks 1 menggunakan ungkapan
teks 2 menggunakan dialog
D. teks 1 menggunakan kata ganti
teks 2 menggunakan majas

14 | P a g e
Kunci Jawaban TO ASPD Bahasa Indonesia
1. B 21. C
2. C 22. A
3. A 23. D
4. C 24. A
5. C 25. B
6. A 26. B
7. D 27. B
8. A 28. B
9. A 29. A
10. C 30. D
11. D 31. B
12. B 32. D
13. A 33. A
14. D 34. D
15. C 35. C
16. B 36. C
17. A 37. D
18. A 38. D
19. D 39. C
20. A 40. B

15 | P a g e
DINAS PENDIDIKAN KABUPATEN BANTUL
ASESMEN STANDAR PENDIDIKAN DAERAH
MKKS SMP KABUPATEN BANTUL
TAHAP 1
TAHUN PELAJARAN 2020/2021

Mata Pelajaran : Bahasa Indonesia


Kelas : IX (Sembilan) PAKET 1
Hari dan Tanggal : Senin, 15 Februari 2021
Pukul : 07.30 – 09.00 (90 Menit)

PETUNJUK UMUM
1. Isikan identitas Anda dengan benar
2. Tersedia waktu 90 menit untuk mengerjakan paket tes tersebut.
3. Jumlah soal sebanyak 40 butir, pada setiap butir soal terdapat 4 (empat)
pilihan jawaban.
4. Periksa dan bacalah soal-soal sebelum Anda menjawabnya dengan cermat.
5. Periksalah pekerjaan Anda sebelum dikirim / submit
6. Berdoalah sebelum mengerjakan

PETUNJUK KHUSUS
Jawablah dengan menghitamkan bulatan ( ) huruf A, B, C, atau D pada lembar
jawab yang tersedia!

1. Cermati teks berikut!

Di Kabupaten Administratitif Kepulauan Seribu terdapat zona konservasi berupa


taman nasional laut bernama Taman Nasional Kepulauan Seribu (TNKS). Sebagai
daerah yang sebagian besar wilayahnya merupakan perairan dan di dalamnya juga
terdapat zona konservasi, maka tidaklah mengherankan bilamana pengembangan
wilayah kabupaten ini lebih ditekankan pada pengembangan budidaya laut dan
pariwisata. Dua sektor ini diharapkan menjadi prime-mover pembangunan masyarakat
dan wilayah Kabupaten Administratif Kepulauan Seribu.

Makna kata konservasi pada teks tersebut adalah ….


A. pemanfaatan
B. pemeliharaan
C. pengelolaan
D. pelestarian

Asesmen Standar Pendidikan Daerah Kelas. 9 Paket-1 Bahasa Indonesia 1


(ASPD)
2. Cermati kalimat berikut!

Tumbuhan zuriat berasal dari beberapa negara di Timur Tengah, seperti Israel, Sinai,
Yaman, dan Saudi Arabia. Zuriat juga ditanam di Antiles dan Karibia. Habitat aslinya,
banyak tumbuh di sekitar sungai yang mengering dan oase. Namun, terkadang juga
tumbuh jauh dari perairan, bahkan di perbukitan berbatu. Tanaman zuriat sangat tidak
menyukai tanah tergenang air dan sangat rentan terhadap kebakaran hutan.Selain di
Timur Tengah, zuriat juga berasal dari beberapa negara benua Afrika bagian utara.
Tanaman ini tersebar di Sahel dan ditanam mulai dari Mauritania hingga Senegal di
bagian barat, kemudian Afrika Tengah dan ke arah timur di Mesir, Kenya, dan
Tanzania.

Pernyataan yang sesuai dengan isi teks tersebut adalah …


A. Tumbuhan zuriat banyak tumbuh di habitat aslinya yaitu perbukitan berbatu yang
jauh dari oase dan perairan karena zuriat menyukai tanah yang tidak tergenang air.
B. Beberapa negara di Benua Afrika bagian utara juga merupakan negara asal
tumbuhan zuriat dan penyebarannya dari Sahel hingga Senegal di bagian barat.
C. Tanah yang tergenang air sangat tidak disukai zuriat sehingga tumbuhan ini
habitat aslinya banyak tumbuh di oase dan sekitar sungai yang mengering.
D. Penyebaran tumbuhan zuriat meliputi beberapa negara di Timur Tengah seperti
Antiles dan Karibia, Afrika bagian utara,barat, tengah ,dan timur.

3. Cermati teks berikut!

(1) Di dekat jembatan Ratulangi Mbok Aisah bergidik. (2) Aroma-aroma tak sedap
serta bayangan kabar kecelakaan waktu itu menyambutnya. (3) Wajah Mbok Aisah
mulai pasi. (4) Rintik gerimis membuatnya menggigil. (5) Kakinya lelah menapaki
jalan aspal yang berlubang. (6) Dari arah belakang sorot lampu mobil berkedip-kedip.
(7) Mbok Aisah menoleh, sempat ragu karena rumor hantu bus Karona tiba-tiba
melintas di kepalanya. (8) Kebanyakan berpikir, Mbok Aisah nekat menyetop. (9)
Yang penting bisa pulang.

Bukti latar suasana menakutkan pada kutipan cerita tersebut adalah ….


A. (1), (2), (3)
B. (2), (3), (5)
C. (3), (6), (8)
D. (4), (7), (9)

Asesmen Standar Pendidikan Daerah Kelas. 9 Paket-1 Bahasa Indonesia 2


(ASPD)
Cermati teks berikut kemudian kerjakan soalnya

(1) Pasangan petani memiliki luwak sebagai hewan peliharaan. (2) Suatu hari, petani dan
istrinya harus segera keluar rumah untuk bekerja, dan mereka meninggalkan luwak
dengan bayi mereka. (3) Mereka yakin bahwa si luwak akan menjaga bayi mereka dengan
baik. (4) Sementara mereka pergi, seekor ular diam-diam memasuki rumah dan bergerak
menuju tempat tidur untuk menyerang bayi itu. (5) Si luwak yang pintar bertarung
akhirnya membunuh ular itu untuk melindungi bayi itu. (6) Ketika istri petani itu kembali
ke rumah, dia terkejut disambut oleh noda darah di mulut dan gigi luwak itu. (7) Dia
kehilangan kesabaran dan berteriak, “Kamu membunuh bayiku!” (8) Dalam amarahnya,
dia kehilangan semua kendali dan membunuh luwak yang setia. (9) Ketika dia memasuki
rumahnya, dia melihat bayinya masih hidup, dan ular mati di sampingnya. (10) Dia
menyadari apa yang terjadi dan menyesali tindakannya.

4. Bukti luwak memiliki watak tanggung jawab dan setia terdapat pada kalimat nomor....
A. (3) dan (5)
B. (5) dan (8)
C. (6) dan (9)
D. (7) dan (10)

Cermati teks berikut kemudian kerjakan soalnya

(1) Pasangan petani memiliki luwak sebagai hewan peliharaan. (2) Suatu hari, petani dan
istrinya harus segera keluar rumah untuk bekerja, dan mereka meninggalkan luwak
dengan bayi mereka. (3) Mereka yakin bahwa si luwak akan menjaga bayi mereka
dengan baik. (4) Sementara mereka pergi, seekor ular diam-diam memasuki rumah dan
bergerak menuju tempat tidur untuk menyerang bayi itu. (5) Si luwak yang pintar
bertarung akhirnya membunuh ular itu untuk melindungi bayi itu. (6) Ketika istri petani
itu kembali ke rumah, dia terkejut disambut oleh noda darah di mulut dan gigi luwak itu.
(7) Dia kehilangan kesabaran dan berteriak, “Kamu membunuh bayiku!” (8) Dalam
amarahnya, dia kehilangan semua kendali dan membunuh luwak yang setia. (9) Ketika
dia memasuki rumahnya, dia melihat bayinya masih hidup, dan ular mati di sampingnya.
(10) Dia menyadari apa yang terjadi dan menyesali tindakannya.

5. Nilai moral pada kutipan cerita tersebut adalah...


A. Sebaiknya menghindari sifat tergesa-gesa
B. Hindari kemarahan bila ada masalah
C. Bersabarlah menghadapi masalah
D. Berpikirlah sebelum bertindak

Asesmen Standar Pendidikan Daerah Kelas. 9 Paket-1 Bahasa Indonesia 3


(ASPD)
Cermati teks berikut kemudian kerjakan soalnya

(1) Pasangan petani memiliki luwak sebagai hewan peliharaan. (2) Suatu hari, petani dan
istrinya harus segera keluar rumah untuk bekerja, dan mereka meninggalkan luwak
dengan bayi mereka. (3) Mereka yakin bahwa si luwak akan menjaga bayi mereka
dengan baik. (4) Sementara mereka pergi, seekor ular diam-diam memasuki rumah dan
bergerak menuju tempat tidur untuk menyerang bayi itu. (5) Si luwak yang pintar
bertarung akhirnya membunuh ular itu untuk melindungi bayi itu. (6) Ketika istri petani
itu kembali ke rumah, dia terkejut disambut oleh noda darah di mulut dan gigi luwak itu.
(7) Dia kehilangan kesabaran dan berteriak, “Kamu membunuh bayiku!” (8) Dalam
amarahnya, dia kehilangan semua kendali dan membunuh luwak yang setia. (9) Ketika
dia memasuki rumahnya, dia melihat bayinya masih hidup, dan ular mati di sampingnya.
(10) Dia menyadari apa yang terjadi dan menyesali tindakannya.

6. Makna simbol kata ular yang tepat pada cerita tersebut adalah....
A. ancaman tersembunyi
B. kesembuhan
C. perubahan
D. kelicikan

7. Cermati petunjuk berikut!

Cara Membuat Keranjang dari Koran

(1) Persiapkan alat dan bahan


(2) Potong koran memanjang dengan dimensi yang sama
(3) […]
(4) Rakit gulungan menjadi bentuk keranjang
(5) Lekatkan lem dan ikat keranjang memakai tali
(6) keranjang dari koran siap dipakai

Kalimat yang tepat untuk melengkapi bagian rumpang petunjuk tersebut adalah...
A. Lipat koran dalam bentuk gulungan
B. Gulung potongan koran seperti rotan
C. Buatlah pola pada gulungan koran
D. Potong sesuai ukuran

Asesmen Standar Pendidikan Daerah Kelas. 9 Paket-1 Bahasa Indonesia 4


(ASPD)
8. Cermati petunjuk berikut ini!

Cara Membuat Serundeng Empal Daging

(1) Masukkan daging sapi dalam rendaman bumbu selama 20 menit


(2) Setelah matang angkat dan iris menjadi 8 potong
(3) Campur bersama daging goreng lalu angkat dan sajikan
(4) Goreng daging sapi yang sudah direndam
(5) Rebus daging sapi dan beri garam sedikit
(6) Pukul menggunakan alat pemukul daging
(7) Gongseng kelapa parut dengan sisa rendaman bumbu sampai kecoklatan

Urutan petunjuk yang tepat adalah ….


A. (5), (1), (6), (2), (7), (4), (3)
B. (5), (2), (6), (1), (4), (7), (3)
C. (5), (6), (2), (4), (1), (3), (7)
D. (5), (2), (1), (6), (7), (4), (3)

9. Cermati data laporan berikut!

Objek pengamatan : Gunung Merapi


Tujuan : Memaparkan kondisi perkembangan Gunung Merapi
Tempat : Kaliurang, Yogyakarta
Waktu pengamatan : Kamis, 28 Januari 2021
Data :
- Pukul 12.00-18.00 WIB tercatat 30 kali gempa
- Pukul 10.13 WIB luncuran awan panas jarak 2000 meter ke arah barat daya
- Status level III atau siaga
- Mengalir ke Kali Krasak dan Boyong
- Waspada lahar dingin saat hujan turun

Laporan yang tepat sesuai data tersebut adalah …


A. Dengan jarak luncur 2000 meter ke arah barat daya, masyarakat yang tinggal di
dekat Kali Krasak dan Boyong harus waspada terhadap lahar dingin saat hujan.
Hal ini disebabkan karena Kamis 28 Januari telah terjadi 30 kali gempa dan pada
saaat yang sama terjadi luncuran awan panas.
B. Sampai dengan hari ini Kamis, 28 Januari 2021 perkembangan Merapi masih
status waspada. Pukul 12.00 terjadi luncuran awan panas dengan jarak luncur
2000 meter dan tercatat 30 kali gempa. Masyarakat diminta waspada saat hujan
turun karena lahar akan mengalir ke Kali Krasak dan Boyong.
C. Masyarakat diminta waspada terhadap lahar dingin yang mengalir ke Kali Krasak
dan Boyong. Hal ini terjadi karena pada pukul 12.00 terjadi luncuran awan panas
dan pada pukul 10.30 tercatat 30 kali gempa. Karena berstatus level III atau status
siaga, masyarakat diimbau untuk berhati-hati bila hujan turun.
D. Kamis, 28 Januari 2021 Merapi berstatus siaga atau level III. Pada pukul 10.13
terjadi luncuran awas panas dengan jarak luncur 2000 meter ke arah barat daya
menuju Kali Krasak dan Boyong, sedangkan pukul 12.00-18.00 tercatat 30 kali
gempa. Masyarakat diminta waspada lahar dingin saat hujan turun.

Asesmen Standar Pendidikan Daerah Kelas. 9 Paket-1 Bahasa Indonesia 5


(ASPD)
10. Cermati kedua teks berikut, kemudian kerjakan soalnya.

Teks I
Pasir Berbisik merupakan salah satu keunikan lain dari Taman Nasional Bromo yaitu
berupa hamparan pasir yang harus Anda kunjungi. Hamparan pasir ini sangat luas
dengan pemandangan alam yang luar biasa. Kawasan ini berada di bagian timur dari
kawah Bromo. Ketinggian tempat ini sekitar 2.000 m di atas permukaan laut. Pasir
Berbisik sebenarnya merupakan kaldera raksasa, bagian dari kawasan pegunungan
Tengger yang diameternya sekitar 8-10 k m.

Teks II
Jalur Tretes adalah salah satu jalur pendakian menuju Gunung Arjuna yang
membutuhkan waktu sekitar 7-8 jam. Jalur ini melewati objek wisata air terjun Kakek
Bodo. Selain itu, ada beberapa jalur pendakian seperti jalur Lawang, Purwosari, dan
Batu. Jalur Lawang menyajikan destinasi wisata seperti Oro-Oro Ombo, alas Lali
Jiwo dan perkebunan teh. Sementara itu, jalur Purwosari merupakan jalur yang
memacu edrenalin kita karena melewati Gua Abiyasa, Petilasan Eyang Abiyasa, dan
Situs Eyang Saktri. Berbeda dengan jalur lainnya, jalur Batu merupakan jalur
pendakian terbaik karena banyak menyajikan panorama alam sekitar yang memesona.
Itulah beberapa jalur pendakian yang bisa Anda pilih menuju puncak Gunung Arjuna.

Perbedaan pola penyajian kedua teks tersebut adalah ….


A. urutan umum-khusus, urutan khusus-umum
B. urutan umum-khusus, urutan tempat
C. urutan peristiwa, urutan sebab-akibat
D. urutan tempat, urutan khusus-umum

Asesmen Standar Pendidikan Daerah Kelas. 9 Paket-1 Bahasa Indonesia 6


(ASPD)
11. Cermati kedua teks berikut, kemudian kerjakan soalnya.

Teks I
Pasir Berbisik merupakan salah satu keunikan lain dari Taman Nasional Bromo yaitu
berupa hamparan pasir yang harus Anda kunjungi. Hamparan pasir ini sangat luas
dengan pemandangan alam yang luar biasa. Kawasan ini berada di bagian timur dari
kawah Bromo. Ketinggian tempat ini sekitar 2.000 m di atas permukaan laut. Pasir
Berbisik sebenarnya merupakan kaldera raksasa, bagian dari kawasan pegunungan
Tengger yang diameternya sekitar 8-10 k m.

Teks II
Jalur Tretes adalah salah satu jalur pendakian menuju Gunung Arjuna yang
membutuhkan waktu sekitar 7-8 jam. Jalur ini melewati objek wisata air terjun Kakek
Bodo. Selain itu, ada beberapa jalur pendakian seperti jalur Lawang, Purwosari, dan
Batu. Jalur Lawang menyajikan destinasi wisata seperti Oro-Oro Ombo, alas Lali
Jiwo dan perkebunan teh. Sementara itu, jalur Purwosari merupakan jalur yang
memacu edrenalin kita karena melewati Gua Abiyasa, Petilasan Eyang Abiyasa, dan
Situs Eyang Saktri. Berbeda dengan jalur lainnya, jalur Batu merupakan jalur
pendakian terbaik karena banyak menyajikan panorama alam sekitar yang memesona.
Itulah beberapa jalur pendakian yang bisa Anda pilih menuju puncak Gunung Arjuna.

Perbedan penggunaan bahasa kedua teks tersebut adalah ….


A. menggunakan kata tunjuk, tidak menggunakan kata tunjuk
B. menggunakan kalimat aktif, menggunakan kalimat pasif
C. menggunakan kata depan, tidak menggunakan kata depan
D. menggunakan istilah, tidak menggunakan istilah

12. Terjadinya transformasi pemerintahan Kerajaan Buton menjadi Kesultanan Buton


sebagai pembaharuan, ditandai dengan dilantiknya Lakilaponto sebagai Sultan Buton
I dengan Gelar Sultan Murhum Kaimuddin Khalifatul Khamis

Variasi yang dapat menggantikan kata transformasi pada kalimat tersebut adalah
A. pergeseran
B. perubahan
C. perbaikan
D. perbedaan

Asesmen Standar Pendidikan Daerah Kelas. 9 Paket-1 Bahasa Indonesia 7


(ASPD)
13. Cermati kalimat berikut!

Zelia de Carvalho Morley, seorang nenek berusia 106 tahun, mendapatkan prioritas
utama pemberian vaksin covid-19 di Brasil.

Variasi kalimat tersebut yang tepat adalah...


A. Prioritas utama pemberian vaksin covid-19 di Brasil didapatkan oleh Zelia de
Carvalho Morley, seorang nenek berusia 106 tahun.
B. Seorang nenek berusia 106 tahun, Zelia de Carvalho Morley, memprioritaskan diri
mendapatkan pemberian vaksin covid-19 di Brasil.
C. Pemberian vaksin covid-19 di Brasil memprioritaskan utama oleh seorang nenek
berusia 106 tahun, Zelia de Carvalho Morley.
D. Di Brasil seorang nenek berusia 106 tahun, Zelia de Carvalho Morley, didapati
prioritas utama pemberian vaksin covid-19.

14. Cermati kalimat berikut!

Tim Universitas Gadjah Mada atau UGM Yogyakarta membuat terobosan penting
dengan memperkenalkan inovasi alat deteksi covid-19, GeNose C19.

Kalimat tersebut tidak efektif karena ….


A. penggunaan kata tugas yang tidak tepat
B. penggunaan pilihan kata tidak tepat
C. penggunaan kata yang berlebih
D. kalimat bermakna ganda

15. Cermati paragraf berikut!

(1) Sejak pandemi covid-19 melanda, dunia pendidikan mengalihkan proses belajar
mengajar dari sekolah ke rumah untuk memutus mata rantai penyebaran virus corona.
(2) Meski banyak kendala yang dihadapi, satuan pendidikan mulai terbiasa
menyelenggarakan BDR. (3) Pelaksanaan PJJ menimbulkan reaksi pro dan kontra. (4)
Metode BDR ada dua, yaitu Pembelajaran Jarak Jauh Dalam Jaringan (PJJ Daring)
dan PPJ Luar Jaringan (Luring).

Paragraf tersebut tidak padu karena ….


A. Kalimat (1) membahas pengalihan proses belajar mengajar dari sekolah ke rumah,
sedangkan kalimat lainnya membahas pelaksanaan PJJ Daring dan Luring.
B. Kalimat (2) membahas satuan pendidikan terbiasa melaksanakan proses BDR,
sedangkan kalimat lainnya membahas proses PJJ Daring dan Luring.
C. Kalimat (3) membahas pelaksanaan PJJ yang menimbulkan reaksi pro dan kontra,
sedangkan kalimat lainnya membahas BDR.
D. Kalimat (4) membahas perbedaan PJJ Daring dan Luring, sedangkan kalimat
lainnya membahas kendala dalam penerapan BDR.

Asesmen Standar Pendidikan Daerah Kelas. 9 Paket-1 Bahasa Indonesia 8


(ASPD)
Cermati teks berikut, kemudian kerjakan soalnya

Pembelajaran Jarak Jauh (PJJ) diterapkan oleh Kemendikbud sebagai upaya


percepatan penanganan covid-19 dalam dunia pendidikan. PJJ dinilai sebagai solusi
yang tepat untuk tetap menjalankan proses pembelajaran di tengah pandemi. Karena
kondisi tersebut, hanya ada dua pilihan, melakukan pembelajaran jauh jauh meskipun
tidak optimal atau tidak melaksanakan pembelajaran sama sekali.
Namun, tidak bisa dipungkiri PJJ yang diterapkan mengalami banyak
tantangan dan hambatan. Hambatan yang dihadapi seperti keterbatasan fasilitas,
sulitnya jaringan internet di beberapa daerah, pembelian kuota internet, keefektifan
pembelajaran yang mulai dipertanyakan, hingga dampak psikologis pelajar jika PJJ
dilaksanakan dalam jangka panjang. Oleh karena itu, PJJ saat ini mendapat perspektif
yang kurang positif dari masyarakat, baik pelajar, pendidik, bahkan pihak sekolah.

16. Ide pokok paragraf pertama adalah ….


A. penerapan PJJ sebagai upaya penanganan covid-19
B. solusi penangangan covid-19 oleh Kemendikbud
C. percepatan penanganan covid-19 dalam dunia pendidikan
D. dua pilihan penerapan pembelajaran pada masa pandemi

Cermati teks berikut, kemudian kerjakan soalnya

Pembelajaran Jarak Jauh (PJJ) diterapkan oleh Kemendikbud sebagai upaya


percepatan penanganan covid-19 dalam dunia pendidikan. PJJ dinilai sebagai solusi
yang tepat untuk tetap menjalankan proses pembelajaran di tengah pandemi. Karena
kondisi tersebut, hanya ada dua pilihan, melakukan pembelajaran jauh jauh meskipun
tidak optimal atau tidak melaksanakan pembelajaran sama sekali.
Namun, tidak bisa dipungkiri PJJ yang diterapkan mengalami banyak
tantangan dan hambatan. Hambatan yang dihadapi seperti keterbatasan fasilitas,
sulitnya jaringan internet di beberapa daerah, pembelian kuota internet, keefektifan
pembelajaran yang mulai dipertanyakan, hingga dampak psikologis pelajar jika PJJ
dilaksanakan dalam jangka panjang. Oleh karena itu, PJJ saat ini mendapat perspektif
yang kurang positif dari masyarakat, baik pelajar, pendidik, bahkan pihak sekolah.

17. Ringkasan teks tersebut adalah …


A. Pembelajaran Jarak Jauh dinilai sebagai solusi penanganan covid-19 meski dalam
penerapannya banyak mengalami tantangan dan hambatan.
B. Penerapan PJJ oleh Kemendikbud sebagai upaya penanganan covid-19 dalam
dunia pendidikan mendapatkan perspektif yang kurang positif.
C. Keterbatan fasilitas dan jaringan internet di beberapa daerah menyebabkan proses
pembelajaran jarak jauh tidak bisa berjalan secara optimal
D. Karena kondisi pandemi, Kemendikbud menerapkan dua pilihan dalam proses
pembelajaran sehingga banyak mengalami hambatan.

Asesmen Standar Pendidikan Daerah Kelas. 9 Paket-1 Bahasa Indonesia 9


(ASPD)
18. Cermati teks berikut!

Pemerintah Indonesia sejauh ini belum mengumumkan seperti apa pendistribusian


vaksin covid-19 yang akan dibagikan secara gratis kepada seluruh masyarakat. Hal ini
disebabkan pemerintah belum menentukan secara resmi jenis vaksin yang akan
digunakan. Seorang pakar kesehatan mengingatkan bahwa pemerintah harus
memperhatikan beberapa faktor yang dapat menjadi kendala pendistribusian vaksin,
mulai fasilitas penyimpanan yang harus memenuhi syarat hingga masalah geografi
wilayah Indonesia yang luas.

Simpulan isi teks tersebut adalah …


A. Pendistribusian vaksin covid-19 oleh pemerintah harus mempertimbangkan
beberapa kendala yang dihadapi.
B. Pemerintah harus segera mengumumkan jenis vaksin covid-19 yang akan
diberikan kepada masyarakat.
C. Beberapa faktor yang berpengaruh terhadap pendistribusian vaksin covid-19 harus
segera diatasi pemerintah.
D. Pendistribusian vaksin covid-19 terkendala banyak faktor karena itu harus segera
diatasi Bersama.

19. Cermatilah teks berikut!

Rencana pembangunan megaproyek jembatan yang menghubungkan Pulau Jawa dan


Pulau Sumatra diharapkan tidak sampai menelantarkan ekosistem hutan yang ada di
sekitar lokasi pembangunan. Pembangunan jembatan tersebut kemungkinan besar
akan mengonversi lahan pertanian di Banten serta berpotensi pada pembabatan hutan
dan perusakan lingkungan di Sumatra. Hendaknya pemerintah mengadakan
kesepakatan kerjasama secara terbuka dengan masyarakat untuk tidak merusak hutan,
mempertahankan lahan pertanian, dan memperhatikan industri maritim.
Komentar yang tepat berdasarkan isi teks tersebut adalah …
A. Pembangunan yang berhubungan dengan fasilitas umum memang banyak
risikonya.
B. Proyek pembangunan akan merusak ekosistem yang terdapat di hutan dan
menggusur lahan pertanian.
C. Pemerintah dan masyarakat harus mengadakan kesepakatan agar tidak terjadi
permasalahan di kemudian hari.
D. Pembangunan jembatan penghubung harus memperhatikan kelestarian dan
keberadaan lingkungan sekitar.

Asesmen Standar Pendidikan Daerah Kelas. 9 Paket-1 Bahasa Indonesia 10


(ASPD)
20. Cermati data buku berikut!

Judul : Generasi Langgas (Mileneals Indonesia)


Pengarang : Yoris Sebastian, Dilla Amran, Youth Lab
Penerbit : Gagas Media
Tahun terbit : 2016
Kelebihan : gaya penulisan sopan, penuh kata motivasi, cover unik
Kelemahan : ada kata yang ditipo dan buku tebal
Tebal buku : 194 halaman

Teks ulasan yang tepat sesuai data buku tersebut adalah …


A. Buku ini menceritakan tentang generasi milenials di Indonesia yang lahir tabun
1980 hingga 2000. Kelebihan buku ini terletak pada gaya penulisannya yang
unik,bahasa yang sopan penuh motivasi, dengan cover yang menarik. Namun
sayang buku ini terlalu tebal sehingga membosankan.
B. Buku terbitan Gagas Media ini memberikan pandangan berbeda bahwa generasi
milenials ini merupakan generasi terbaik yang ada di Indonesia. Kelemahan dari
buku ini terdapat banyak kata yang ditipo. Namun buku ini memiliki gaya
penulisan yang sopan penuh motivasi, dan cover unik.
C. Generasi Langgas merupakan buku yang menjelaskan tentang generasi milenial
yang mau mengambil inspiratif posistif dari berbagai sumber terutama generasi
mileneal yang lain. Meski buku ini terlalu tebal, tetapi buku ini memiliki cover
yang unik dengan bahasa yang mudah dipahami serta santun.
D. Buku setebal 194 halaman ini bercerita tentang generasi milenal sehingga banyak
kata motivasi yang berharga bagi mereka. Selain itu, gaya penulisan yang sopan
dan cover yang unik merupakan daya tarik dari buku ini. Sayangnya buku ini tebal
dan ada kata yang ditipo

Asesmen Standar Pendidikan Daerah Kelas. 9 Paket-1 Bahasa Indonesia 11


(ASPD)
Cermati teks berikut kemudian kerjakan soalnya.

(1) Aku pergi melanglang buana entah ke mana saja. Mencari gunungan-gunungan
uang yang tercecer di kota orang dan mengabaikan bapak yang kesepian
menungguku pulang. Membiarkan pria renta itu mati dalam kesendirian.
(2) “Kamu kapan pulang? Bapak bosan sendirian.”
“Aku sibuk. Tak bisa pulang. Bapak pergi saja ke pinggir jendela. Lidah tetangga
yang ramai pasti bisa menghilangkan kesepian.” Kututup telepon itu sepihak.
Memupuskan semua harapan yang telah lama bapak rawat dalam senyap.
(3) Bapak menahan sesak. Untuk kesekian kalinya, dinding-dinding kaca itu pecah.
Meluruh pada kulit yang tak lagi kencang. Gelap, menghitam karena terkena
sengatan matahari sewaktu muda demi membahagiakan putra tercinta yang
nyatanya meninggalkan juga.
(4) Kini aku merasakan apa yang bapak rasakan. Jiwa durhaka ini hidup menua dalam
kesendirian. Tanpa teman. Ditinggalkan juga diabaikan. Sekarang aku tahu
hukum sebab-akibat selalu berlaku. Yang meninggalkan akan ditinggalkan. Yang
melukai akan dilukai.

21. Bagian klimaks pada kutipan cerita tersebut ditunjukkan nomor ....
A. (1)
B. (2)
C. (3)
D. (4)

Cermati teks berikut kemudian kerjakan soalnya

(1) Aku pergi melanglang buana entah ke mana saja. Mencari gunungan-gunungan
uang yang tercecer di kota orang dan mengabaikan bapak yang kesepian
menungguku pulang. Membiarkan pria renta itu mati dalam kesendirian.
(2) “Kamu kapan pulang? Bapak bosan sendirian.”
“Aku sibuk. Tak bisa pulang. Bapak pergi saja ke pinggir jendela. Lidah tetangga
yang ramai pasti bisa menghilangkan kesepian.” Kututup telepon itu sepihak.
Memupuskan semua harapan yang telah lama bapak rawat dalam senyap.
(3) Bapak menahan sesak. Untuk kesekian kalinya, dinding-dinding kaca itu pecah.
Meluruh pada kulit yang tak lagi kencang. Gelap, menghitam karena terkena
sengatan matahari sewaktu muda demi membahagiakan putra tercinta yang
nyatanya meninggalkan juga.
(4) Kini aku merasakan apa yang bapak rasakan. Jiwa durhaka ini hidup menua dalam
kesendirian. Tanpa teman. Ditinggalkan juga diabaikan. Sekarang aku tahu
hukum sebab-akibat selalu berlaku. Yang meninggalkan akan ditinggalkan. Yang
melukai akan dilukai.

22. Penyebab konflik pada kutipan cerpen tersebut yang paling tepat adalah ...
A. Bapak menghantam dinding kaca hingga pecah menahan sesak.
B. Aku merasakan kesendirian seperti kesendirian bapak dahulu.
C. Ketika bapak menelepon, aku menutup telepon secara sepihak.
D. Aku meninggalkan bapak dalam kesendirian untuk merantau.

Asesmen Standar Pendidikan Daerah Kelas. 9 Paket-1 Bahasa Indonesia 12


(ASPD)
Cermati teks berikut kemudian kerjakan soalnya.

(1) Aku pergi melanglang buana entah ke mana saja. Mencari gunungan-gunungan
uang yang tercecer di kota orang dan mengabaikan bapak yang kesepian
menungguku pulang. Membiarkan pria renta itu mati dalam kesendirian.
(2) “Kamu kapan pulang? Bapak bosan sendirian.”
“Aku sibuk. Tak bisa pulang. Bapak pergi saja ke pinggir jendela. Lidah tetangga
yang ramai pasti bisa menghilangkan kesepian.” Kututup telepon itu sepihak.
Memupuskan semua harapan yang telah lama bapak rawat dalam senyap.
(3) Bapak menahan sesak. Untuk kesekian kalinya, dinding-dinding kaca itu pecah.
Meluruh pada kulit yang tak lagi kencang. Gelap, menghitam karena terkena
sengatan matahari sewaktu muda demi membahagiakan putra tercinta yang
nyatanya meninggalkan juga.
(4) Kini aku merasakan apa yang bapak rasakan. Jiwa durhaka ini hidup menua dalam
kesendirian. Tanpa teman. Ditinggalkan juga diabaikan. Sekarang aku tahu
hukum sebab-akibat selalu berlaku. Yang meninggalkan akan ditinggalkan. Yang
melukai akan dilukai.

23. Akibat konflik pada kutipan cerita tersebut yang paling tepat adalah ...
A. Aku menjadi anak yang durhaka.
B. Aku juga ditinggalkan dalam kesendirian.
C. Aku terluka karena tidak memiliki teman
D. Aku merasakan kesepian yang mendalam.

24. Cermati teks berikut!

Kementerian Pertahanan (Kemenhan) di bawah kepemimpinan Prabowo


Subianto sedang menjajaki kerja sama dengan Kementerian Pendidikan dan
Kebudayaan (Kemendikbud) yang dipimpin Nadiem Makarim. Tujuannya, agar
mahasiswa bisa mengikuti program Bela Negara pada semester awal perkuliahan.
Merespon hal ini, suara netizen di media sosial seperti Twitter terpecah
menjadi dua. Ada yang pro, ada pula yang kontra dengan usulan ini karena dianggap
belum ada urgensi pendidikan militer untuk Indonesia saat ini.
Sementara itu juga ada yang menyambut baik usulan tersebut dan beranggapan
bahwa ini bisa menjadi panduan untuk masyarakat Indonesia lebih sigap menghadapi
berbagai kondisi termasuk bencana dan untuk pertahanan diri.

Simpulan pendapat pro yang paling tepat pada teks tersebut adalah …
A. Pelaksanaan program Bela Negara banyak wujudnya dan memberikan manfaat
untuk menjaga keutuhan NKRI.
B. Mahasiswa perlu dibekali ilmu bela negara untuk membentuk jiwa patriotisme
yang mulai luntur.
C. Program Bela Negara sebaiknya disesuaikan dengan situasi dan kondisi masa
depan bangsa.
D. Program Bela Negara sebaiknya diterapkan kepada mahasiswa yang memiliki
persyaratan.

Asesmen Standar Pendidikan Daerah Kelas. 9 Paket-1 Bahasa Indonesia 13


(ASPD)
25. Cermati kalimat berikut!

Menteri Pendidikan dan Kebudayaan mengatakan bahwa sistem pendidikan di


Indonesia harus menghasilkan sumber daya manusia yang handal, kreatif, dan inovatif.
Kata tidak baku pada kalimat tersebut adalah....
A. menteri
B. sistem
C. handal
D. kreatif

26. Cermati kalimat berikut!

Ngarai Sianok dibentang sepanjang 15 km dengan lebar sekitar 200 meter yang
memisahkan pulau Sumatra menjadi dua bagian memanjang.

Perbaikan kata bentukan pada kalimat tersebut adalah....


A. bentangan
B. terbentang
C. dibentangkan
D. membentang

27. Cermati kalimat berikut!

Ia menyampaikan […] gempa bumi yang mengguncang Kabupaten Majene […]


Mamuju menyebabkan perekonomian di Sulbar sangat terpuruk.

Konjungsi yang tepat untuk melengkapi kalimat tersebut adalah....


A. tentang, serta
B. tentang, dan
C. bahwa, dan
D. bahwa, di

28. Cermati kalimat berikut!

Untuk warga yang tidak terdampak langsung banjir bandang, diminta diungsikan
sementara ke tempat yang lebih aman.

Perbaikan kalimat tersebut adalah …


A. Untuk warga yang tidak terdampak langsung banjir bandang, diminta sementara
mengungsi ke tempat yang lebih aman.
B. Warga yang tidak terdampak langsung banjir bandang diminta mengungsi
sementara ke tempat yang lebih aman.
C. Warga yang tidak terdampak langsung banjir bandang diminta diungsikan
sementara ke tempat yang lebih aman.
D. Untuk warga yang tidak terdampak langsung banjir bandang, sementara diminta
diungsikan ke tempat yang lebih aman.

Asesmen Standar Pendidikan Daerah Kelas. 9 Paket-1 Bahasa Indonesia 14


(ASPD)
29. Cermati teks berikut!

(1) Akhirnya, kancil mengajak tabib sakti menemui paus. (2) Mereka bertiga
berangkat ke istana paus. (3) Di sana mereka disambut oleh pembesar istana. (4) Aku
sendiri juga bingung harus mencari tabib ke mana.(5) Tabib memeriksa penyakit raja,
lalu memberikan ramuan obat untuk diminum. (6) Selang beberapa raja paus langsung
sembuh dari sakitnya.

Perbaikan kalimat (4) agar teks tersebut padu adalah...


A. Tabib sakti langsung diantar menemui raja yang sedang sakit.
B. Tabib sakti mengajak kancil dan rusa mencari raja paus.
C. Rusa mengantarkan kancil mencari rumah tabib itu.
D. Dia adalah tabib sakti yang selama ini mereka cari.

30. Cermati kalimat berikut!

Gejala umum seseorang terinfeksi covid-19 di antaranya sesak nafas, influenza, dan
hilangnya indra penciuman dan perasa.

Perbaikan penulisan kata yang tidak tepat pada kalimat tersebut adalah....
A. influensa, indera
B. terinveksi, napas
C. napas, influensa
D. napas, indera

31. Cermatilah kalimat berikut!

Drs, Ali Rahmadi, MM membeli barang-barang antik, guci cangkir dan lampu

Perbaikan penggunaan tanda baca pada kaJimat tersebut adalah...


A. Drs. Ali Rahmadi, M.M, membeli barang-barang antik, guci, cangkir , dan lampu.
B. Drs. Ali Rahmadi, M.M. membeli barang-barang antik : guci, cangkir dan lampu.
C. Drs. Ali Rahmadi, M.M. membeli barang-barang antik ; guci, cangkir, dan lampu.
D. Drs. Ali Rahmadi, M.M. membeli barang-barang antik : guci, cangkir, dan lampu.

32. Menteri Kelautan dan Perikanan Sakti Wahyu Trenggono mencontohkan kesuksesan
bisnis ikan hias seperti di Bandung Jawa Barat

Tanda baca yang tepat untuk melengkapi kalimat tersebut adalah...


A. Menteri Kelautan dan Perikanan, Sakti Wahyu Trenggono, mencontohkan
kesuksesan bisnis ikan hias : seperti di Bandung, Jawa Barat.
B. Menteri Kelautan dan Perikanan Sakti Wahyu Trenggono, mencontohkan
kesuksesan bisnis ikan hias seperti di Bandung, Jawa Barat.
C. Menteri Kelautan dan Perikanan, Sakti Wahyu Trenggono, mencontohkan
kesuksesan bisnis ikan hias seperti di Bandung, Jawa Barat.
D. Menteri Kelautan dan Perikanan Sakti Wahyu Trenggono, mencontohkan
kesuksesan bisnis ikan hias, seperti di Bandung Jawa Barat.

Asesmen Standar Pendidikan Daerah Kelas. 9 Paket-1 Bahasa Indonesia 15


(ASPD)
33. Cermati kalimat berikut!

Dropping air bersih di dusun lereng Merapi sudah dihentikan, karena pipa air bersih
sudah dapat berfungsi dan pasokan air sudah normal.

Penggunaan tanda baca koma (,) pada kalimat tersebut tidak tepat karena....
A. tanda koma (,) dipakai di antara unsur-unsur dalam suatu pemerincian
B. tanda koma (,) tidak dipakai jika induk kalimat mendahului anak kalimat
C. tanda koma (,) dipakai di belakang kata atau ungkapan penghubung antarkalimat
D. tanda koma (,) dipakai sebelum kata penghubung dalam kalimat majemuk setara

34. Cermati kalimat-kalimat berikut!

(1) Kegiatan bakti sosial ini dilaksanakan hari Kamis, 17 September 2020.
(2) Selain masker, juga dibagikan 4.754 paket sembako kepada warga tidak mampu.
(3) Dalam kegiatan itu dibagikan 11.500 masker kepada para pengguna jalan.
(4) Pemberian sembako merupakan hasil patungan dari sejumlah pengusaha mitra
Ditlantas.
(5) Kegiatan bakti sosial Ditlantas Polda Sulsel berjalan dengan tertib dan lancar.
(6) Dalam rangka Hari Lalu Lintas Bhayangkara ke-65 Ditlantas Polda Sulsel
mengadakan bakti sosial.

Kalimat-kalimat tersebut akan menjadi teks laporan yang padu dengan urutan …
A. (6), (1), (2), (3), (4), (5)
B. (6), (5), (1), (4), (2), (3)
C. (6), (1), (3), (2), (4), (5)
D. (6), (3), (2), (1), (5), (4)

35. Cermati kalimat-kalimat berikut!

(1) Para kru-kru Sriwijaya Air mengantarkan jenazah Kapten Afwan ke Taman Makam
Pahlawan Pondok Rajeg.
(2) Beliau disalatkan di Masjid Ad-Daulah kompleks perumahan BCE pada pukul 11.30
WIB.
(3) Kapten Afwan semasa hidupnya dikenal sebagai pribadi yang sangat santun.
(4) Keluarga besar Sriwijaya Air menyampaikan rasa bela sungkawa dan turut
berduka cita yang mendalam.

Kalimat tidak efektif terdapat pada nomor ....


A. (1) dan (2)
B. (1) dan (4)
C. (2) dan (3)
D. (3) dan (4)

Asesmen Standar Pendidikan Daerah Kelas. 9 Paket-1 Bahasa Indonesia 16


(ASPD)
36. Cermati teks berikut kemudian kerjakan soalnya.

“Maaf, Pak. Saya boleh masuk?” tanya Agus pada bosnya yang sedang memimpin
meeting.
”Iya, silahkan duduk, Gus, tapi maaf hari ini proyekmu digantikan oleh Riyan.”
“Tapi kenapa, Pak? Saya hanya terlambat sebentar.”
“Ini bukan masalah sebentar atau lama. Kita di perusahaan ini para pekerja
profesional. Proyek itu dari dulu saya percayakan sama kamu tapi kamu ternyata tidak
bisa konsisten. Meskipun telat sebentar, ada diantara temanmu yang bisa memberi ide
bagus untuk proyek itu. Jadi maaf sekali lagi, sudah bagus kamu tidak saya keluarkan
dari tim,” jelas bosnya dengan tegas.
Langsung seketika Agus terdiam dengan wajah yang penuh dengan penyesalan.
Setelah meeting selesai Agus pergi menuju meja kerjanya.
“Kamu kenapa hari ini, Gus? Sampai telat seperti ini tak seperti biasannya.”
“Ini salahku, Dev. Aku begadang semalam nonton bola tim kesukaanku sampai larut
malam, sampai-sampai aku lupa kalau ada proyek penting dan seharusnya
menguntungkan bagiku.”
“Hmm makanya kamu harus mengutamakan profesi dari pada hobi,” sambung Devi
sedikit menasihati.

Tokoh utama pada kutipan teks cerita tersebut adalah....


A. Agus
B. Devi
C. Riyan
D. Pak bos

37. Cermati teks berikut kemudian kerjakan soalnya.

“Maaf, Pak. Saya boleh masuk?” tanya Agus pada bosnya yang sedang memimpin
meeting.
”Iya, silahkan duduk, Gus, tapi maaf hari ini proyekmu digantikan oleh Riyan.”
“Tapi kenapa, Pak? Saya hanya terlambat sebentar.”
“Ini bukan masalah sebentar atau lama. Kita di perusahaan ini para pekerja
profesional. Proyek itu dari dulu saya percayakan sama kamu tapi kamu ternyata tidak
bisa konsisten. Meskipun telat sebentar, ada diantara temanmu yang bisa memberi ide
bagus untuk proyek itu. Jadi maaf sekali lagi, sudah bagus kamu tidak saya keluarkan
dari tim,” jelas bosnya dengan tegas.
Langsung seketika Agus terdiam dengan wajah yang penuh dengan penyesalan.
Setelah meeting selesai Agus pergi menuju meja kerjanya.
“Kamu kenapa hari ini, Gus? Sampai telat seperti ini tak seperti biasannya.”
“Ini salahku, Dev. Aku begadang semalam nonton bola tim kesukaanku sampai larut
malam, sampai-sampai aku lupa kalau ada proyek penting dan seharusnya
menguntungkan bagiku.”
“Hmm makanya kamu harus mengutamakan profesi dari pada hobi,” sambung Devi
sedikit menasihati.

Amanat pada kutipan cerita tersebut adalah...


A. Jangan melakukan aktivitas yang tidak penting.
B. Belajarlah bersikap disipin dan tanggung jawab.
C. Hargailah waktu sebaik-baiknya.
D. Dengarkanlah nasihat teman.

Asesmen Standar Pendidikan Daerah Kelas. 9 Paket-1 Bahasa Indonesia 17


(ASPD)
38. Cermati kutipan teks cerpen berikut!

(1) Di bawah langit kemerahan yang akan menjemput bulan, kedatangan mereka
selalu terlihat bagaikan siluet iring -iringan kafilah melintasi gurun perbatasan,
membawa bermacam perbekalan piknik. (2) Berkarung -karung gandum yang
diangkut gerobak pedati, daging asap yang digantungkan di punuk unta terlihat
bergoyang -goyang, roti kering yang disimpan dalam kaleng, botol -botol cuka dan
saus, biskuit dan telor asin, rendang dalam rantang juga berdus -dus mi instan yang
kadang mereka bagikan pada kami. (3) Terdengar bunyi alarm begitu keras mengusik
tidur agus yang begitu terlelap. (4) Dia menggeliat menahan rasa kantuk. Kemudian
dia membuka matanya secara perlahan.

Bukti latar waktu kutipan cerpen tersebut terjadi pada senja hari terdapat pada kalimat
nornor....
A. (1)
B. (2)
C. (3)
D. (4)

39. Cermati kedua teks cerita berikut!

Teks I
“Apa? Ketinggalan? Yang benar aja, kita kan sudah jauh dari warung tempat kita
makan tadi,” jawabku dengan kesal.
“Duh, maaf banget ya Vir, aku benar-benar lupa,” jawabnya dengan berkeringat.
“Apa? minta maaf? kamu pikir dengan minta maaf bisa membuat barangku kembali
dan masalah selesai? Enggak kan? Seenaknya aja kamu minta maaf,” jawabku dengan
kesal, lalu tanpa basa basi aku pergi meninggalkannya

Teks II
Rombongan sirkus itu mungkin sekarang sudah pergi ke Montana atau Wisconsin
atau Toronto atau terus melintasi Teluk Hudson, Laut Labrador, Green Land sampai
Kutub Utara. Namun aku selalu terkenang sirkus itu. Teringat manusia-manusia
terbang yang berkulit sawo matang dan berhidung kecil mirip aku.

Perbedaan pola pengembangan kedua teks tersebut adalah ….


A. menampilkan aksi tokoh, menampilkan lokasi kejadian
B. menampilkan lokasi kejadian, menampilkan aksi tokoh
C. pemunculan masalah, mendeskripsikan aksi tokoh
D. pemunculan masalah , menampilkan latar cerita

Asesmen Standar Pendidikan Daerah Kelas. 9 Paket-1 Bahasa Indonesia 18


(ASPD)
40. Cermati kedua teks cerita berikut!

Teks I
“Apa? Ketinggalan? Yang benar aja, kita kan sudah jauh dari warung tempat kita
makan tadi,” jawabku dengan kesal.
“Duh, maaf banget ya Vir, aku benar-benar lupa,” jawabnya dengan berkeringat.
“Apa? minta maaf? kamu pikir dengan minta maaf bisa membuat barangku kembali
dan masalah selesai? Enggak kan? Seenaknya aja kamu minta maaf,” jawabku dengan
kesal, lalu tanpa basa basi aku pergi meninggalkannya

Teks II
Rombongan sirkus itu mungkin sekarang sudah pergi ke Montana atau Wisconsin
atau Toronto atau terus melintasi Teluk Hudson, Laut Labrador, Green Land sampai
Kutub Utara. Namun aku selalu terkenang sirkus itu. Teringat manusia-manusia
terbang yang berkulit sawo matang dan berhidung kecil mirip aku.

Perbedaan penggunaan bahasa kedua teks tersebut adalah….


A. menggunakan kalimat perbandingan , menggunakan kalimat pertentangan
B. menggunakan bahasa yang berbelit-belit, menggunakan bahasa yang lugas
C. menggunakan kalimat langsung, menggunakan kalimat tidak langsung
D. menggunakan bahasa kiasan, menggunakan bahasa yang lugas

Asesmen Standar Pendidikan Daerah Kelas. 9 Paket-1 Bahasa Indonesia 19


(ASPD)
DINAS PENDIDIKAN KABUPATEN BANTUL
ASESMEN STANDAR PENDIDIKAN DAERAH
MKKS SMP KABUPATEN BANTUL
TAHAP 1
TAHUN PELAJARAN 2020/2021

Mata Pelajaran : Bahasa Indonesia


Kelas : IX (Sembilan) PAKET 2
Hari dan Tanggal : Senin, 15 Februari 2021
Pukul : 07.30 – 09.00 (90 Menit)

PETUNJUK UMUM
1. Isikan identitas Anda dengan benar
2. Tersedia waktu 90 menit untuk mengerjakan paket tes tersebut.
3. Jumlah soal sebanyak 40 butir, pada setiap butir soal terdapat 4 (empat)
pilihan jawaban.
4. Periksa dan bacalah soal-soal sebelum Anda menjawabnya dengan cermat.
5. Periksalah pekerjaan Anda sebelum dikirim / submit
6. Berdoalah sebelum mengerjakan

PETUNJUK KHUSUS
Jawablah dengan menghitamkan bulatan ( ) huruf A, B, C, atau D pada lembar
jawab yang tersedia!

1. Cermati teks berikut!

Hubungan manusia dan lingkungannya berlangsung melalui dua cara. Pertama, manusia
dipengaruhi oleh lingkungan. Kedua, manusia memiliki kemampuan untuk mengubah
lingkungan. Karakteristik hubungan tersebut berbeda antara satu daerah dengan daerah
yang lainnya atau satu masyarakat dan masyarakat lainnya.

Makna kata karakteristik pada teks tersebut adalah....


A. sifat khas
B. keistimewaan
C. watak tertentu
D. kemampuan khusus

Asesmen Standar Pendidikan Daerah Kelas. 9 Paket-2 Bahasa Indonesia 1


(ASPD)
2. Cermati teks berikut!

Guna menjaga kelestarian bangunan bernilai budaya, Pemerintah Kota Yogyakarta


menetapkan lima kawasan sebagai kawasan cagar budaya. Selain itu, Pemkot Yogyakarta
juga menguatkan posisi Kota Yogyakarta sebagai salah satu Kota Pusaka. Lima kawasan
cagar budaya yang ditetapkan adalah Kotabaru, Kotagede, Pakualaman, Malioboro dan
Kraton. Kawasan tersebut memiliki keunikan dalam berbagai sisi, salah satunya gaya
bangunan di kawasan tersebut. Bangunan cagar budaya tidak diperbolehkan dibongkar
dan diubah menjadi bangunan berarsitektur lain. Akan tetapi, bangunan tersebut dapat
dialihfungsikan sehingga memiliki nilai tambah.

Pernyataan yang sesuai dengan isi teks adalah …


A. Yogyakarta menyatakan diri sebagai kota pelestari bangunan bernilai budaya.
B. Pemerintah menetapkan lima kawasan di Yogyakarta sebagai Kota Budaya.
C. Kota Yogyakarta dikuatkan sebagai Kota Pusaka oleh Pemkot Yogyakarta.
D. Bangunan cagar budaya mempunyai keunikan tersendiri dan dapat diubah.

3. Bacalah kutipan cerpen berikut!

(1)Semua mata terarah padanya.(2) Sepatunya yang mengkilap menginjak lantai keramik
kelas kami dengan bunyi yang berirama. (3) Baju yang dikenakan terbuat dari katun
berkualitas disemprot lima jenis parfum. (4) Kami semakin tegang . (5) Tatapannya
ramah ketika dia mengedarkan pandangannya ke seluruh sudut kelas.(6) Sudut bibirnya
sedikit terangkat ketika tatapan matanya menangkap sosok Winda. (7) Dan bisa kulihat
Winda semakin tegang ketika tatapan itu bertahan lama.

Kalimat yang membuktikan latar tempat cerita ada di kelas adalah....


A. (1) dan (3)
B. (2) dan (5)
C. (3) dan (7)
D. (4) dan (6)

4. Bacalah teks cerpen berkut!

(1) Setahun lalu, motor yang belum lunas kreditnya itu harus direlakan untuk ditukar
tambah dengan motor tetangganya. (2) Bukan cuma tukar tambah, jika perlu motor ini
akan aku jual dan kubelikan sepeda. (3) “Aku tak pernah turun gengsi hanya karena
bersepeda ke sekolah,” ucap Kemin kepada istrinya saat itu. (4) “Dengan cara inilah, kita
pasti bisa membiayai kuliah anak kita,” kata Kemin meyakinkan istrinya.

Bukti watak tokoh aku optimis ditandai kalimat nomor ….


A. (1)
B. (2)
C. (3)
D. (4)

Asesmen Standar Pendidikan Daerah Kelas. 9 Paket-2 Bahasa Indonesia 2


(ASPD)
5. Bacalah kutipan fabel berikut!

Si Rusa hanya memandang remeh kemampuan si Kura-Kura. “Kerahkan segenap


kemampuanmu untuk menendang betisku. Ayo, jangan ragu-ragu!”
Si Kura-Kura bersiaga dan mengambil ancang-ancang di tempat tinggi. Ia lalu
menggelindingkan tubuhnya. Ketika hampir tiba di dekat tubuh si Rusa, ia pun
menaikkan tubuhnya hingga tubuhnya melayang. Si Kura-Kura mengincar hidung si
Rusa. Begitu kerasnya tempurung si Kura-Kura mengenai hingga hidung si Rusa putus.
Seketika itu si Rusa yang sombong itu pun mati.

Nilai moral kutipan fabel tersebut adalah....


A. Mempersiapkan apa yang akan dilakukan agar berhasil
B. Tidak ragu-ragu dalam bertindak demi mencapai tujuan
C. Tidak sombong dan meremehkan kemampuan orang lain
D. Mengerahkan segala kemampuan untuk mengalahkan musuh

6. Bacalah teks berikut!

Awalnya kami duduk tenang. Aku, Yonda, Maridi, dan Pardouman satu baris.
Eliza, Bambang, dan Martin di barisan paling depan.
Kami terus bersenda gurau. Seperti biasa Yonda yang menjadi sasaran bully dari
kami, terutama Pardouman.
Tiba-tiba masuk seorang ibu umur 50-an, tetapi pakaiannya cukup nyentrik.
kedatangan ibu membuat Pardouman seperti cacing kepanasan.

Makna simbol dari cacing kepanasan pada teks tersebut adalah....


A. malu
B. takut
C. cemas
D. gelisah

Asesmen Standar Pendidikan Daerah Kelas. 9 Paket-2 Bahasa Indonesia 3


(ASPD)
7. Cermati teks petunjuk berikut!

Cara Membuat Kue Keju

(1) Kocok mentega sampai lembut.


(2) Masukkan kuning telur satu per satu sambil dikocok rata.
(3) [....]
(4) Tambahkan tepung terigu sedikit demi sedikit sambil diaduk sampai rata.
(5) Giling adonan hingga mencapai ketebalan kira-kira 0,5 cm, lalu cetak
(6) Setelah dicetak, letakkan pada loyang yang sudah diolesi mentega.
(7) [....]
(8) Panggang dengan suhu 160 derajat celcius sampai matang dan kering kecoklatan.

Kalimat yang tepat untuk mengisi bagian yang rumpang adalah....


A. (3) Masukkan keju parut, aduk rata.
(7) Oleskan kuning telur di atas adonan
B. (3) Masukkan mentega cair.
(7) Berilah alas kertas mimyak agar tidak lengket.
C. (3) Masukan santan kemudian aduk.
(7) Beri parutan keju agar gurih.
D. (3) Campur keju dengan tepung terigu.
(7) Masukkan kuning telor pada adonan

8. Cermati teks petunjuk berikut!

6 Langkah Cuci Tangan Menurut WHO

(1) Usap dan gosok juga kedua punggung tangan secara bergantian
(2) Bersihkan ujung jari secara bergantian dengan posisi saling mengunci
(3) Gosok dan putar kedua ibu jari secara bergantian
(4) Gosok sela-sela jari tangan hingga bersih
(5) Letakkan ujung jari ketelapak tangan kemudian gosok perlahan.
(6) Tuang sabun cair pada telapak tangan kemudian usap dan gosok kedua telapak tangan
secara lembut dengan arah memutar.

Urutan kalimat-kalimat tersebut agar menjadi petujuk yang sistematis adalah....


A. (6)-(1)-(4)-(2)-(3)-(5)
B. (6)-(2)-(5)-(1)-(4)-(3)
C. (6)-(5)-(1)-(4)-(5)-(3)
D. (6)-(3)-(2)-(5)-(1)-(4)

Asesmen Standar Pendidikan Daerah Kelas. 9 Paket-2 Bahasa Indonesia 4


(ASPD)
9. Perhatikan data berikut!

Data Kunjungan Wisman


pada Bulan Desember 2019
No Negara Presentase
1. Arab Saudi 64,91%
2. India 34, 12%
3. Belanda 49,79%
4. Mesir 80,74%
5. Bahrain 64, 29%

Teks laporan berdasarkan data tersebut adalah....


A. Wisman yang berkunjung ke Indonesia pada bulan Desember 2020 didominasi
wisman dari Mesir sebesar 80,74%, sedangkan wisman terendah dari India yang
hanya 34, 12%
B. Kunjungan wisman ke Indonesia pada bulan Desember 2020 meningkat. Wisman dari
Mesir tercacat paling banyak berkunjung. Arab Saudi dan Bahrain menduduki posisi
kedua.
C. Walaupun Belanda pernah menjajah Indonesia, tetapi warga negaranya paling banyak
yang berkunjung ke Indonesia. Tercatat pada bulan Desember sebesar 49, 79%
D. Negara-negara dari Timur Tengah lebih suka berkunjung ke Indonesia daripada ke
Belanda. Hal ini terlihat pada bulan Desember 2019 kebanyakan wisman dari Mesir
dan Bahrain.

Asesmen Standar Pendidikan Daerah Kelas. 9 Paket-2 Bahasa Indonesia 5


(ASPD)
Bacalah teks berikut untuk menjawab soalnya!

Teks I
Pelangi adalah fenomena optik yang terjadi ketika sinar matahari dengan hujan saling
bereaksi melalui cara tertentu. Pelangi terbentuk karena adanya pembiasan sinar matahari
yang dibelokkan. Sinar ini berpindah arah dari perjalanan satu medium ke medium
lainnya oleh tetesan air yang ada di atmosfer. Ketika membentur hujan, sinar matahari
dibiaskan oleh butiran air di udara. Adanya perbedaan panjang gelombang ketika sinar
matahari dibiaskan meyebabkan warna-warna pada sinar matahari menyebar.

Teks II
Salah satu tempat wisata malam di Jogja yang indah dan menarik untuk
didokumentasikan di sosial media Anda adalah Taman Pelangi Jogja. Taman Pelangi
Jogja ini menyuguhkan gemerlap warna warni lampion. Lokasinya pun sangat mudah
ditemukan dan dijangkau oleh berbagai jenis kendaraan, yaitu di Jalan Ring Road Utara,
Sariharjo, Ngaglik, Sleman, Daerah Istimewa Yogyakarta. Tempatnya masih dalam satu
kawasan dengan Monumen Jogja Kembali. Memang, Taman Pelangi Jogja sangat indah
untuk didokumentasikan.

10. Perbedaan pola penyajian kedua teks tersebut adalah....

Teks I Teks II
A pola peristiwa pola kausaliatas
B khusus- umum urutan waktu
C umum-khusus umum-khusus-umum
D umum khusus khusus-umum

Asesmen Standar Pendidikan Daerah Kelas. 9 Paket-2 Bahasa Indonesia 6


(ASPD)
Bacalah teks berikut untuk menjawab soalnya!

Teks I
Pelangi adalah fenomena optik yang terjadi ketika sinar matahari dengan hujan saling
bereaksi melalui cara tertentu. Pelangi terbentuk karena adanya pembiasan sinar matahari
yang dibelokkan. Sinar ini berpindah arah dari perjalanan satu medium ke medium
lainnya oleh tetesan air yang ada di atmosfer. Ketika membentur hujan, sinar matahari
dibiaskan oleh butiran air di udara. Adanya perbedaan panjang gelombang ketika sinar
matahari dibiaskan meyebabkan warna-warna pada sinar matahari menyebar.

Teks II
Salah satu tempat wisata malam di Jogja yang indah dan menarik untuk
didokumentasikan di sosial media Anda adalah Taman Pelangi Jogja. Taman Pelangi
Jogja ini menyuguhkan gemerlap warna warni lampion. Lokasinya pun sangat mudah
ditemukan dan dijangkau oleh berbagai jenis kendaraan, yaitu di Jalan Ring Road Utara,
Sariharjo, Ngaglik, Sleman, Daerah Istimewa Yogyakarta. Tempatnya masih dalam satu
kawasan dengan Monumen Jogja Kembali. Memang, Taman Pelangi Jogja sangat indah
untuk didokumentasikan.

11. Perbedaan penggunaan bahasa kedua teks tersebut adalah menggunakan konjungsi....

Teks I Teks II
A sebab akibat penambahan
B penambahan sebab akibat
C pertentangan waktu
D waktu pertentangan

12. Bacalah kalimat berikut!

Puluhan pengemudi bus antarkota menjalani tes rapid di Terminal Kali Jaya Cikarang,
Bekasi.

Variasi lain dari kalimat tersebut yang tepat adalah....


A. Beberapa pengemudi bus di Terminal Kali Jaya Cikarang, Bekasi dites rapid.
B. Pengemudi-pengemudi bus antarkota melakukan tes rapid di Terminal Kali Jaya
Cikarang, Bekasi.
C. Bus-bus antarkota di Terminal Kali Jaya Cikarang , Bekasi melakukan rapid untuk
para pengemudinya.
D. Tes rapid diberlakukan kepada puluhan pengemudi bus yang melintas di Terminal
Kali Jaya Cikarang, Bekasi.

Asesmen Standar Pendidikan Daerah Kelas. 9 Paket-2 Bahasa Indonesia 7


(ASPD)
13. Perhatikan kalimat berikut!

Sheeran membuai para pendengarnya dengan lagu-lagu seperti ‘Thinking Out Loud’,
‘Photograph’, ‘Perfect’, ‘Dive’, ‘Happier’, ‘Don’t’, ‘Sing’, ‘Galway Girl’, serta lagu
ciptaannya yang dinyanyikan oleh Justin Bieber, ‘Love Yourself’.

Variasi kata membuai dalam kalimat tersebut adalah ....


A. merayu
B. menyapa
C. memuji
D. melenakan

14. Bacalah kalimat berikut!

Uang koin yang hanya digunakan sebagai alat kerokan belaka

Alasan ketidakefektifan kalimat tersebut adalah....


A. penggunaan kata hanya
B. penggunaan kata hubung yang
C. penggunaan kata sebagai sebelum kata alat
D. kata belaka yang diletakkan di akhir kalimat

15. Cermati paragraf berikut!

(1) Bunga kelelawar hitam termasuk tanaman umbi-umbian. (2) Hal tersebut dikarenakan
bunga kelelawar hitam menyimpan cadangan makanannya di akar. (3) Cadangan
makanan tersebut disebut umbi akar. (4) Kelelawar merupakan hewan pemakan buah-
buahan.

Alasan ketidakpaduan paragraf tersebut adalah ....


A. Kalimat (4) tidak berkaitan dengan kalimat (1), (2), dan (3)
B. Kalimat (3) tidak berkaitan dengan kalimat (1), (2), dan (4)
C. Kalimat (2) tidak berkaitan dengan kalimat (1), (3), dan (4)
D. Kalimat (1) tidak berkaitan dengan kalimat (2), (3), dan (4)

Asesmen Standar Pendidikan Daerah Kelas. 9 Paket-2 Bahasa Indonesia 8


(ASPD)
16. Cermati teks berikut!

Kini, pada bulan Juni, Pemerintah kita dan juga banyak negara mulai
melonggarkan kebijakan karantina atau kita mengenalnya sebagai Pembatasan Sosial
Berskala Besar (PSBB). Masyarakat sudah gerah ingin kembali beraktivitas seperti
dahulu. Pemerintah sudah gerah ingin menggulirkan kembali roda perekonomian. New
Normal begitu gaung yang sering muncul belakangan ini.

Ide pokok paragraf tersebut adalah ….


A. Pemerintah mulai melonggarkan kebijakan karantina
B. Pemerintah segera menggulirkan kembali roda perekonomian
C. kerinduan masyarakat untuk kembali beraktivitas seperti dahulu
D. New Normal begitu gaung yang sering muncul belakangan ini

17. Bacalah teks berikut!

Belajar adalah suatu kegiatan yang harus dilakukan oleh pelajar yang bertujuan
untuk memahami suatu materi. Kegiatan belajar merupakan kegiatan yang harus
dilakukan secara rutin oleh pelajar. Dengan kata lain belajar adalah kegiatan wajib bagi
pelajar. Belajar tidak hanya bisa dilakukan di kelas, tetapi juga bisa dilakukan secara
mandiri maupun kelompok di tempat lain.
Pada kenyataannya, sebagian pelajar merasa kesulitan dalam mengatur waktu
belajar. Mereka tidak mengetahui bagaimana mengatur belajar agar efisien dan efektif.
Belajar menjadi tidak terarah dan membutuhkan waktu yang lebih lama. Akibatnya,
belajar yang mereka lakukan kurang menghasilkan prestasi.

Ringkasan teks tersebut adalah....


A. Sebagian pelajar merasa kesulitan dalam mengatur waktu belajar sehingga kurang
menghasilkan prestasi.
B. Pelajar merasa kesulitan dalam mengatur waktu belajar karena tidak mengetahui cara
mengatur belajar.
C. Belajar adalah suatu kegiatan yang harus dilakukan oleh pelajar dan harus dilakukan
secara rutin oleh pelajar.
D. Belajar suatu kegiatan yang harus dilakukan pelajar, tetapi sebagian pelajar merasa
kesulitan mengatur waktunya.

Asesmen Standar Pendidikan Daerah Kelas. 9 Paket-2 Bahasa Indonesia 9


(ASPD)
18. Bacalah teks berikut!

Hubungan antara olahraga dan kinerja otak telah menjadi perbincangan sejak lama.
Baru-baru ini ditemukan bahwa olahraga yang dilakukan secara rutin membantu
seseorang untuk menyimpan informasi dengan tingkat kecepatan yang lebih tinggi. Ada
penelitian yang menunjukkan bahwa neurotransmitter di otak menjadi lebih terbuka
untuk menerima informasi setelah melakukan olahraga. Aktivitas tersebut juga dikaitkan
dengan kemampuan untuk mendorong pertumbuhan sel-sel baru yang kemungkinan
menambah volume otak manusia.

Simpulan isi paragraf tersebut adalah....


A. Sudah sejak lama ada hubungan antara olahraga dan kinerja otak.
B. Dengan berolahraga rutin, kinerja otak seseorang akan meningkat.
C. Aktivitas olahraga selalu dikaitkan dengan kinerja otak seseorang.
D. Hubungan olahraga dan kinerja otak hanya perbincangan sejak lama.

19. Perhatikan teks berikut!

Sersan Kasim membelai anaknya yang dalam gendongan, ”Saya minta izin untuk
membawanya,” katanya.
”Kau yakin dia tidak menangis?”
”Insya Allah, tidak.”
”Baik kalau begitu. Hati-hati saja.”
”Siap Pak. Terima kasih.”
Ketika giliran peletonnya untuk menyeberang, Kasim menggigil lebih keras lagi. Bukan
hanya karena hujan tambah keras turun. Bukan hanya karena angin pegunungan yang
menembus sela-sela rusuknya. Ia juga menggigil karena Acep mulai resah dalam
gendongannya. Air hujan sudah merembes masuk mengenai kulitnya dan ia menggeliat-
geliat kebasahan dan kedinginan. Sersan Kasim mulai memegang tali yang terentang dari
tepi ke tepi. Air membasahi kakinya, membasahi celananya, membasahi sebagian
bajunya, menjilat-jilat gendongan anaknya.. Air mencapai dada, merendam anaknya. Dan
tiba-tiba Acep menangis....

Komentar yang sesuai dengan isi cerpen tersebut adalah …


A. Karena rasa cinta kepada anaknya, tindakan sersan Kasim tetap nekat membawa
bayinya bergerilya adalah tindakan totalitas seorang ayah.
B. Sersan Kasim sudah memikirkan risiko terbesar ketika membawa bayinya untuk
bergerilya apalagi diizinkan oleh komandan peleton.
C. Komandan peleton mestinya melarang sersan Kasim untuk membawa bayinya
bergerilya karena akan membahayakan keselamatan jiwa.
D. Seharusnya Sersan Kasim tidak ikut menyeberang sungai bersama bayinya karena
risiko bahaya yang dihadapi pasti sangat besar.

Asesmen Standar Pendidikan Daerah Kelas. 9 Paket-2 Bahasa Indonesia 10


(ASPD)
20. Cermati data buku berikut ini!

Judul buku : Hujan Kepagian


Pengarang : Nugroho Notosusanto
Penerbit : Balai Pustaka
Tahun terbit : 2011
Keunggulan : Isi
: Isiceritanya
ceritanyasangat
sangatunik
unikdan
dan menarik.
menarik. Bahasa
Bahasa yang
yang digunakan
sederhan tetapi memikat.
Kelemahan : Menggunakan beberapa kosa kata Belanda sehingga pembaca kurang
memahami arti kata tersebut.

Teks ulasan yang sesuai dengan data buku tersebut adalah....


A. Isi cerita buku Hujan Kepagian sangat unik dan menarik. Sayang, bahasa yang
digunakan sederhana, meskipun memikat. Selain itu buku ini menggunakan kosa kata
Belanda sehingga pembaca kurang memahami arti kata tersebuat. Namun demikian
buku ini layak dibaca karena mengajak generasi muda untuk selalu berbuat baik.
B. Isi cerita buku Hujan Kepagian sangat unik dan menarik. Selain itu, bahasa yang
digunakan sederhana, tetapi memikat. Hanya saja, buku ini menggunakan beberapa
kosakata Belanda sehingga pembaca kurang memahami arti kata tersebut. Namun
demikian buku ini layak dibaca karena mengajak generasi muda untuk selalu berbuat
baik.
C. Isi cerita buku Hujan Kepagian sangat unik dan menarik. Bahasa yang digunakan
sederhana, tetapi memikat karena buku ini menggunakan beberapa kosakata Belanda.
Namun sayang, pembaca kurang memahami arti kata tersebut.
D. Isi cerita buku Hujan Kepagian sangat unik dan menarik. Bahasa yang digunakan
sederhana,tetapi memikat. Keunikan lain buku ini adalah menggunakan kosakata
Belanda sehingga pembaca kurang memahami arti kata tersebut. Jika ingin pintar
buku ini layak dibaca.

Asesmen Standar Pendidikan Daerah Kelas. 9 Paket-2 Bahasa Indonesia 11


(ASPD)
21. Bacalah teks berikut!

(1) Rin, kayaknya kamu betul-betul sakit. Bagaimana kalau aku antarkan kamu ke
rumah sakit hari ini?” Sri, teman Rin satu kos menawarkan bantuan karena melihat
muka pucat pasi yang sedang duduk di ruang makan.
(2) “Ya Tuhan! Rin! Sri, kenapa kamu diam saja. Cepat kita bawa ke rumah sakit. Ibu
Sundari cepat-cepat membantu Rin berdiri dari kursinya dan memapahnya ke mobil
menuju rumah sakit.
(3) Sesampainya di rumah sakit, Rin sudah tak dapat membuka mata, pingsan tak
berdaya. Tubuhnya benar-benar tak dapat bergerak. Kami tunduk dan berdoa demi
keselamatan Rin sambil menitikkan air mata.
(4) Tak pernah ada sedikit pun tanda-tanda bahwa Rin akan mengalami ini. Kejadian
begitu cepat dan tak disangka-sangka. Kami hanya bisa menyerahkan pada
kekuasaan yang mencipta kehidupan dan berharap Rin bisa kembali ceria.

Bagian klimaks kutipan cerita tersebut terdapat pada nomor....


A. (1)
B. (2)
C. (3)
D. (4)

22. Bacalah teks berikut!

Di padang rumput yang indah dan damai hiduplah seekor kelinci yang sangat nakal,
setiap hari kerjaannya hanya mengusili penghuni padang rumput. Pada suatu hari, si
Kelinci bertemu dengan Pak Kijang. Dalam hati, kelinci berpikir “Saya kerjain saja Pak
Kijang, tapi bagaimana ya?” Si Kelinci berpikir sangat keras dan tiba-tiba ada ide nakal
sampai di kepalanya. “Saya pura-pura saja lari di ke arah Pak Kijang sambil berteriak,
“Pak Singa ngamuk! Pak Singa ngamuk!”
Pada hari berikutnya, Pak Kijang bertemu dengan Pak Kerbau, mereka
menceritakan kejadian yang telah mereka alami kemarin. Selagi mereka asyik membahas
masalah yang menimpa keluarga mereka yang sudah pasti disebabkan oleh si nakal
Kelinci, tiba-tiba terdengarlah suara teriakan si Kelinci dari kejauhan. “Tolong, saya
dikejar-kejar Pak Singa, Pak Singa ngamuk! Toloong, tolooong, tolooooong..!!,” tapi
tidak ada yang peduli. “Ah, paling-paling Si Kelinci lagi-lagi membohongin kita” pikir
mereka sambil tertawa melihat si kelinci.

Tokoh utama cerita tersebut adalah...


A. Pak Kijang
B. Pak Singa
C. Si Kelinci
D. Pak Kerbau

Asesmen Standar Pendidikan Daerah Kelas. 9 Paket-2 Bahasa Indonesia 12


(ASPD)
23. Bacalah teks berikut!

Di padang rumput yang indah dan damai hiduplah seekor kelinci yang sangat nakal,
setiap hari kerjaannya hanya mengusili penghuni padang rumput. Pada suatu hari, si
Kelinci bertemu dengan Pak Kijang. Dalam hati, kelinci berpikir “Saya kerjain saja Pak
Kijang, tapi bagaimana ya?” Si Kelinci berpikir sangat keras dan tiba-tiba ada ide nakal
sampai di kepalanya. “Saya pura-pura saja lari di ke arah Pak Kijang sambil berteriak,
“Pak Singa ngamuk! Pak Singa ngamuk!”
Pada hari berikutnya, Pak Kijang bertemu dengan Pak Kerbau, mereka
menceritakan kejadian yang telah mereka alami kemarin. Selagi mereka asyik membahas
masalah yang menimpa keluarga mereka yang sudah pasti disebabkan oleh si nakal
Kelinci, tiba-tiba terdengarlah suara teriakan si Kelinci dari kejauhan. “Tolong, saya
dikejar-kejar Pak Singa, Pak Singa ngamuk! Toloong, tolooong, tolooooong..!!,” tapi
tidak ada yang peduli. “Ah, paling-paling Si Kelinci lagi-lagi membohongin kita” pikir
mereka sambil tertawa melihat si kelinci.

Sebab terjadinya konflik pada teks tersebut adalah…


A. Kelinci tidak mau mendengar nasihat teman-temannya.
B. Kelinci selalu mengganggu teman-temannya.
C. Pak Singa memberi hukman kepada kancil yang nakal.
D. Pak Singa tidak terima dituduh mencelakai kelinci.

Asesmen Standar Pendidikan Daerah Kelas. 9 Paket-2 Bahasa Indonesia 13


(ASPD)
24. Bacalah teks berikut!

Di padang rumput yang indah dan damai hiduplah seekor kelinci yang sangat nakal,
setiap hari kerjaannya hanya mengusili penghuni padang rumput. Pada suatu hari, si
Kelinci bertemu dengan Pak Kijang. Dalam hati, kelinci berpikir “Saya kerjain saja Pak
Kijang, tapi bagaimana ya?” Si Kelinci berpikir sangat keras dan tiba-tiba ada ide nakal
sampai di kepalanya. “Saya pura-pura saja lari di ke arah Pak Kijang sambil berteriak,
“Pak Singa ngamuk! Pak Singa ngamuk!”
Pada hari berikutnya, Pak Kijang bertemu dengan Pak Kerbau, mereka
menceritakan kejadian yang telah mereka alami kemarin. Selagi mereka asyik membahas
masalah yang menimpa keluarga mereka yang sudah pasti disebabkan oleh si nakal
Kelinci, tiba-tiba terdengarlah suara teriakan si Kelinci dari kejauhan. “Tolong, saya
dikejar-kejar Pak Singa, Pak Singa ngamuk! Toloong, tolooong, tolooooong..!!,” tapi
tidak ada yang peduli. “Ah, paling-paling Si Kelinci lagi-lagi membohongin kita” pikir
mereka sambil tertawa melihat si kelinci.

Akibat konflik pada teks tersebut adalah…


A. Pak Kijang dikucilkan oleh hewan yang lainnya.
B. Pak Kerbau merasa ditipu oleh keluarga Pak Kijang.
C. Si Kelinci tidak percaya lagi oleh hewan yang lain.
D. Kelinci memaafkan perbuatan Pak Kijang.

25. Bacalah teks berikut!

(1) Permainan game online bukanlah hal yang asing lagi bagi anak-anak sekarang ini. (2)
Padahal, permainan ini dapat memberikan dampak kurang baik bagi anak. (3) Pendapat
tersebut bisa dipahami karena banyak game online yang mengusung tema kekerasan
yang sebenarnya tidak disarankan untuk dilihat secara langsung oleh anak-anak. (4)
Namun, alasa-alasan yang disajikan tentu tidak boleh berdasarkan asumsi karena banyak
juga sisi positif yang diperoleh anak jika bermain game online tidak berlebihan.

Simpulan pendapat kontra pada teks tersebut ditandai dengan nomor....


A. (1)
B. (2)
C. (3)
D. (4)

Asesmen Standar Pendidikan Daerah Kelas. 9 Paket-2 Bahasa Indonesia 14


(ASPD)
26. Bacalah teks berikut!

Buku memiliki peran vital dalam distribusi pengetahuan dari satu orang ke orang lain.
Buku mampu mendokumentasikan berbagai pemikiran seseorang dalam bentuk tertulis.
Ativitas-aktivitas yang berhubungan dengan buku memacu orang untuk meningkatkan
kwalitas hidupnya.

Kata tidak baku pada kalimat tersebut adalah....


A. vital
B. mendokumentasikan
C. aktivitas-aktivitas
D. kwalitas

27. Cermati kalimat berikut!

Rendahnya daya beli buku masyarakat Indonesia bukan karena ketidakmampuan


ekonomi, tetapi tidak menjadikan membeli buku sebagai aktivitas belanja.

Perbaikan kata aktivitas pada kalimat tersebut yang tepat adalah....


A. prioritas
B. kualitas
C. efektivitas
D. sensitivitas

28. Bacalah kalimat berikut!

Setiap hari Minggu pemancingan itu ramai pengunjung [...] akses masuk ke tempat itu
macet.

Kata penghubung yang tepat untuk melengkapi kalimat tersebut adalah....


A. karena
B. tetapi
C. padahal
D. sehingga

29. Perhatikan kalimat berikut!

Peserta didik yang berminat mengikuti lomba menulis puisi agar secepatnya segera
mendaftar ke Panitia Lomba Literasi Sekolah.

Perbaikan kalimat tersebut supaya menjadi kalimat efektif adalah menghilangkan kata....
A. didik
B. berminat
C. secepatnya
D. mendaftar

Asesmen Standar Pendidikan Daerah Kelas. 9 Paket-2 Bahasa Indonesia 15


(ASPD)
30. Bacalah teks berikut!

(1) Jika Anda tidak dapat mengatur pemakaian listrik dengan baik, bisa-bisa tagihan
listrik Anda jadi membengkak dan membebani anggaran bulanan. (2) Untuk mencegah
hal tersebut terjadi, maka Anda perlu menetapkan peraturan hemat listridi rumah. (3)
Pilih saja bisnis yang mudah dan tidak membutuhkan modal besar. (4) Misalnya ,
penggunaan lampu hemat energi, kulkas yang hemat listrik, dan jangan lupa cabut kabel
yang tidak digunakan.

Perbaikan teks tersebut supaya padu adalah mengganti kalimat nomor (3) menjadi....
A. Orang tua harus mengingatkan tentang bahaya listrik.
B. Anda harus terus mencoba dan berusaha bekerja keras
C. Caranya bisa dengan meminimalisasi penggunaan listrik
D. Bila bisnis Anda lancar akan membantu keuangan keluarga

31. Perhatikan kalimat berikut!

Pada umumnya kopi disuguhkan pada tradisi kunjungan tamu, sarapan, atau bersantai
sore hari.

Perbaikan penggunaan kata yang tepat adalah....


No Kata yang salah Perbaikan
A. disuguhkan diberikan
B. tradisi acara
C. sarapan makan pagi
D. pertemuan pertemanan

32. Perhatikan kalimat berikut!

H. Sarwoko, S.H. dosen fakultas hukum di salah satu universitas di Malang; tetapi beliau
tinggal di Surabaya.

Perbaikan tanda baca pada kalimat tersebut yang tepat adalah....


A. Koma (,) setelah nama diganti tanda hubung(-)
B. Titik (.) pada H. Sarwoko dihilangkan
C. Titik (.) pada kata S.H. seharusnya S.H
D. Titik koma (;) diganti koma (,)

Asesmen Standar Pendidikan Daerah Kelas. 9 Paket-2 Bahasa Indonesia 16


(ASPD)
33. Cermati kalimat berikut!

Kita harus bersyukur Min rumah kita bertingkat jadi tidak perlu mengungsikan kata
Mama

Penggunaan tanda baca yang tepat pada kalimat tersebut adalah....


A. “ Kita harus bersyukur, Min, Rumah kita bertingkat, jadi tidak perlu mengungsi,
kan?” kata Mama.
B. “Kita harus bersyukur, Min, rumah kita bertingkat jadi tidak perlu mengungsi, kan!”
kata Mama.
C. “Kita harus bersyukur, Min, rumah kita bertingkat jadi tidak perlu mengungsi kata
Mama.”
D. “Kita harus bersyukur, Min, rumah kita bertingkat, jadi tidak perlu mengungsikan kata
Mama!”

34. Bacalah kalimat berikut!

“Saya lihat dulu ya, tunggu sebentar?” ujar orang tua itu.

Alasan ketidaktepatan penggunaan tanda baca pada kalimat tersebut adalah....


A. tanda tanya (?) tidak tepat karena bukan kalimat tanya
B. tanda koma (,) setelah kata ya tidak diperlukan.
C. tanda tanya (?) seharusnya diganti tanda (,)
D. tanda tanya (?) tidak tepat digunakan seharusnya titik (.)

35. Cermati kalimat-kalimat berikut!


(1) Kegiatan tahun ini dilaksanakan di Blitar pada tanggal 7 Januari 2020.
(2) Mereka tampak sangat ceria dan antusias mengikuti kegiatan.
(3) Kegiatan ini diikuti oleh 120 orang siswa dan 10 orang pendamping.
(4) Sebelum berangkat semua berkumpul di lapangan sekolah pukul 07.00
(5) SMP Nusantara melaksanakan kegiatan studi tour.
(6) Setelah mengecek kehadiran peserta, bus siap berangkat menuju Blitar.

Susunan kalimat laporan yang tepat adalah ….


A. (5) - (4) - (1) - (3) - (2) - (6)
B. (5) - (4) - (3) - (2) - (1) - (6)
C. (5) - (3) - (1) - (4) - (2) - (6)
D. (5) - (3) - (1) - (2) - (4) - (6)

Asesmen Standar Pendidikan Daerah Kelas. 9 Paket-2 Bahasa Indonesia 17


(ASPD)
36. Cermati kalimat-kalimat berikut!
(1) Peningkatan minat baca dilakukan dengan kegiatan literasi.
(2) Pendidikan karakter akan membentuk siswa berkarakter lebih baik.
(3) Peran keluarga sangat penting dalam menanamkan kebiasaan yang baik.
(4) Pantai Bali sangat terkenal sekali dengan keindahannya.

Kalimat yang tidak efektif ditandai dengan nomor...


A. (1)
B. (2)
C. (3)
D. (4)

37. Bacalah teks berikut!

Setiap pukul dua pagi, berbekal sebatang bambu, kami sempoyangan memikul berbagai
jenis makhluk laut yang harus sudah tersaji di meja pualam stanplat pada pukul lima,
sehingga pukul enam sudah bisa diserbu ibu-ibu. Artinya, setelah itu kami leluasa untuk
sekolah. (Sang Pemimpi Andrea Hirata)

Amanat yang terkandung dalam cerita tersebut adalah ...


A. Perbanyaklah makan ikan agar menjadi anak yang pandai!
B. Bekerjalah terlebih dahulu sebelum berangkat sekolah!
C. Bekerjalah yang rajin agar mudah mendapat rezeki!
D. Bekerjalah sejak pagi agar meraih keberhasilan!

38. (1) Suara ayam berkokok terdengar begitu nyaring menembus lubang-lubang bilik
kamarku. (2) Suara tumbukan alu yang beradu dengan lesung membangunkan aku dari
lelapku.(3) Segera aku tersadar bahwa hari ini adalah hari Senin. (4) Aku bergegas
mengambil handuk dan melesat menuju sumur yang letaknya tak jauh dari rumahku. (5)
Aku melewati ibu yang sedang sibuk menumbuk padi untuk persediaan makan tiga hari
ke depan.

Kalimat yang membuktikan latar waktu peristiwa terjadi pada pagi hari ditandai dengan
nomor....
A. (1) dan (2)
B. (3) dan (4)
C. (4) dan (5)
D. (1) dan (3)

Asesmen Standar Pendidikan Daerah Kelas. 9 Paket-2 Bahasa Indonesia 18


(ASPD)
Bacalah teks berikut untuk menjawab soalnya!

Teks 1
Matahari baru mengirimkan pasukan jingganya ke angkasa. Embun memeluk ujung daun
kuat-kuat. Kicau sisikatan menyelimuti Sari.
“Sari..., Sari... bangun, Nak! seru ibu dari luar kamar.
“Ibu... kepala Sari pusing,” rengek Sari dari dalam kamar, lalu membukakan pintu
kamarnya.
“Aduh badanmu panas. Sudah jangan sekolah dulu, ya?”kata ibu panik.
“ Tapi, hari ini ada ulangan, Bu,” lirih Sari.
“Udah, nanti ibu memberi tahu Bu Lim, “ kata ibu.

Teks 2
Tangan-tangannya berotot. Padahal Om Bos sudah terbilang tidak muda lagi. Usianya
sudah di atas 50 tahun, tetapi pakaiannya masih seperti anak muda.
“ Mau ke mana, Om?”tanyaku. Ketika Om Bos melintas di depan rumahkau. Wangi
semerbak parfumnya.
“Biasa, Di. Bisnis kecil-kecilan,” katanya singkat.
Aku tidak tahu apa sesungguhnya pekerjaan Om Bos. Kadang-kadang aku lihat setiap
hari dia pergi. Adakalanya seminggu tidak pergi-pergi.

39. Perbedaan pola pengembangan kedua teks tersebut adalah....

Teks 1 Teks 2
A. diawali dengan gambaran latar diawali dengan gambaran tokoh
B. diawali dengan pemunculan masalah diawali dengan perkenalan
C. diawali dengan konflik diawali dengan pemunculan masalah
D. diawali reaksi tokoh diawali dengan aksi tokoh

Asesmen Standar Pendidikan Daerah Kelas. 9 Paket-2 Bahasa Indonesia 19


(ASPD)
Bacalah teks berikut untuk menjawab soalnya!

Teks 1
Matahari baru mengirimkan pasukan jingganya ke angkasa. Embun memeluk ujung daun
kuat-kuat. Kicau Sisikatan menyelimuti Sari.
“Sari..., Sari... bangun, Nak! seru ibu dari luar kamar.
“Ibu... kepala Sari pusing,” rengek Sari dari dalam kamar, lalu membukakan pintu
kamarnya.
“Aduh badanmu panas. Sudah jangan sekolah dulu, ya?”kata ibu panik.
“ Tapi, hari ini ada ulangan, Bu,” lirih Sari.
“Udah, nanti ibu memberi tahu Bu Lim, “ kata ibu.

Teks 2
Tangan-tangannya berotot. Padahal om Bos sudah terbilang tidak muda lagi. Usianya
sudah di atas 50 tahun, tetapi pakaiannya masih seperti anak muda.
“ Mau ke mana ,Om?”tanyaku. Ketika Om Bos melintas di depan rumahkau. Wangi
semerbak parfumnya.
“Biasa, Di. Bisnis kecil-kecilan,” katanya singkat.
Aku tidak tahu apa sesungguhnya pekerjaan Om Bos. Kadang-kadang aku lihat setiap
hari dia pergi. Adakalanya seminggu tidak pergi-pergi.

40. Perbedaan penggunaan bahasa pada kedua teks tersebut adalah....

Teks 1 Teks 2
A. menyelipkan kata – kata daerah menyelipkan kata-kata dari bahasa asing
B. menggunakan kata-kata baku menggunakan kata-kata tidak baku
C. menggunakan kalimat-kalimat bermajas menggunakan kalimat-kalimat lugas
D. menggunakan kalimat-kalimat sebab- menggunakan kalimat-kalimat
akibat pertentangan

Asesmen Standar Pendidikan Daerah Kelas. 9 Paket-2 Bahasa Indonesia 20


(ASPD)
DINAS PENDIDIKAN KABUPATEN BANTUL
ASESMEN STANDAR PENDIDIKAN DAERAH
MKKS SMP KABUPATEN BANTUL
TAHAP 2
TAHUN PELAJARAN 2020/2021

Mata Pelajaran : Bahasa Indonesia


Kelas : IX (Sembilan) PAKET 2
Hari dan Tanggal : Senin, 8 Maret 2021
Pukul : 07.30 – 09.00 (90 Menit)

PETUNJUK UMUM
1. Isikan identitas Anda dengan benar
2. Tersedia waktu 90 menit untuk mengerjakan paket tes tersebut.
3. Jumlah soal sebanyak 40 butir, pada setiap butir soal terdapat 4 (empat)
pilihan jawaban.
4. Periksa dan bacalah soal-soal sebelum Anda menjawabnya dengan cermat.
5. Periksalah pekerjaan Anda sebelum dikirim / submit
6. Berdoalah sebelum mengerjakan

PETUNJUK KHUSUS
Jawablah dengan menghitamkan bulatan ( ) huruf A, B, C, atau D pada lembar jawab
yang tersedia!

1. Cermati teks berikut!

Danau Sentani adalah danau yang terletak di Papua. Danau Sentani berada di
bawah lereng Pegunungan Cagar Alam Cyclops yang memiliki luas sekitar
245.000 hektar. Danau ini terbentang antara Kota Jayapura dan Kabupaten
Jayapura, Papua. Danau Sentani memiliki luas sekitar 9.360 hektar dan berada
pada ketinggian 75 mdpl. Danau Sentani merupakan danau terluas di Papua.
Makna kata Cagar Alam pada teks tersebut adalah ....
A. daerah yang kelestarian tumbuhan dan binatang di dalamnya dilindungi oleh
undang-undang
B. daerah yang kelestarian masyarakat dan peri kehidupannya dilindungi oleh
undang-undang.
C. daerah untuk tempat mengungsi atau menumpang hidup binatang langka dan
dilindungi
D. suatu daerah yang khusus melindungi binatang liar yang termasuk ekosistem
unik.

Asesmen Standar Pendidikan Daerah Tahap-2 Paket-2 Bahasa Indonesia 1


(ASPD)
2. Cermati teks berikut!

Terletak di bagian Selatan Pulau Lombok, Kawasan Ekonomi Khusus (KEK)


Mandalika ditetapkan melalui Peraturan Pemerintah Nomor 52 Tahun 2014 untuk
menjadi KEK Pariwisata. Dengan luas area sebesar 1.035,67 Ha dan menghadap
Samudera Hindia, KEK Mandalika diharapkan dapat mengakselerasi sektor
pariwisata Provinsi Nusa Tenggara Barat yang sangat potensial. KEK Mandalika
menawarkan wisata bahari dengan pesona pantai dan bawah laut yang
memukau. Mandalika berasal dari nama seorang tokoh legenda, yaitu Putri
Mandalika yang dikenal dengan parasnya yang cantik. KEK Mandalika memiliki
konsep pengembangan pariwisata berwawasan lingkungan dengan
pembangunan obyek-obyek wisata dan daya tarik wisata yang selalu berorientasi
kepada kelestarian nilai dan kualitas lingkungan hidup yang ada di masyarakat.

Pernyataan yang sesuai dengan isi teks tersebut adalah ...


A. Mandalika dipilih sebagai KEK karena berasal dari nama tokoh legenda yang
sangat cantik jelita.
B. KEK berkewajiban menarik kunjungan wisatawan mancanegara sebanyak
dua juta per tahun.
C. Sebagai KEK, Mandalika menawarkan wisata bahari dan budaya dengan
tetap berwawasan lingkungan.
D. Mandalika disebut kawasan ekonomi khusus karena mengembangkan wisata
bahari sebagai ciri khas Indonesia.

3. Cermati teks berikut kemudian kerjakan soalnya !

(1) “Ini buruk sekali!” pengunjung di sebelah kami berseru—komentar yang sama
seperti Ali barusan. (2) Ada empat penduduk perkampungan ternak di sana, usia
mereka separuh baya. (3) Aku menoleh. (4) Apakah mereka sedang
mengomentari makanan di depan mereka? (5) Tidak, mereka justru memakannya
dengan lahap. (6) Mereka sedang riuh membicarakan hal lain. (7) “Separuh
domba milikku tewas enam bulan terakhir, Peeg. Ini buruk sekali.” (8) “Sapi
perahku juga demikian,” temannya menambahkan. (9) “Separuh dari unitku
menyisakan sekat-sekat kosong. (10) Produksi susu setahun terakhir turun drastis.

Bukti latar suasana riuh pada kutipan cerita tersebut adalah ....
A. (1) dan (6)
B. (2) dan (7)
C. (3) dan (8)
D. (4) dan (9)

Asesmen Standar Pendidikan Daerah Tahap-2 Paket-2 Bahasa Indonesia 2


(ASPD)
Cermati teks berikut kemudian kerjakan soalnya !

(1) Majalah Bold sudah tidak ada. Tidak bertahan di era digital masih aja ngotot
nge-print majalah. Ini sih romantismenya Ms. Drew, yang mengatakan, “Kertas itu
memanusiakan manusia, Be. Membuka lembar demi lembar, lebih personal.
Belum lagi gambarnya seperti bicara langsung pada kita.”
(2) “Be, ini waktu yang tepat. Dengan merebaknya virus Corona dan kita harus
work from home, saya rasa sekalian saja merumahkan kalian for good.”
(3) “Maksud Ms. Drew?”
(4) “Ya demikian maksud saya. Majalah ini sebentar lagi almarhum, anggap kena
dampak virus…hahaha,” ujar Ms. Drew tertawa miris di suatu sore di kantornya.
(5) “Tidak lucu lho Ms. Drew. Ini urusannya dengan nasib, nyawa seluruh
karyawan Bold.”
(6) “Nah!” Ms. Drew menggerakkan kursi kerjanya yang terbuat dari kulit hitam
360 derajat. Lalu menunjukkan jari ke arahku. “Itu maksudku, jangan sampai
mereka jadi korban!”

4. Bukti tokoh Be berwatak humanis terdapat pada kalimat nomor ....


A. 3
B. 4
C. 5
D. 6

Cermati teks berikut kemudian kerjakan soalnya !

(1) Majalah Bold sudah tidak ada. Tidak bertahan di era digital masih aja ngotot
nge-print majalah. Ini sih romantismenya Ms. Drew, yang mengatakan, “Kertas itu
memanusiakan manusia, Be. Membuka lembar demi lembar, lebih personal.
Belum lagi gambarnya seperti bicara langsung pada kita.”
(2) “Be, ini waktu yang tepat. Dengan merebaknya virus Corona dan kita harus
work from home, saya rasa sekalian saja merumahkan kalian for good.”
(3) “Maksud Ms. Drew?”
(4) “Ya demikian maksud saya. Majalah ini sebentar lagi almarhum, anggap kena
dampak virus…hahaha,” ujar Ms. Drew tertawa miris di suatu sore di kantornya.
(5) “Tidak lucu lho Ms. Drew. Ini urusannya dengan nasib, nyawa seluruh
karyawan Bold.”
(6) “Nah!” Ms. Drew menggerakkan kursi kerjanya yang terbuat dari kulit hitam
360 derajat. Lalu menunjukkan jari ke arahku. “Itu maksudku, jangan sampai
mereka jadi korban!”

5. Nilai moral kutipan cerita tersebut adalah ....


A. Dunia digital di era pandemi
B. Tantangan seorang pemimpin di era pandemi
C. Virus corona menghancurkan perusahaan percetakan
D. Pemimpin sebaiknya memikirkan nasib anak buahnya

Asesmen Standar Pendidikan Daerah Tahap-2 Paket-2 Bahasa Indonesia 3


(ASPD)
Cermati teks berikut kemudian kerjakan soalnya !

(1) Majalah Bold sudah tidak ada. Tidak bertahan di era digital masih aja ngotot
nge-print majalah. Ini sih romantismenya Ms. Drew, yang mengatakan, “Kertas itu
memanusiakan manusia, Be. Membuka lembar demi lembar, lebih personal.
Belum lagi gambarnya seperti bicara langsung pada kita.”
(2) “Be, ini waktu yang tepat. Dengan merebaknya virus Corona dan kita harus
work from home, saya rasa sekalian saja merumahkan kalian for good.”
(3) “Maksud Ms. Drew?”
(4) “Ya demikian maksud saya. Majalah ini sebentar lagi almarhum, anggap kena
dampak virus…hahaha,” ujar Ms. Drew tertawa miris di suatu sore di kantornya.
(5) “Tidak lucu lho Ms. Drew. Ini urusannya dengan nasib, nyawa seluruh
karyawan Bold.”
(6) “Nah!” Ms. Drew menggerakkan kursi kerjanya yang terbuat dari kulit hitam
360 derajat. Lalu menunjukkan jari ke arahku. “Itu maksudku, jangan sampai
mereka jadi korban!”

6. Makna simbol tokoh Miss Drew menunjukkan jari kepada tokoh Be sambil berkata,
“Nahh” yang tepat pada cerita tersebut adalah ....
A. Be bersalah dan ditunjukkan kesalahannya oleh Miss Drew
B. Terjadi persetujuan bahwa Majalah Bold telah gulung tikar
C. Miss Drew sepakat dengan Be untuk menyelamatkan majalah Bold
D. Miss Drew menganggap Be menjadi ganjalan perusahaan majalah Bold

7. Cermati teks petunjuk berikut!


Cara membuat konten yang viral
1. Tentukan judul yang menarik!
2. Buatlah konten singkat dan jelas agar tidak membuat penonton bosan!
3. Gunakan gambar yang menarik perhatian dan tidak bertele-tele!
4. Aturlah agar konten yang kalian buat menjadi enak dipandang!
5. Setinglah agar konten kalian selalu dapat diakses!
6. Selalu berikan referensi yang bisa dipercaya.
7. Buatlah daftar video yang bisa diakses.
8. Lakukan pengaturan waktu yang tepat untuk mengunggah konten.
9. Buatlah konten yang interaktif yang melibatkan penonton.
10. [...]

Kalimat yang tepat untuk melengkapi petunjuk tersebut adalah...


A. Bagikan konten melalui berbagai media.
B. Unggahlah konten di satu media agar lebih valid.
C. Bagikan kepada teman atau pengikut setiamu saja.
D. Penonton diwajibkan membagikan kepada penonton lain.

Asesmen Standar Pendidikan Daerah Tahap-2 Paket-2 Bahasa Indonesia 4


(ASPD)
8. Cermati petunjuk berikut!

Cara Membuat Pengharum Ruangan Potpourri


(1) Setelah siap, buka stoples.
(2) Tambahkan beberapa tetes minyak esensial.
(3) Tempatkan tanaman-tanaman yang sudah mengering ke dalam stoples.
(4) Tutup wadah dan biarkan bunga untuk menyerap minyak selama beberapa
minggu.
Urutan petunjuk yang tepat adalah....
A. (3), (4), (2), (1)
B. (3), (2), (1), (4)
C. (4), (3), (1), (2)
D. (4), (1), (2), (3)

9. Cermati data laporan berikut!

Objek Pengamatan : Perpustakaan SMP Sejati


Tujuan : Memaparkan kondisi perpustakaan
Waktu pengamatan : 40 menit saat pembelajaran
Data:
- 32 siswa
- 1000 koleksi
- terletak di dekat taman sekolah
- 2 petugas pustakawan yang ramah
- 10 meja baca dan 40 kursi warna warni
- jam pelajaran bahasa Indonesia

Laporan yang tepat sesuai dengan data tersebut adalah ....

A. Perpustakaan SMP Sejati sanggup menampung 40 siswa untuk


melaksanakan pembelajaran. Perpustakaan yang menyediakan 1000 koleksi
ini dapat memberikan sumber belajar yang banyak.
B. Sepuluh meja warna-warni memberikan kebahagiaan bagi siswa SMP Sejati
yang berkunjung ke perpustakaan sekolah. Aura ceria yang dipancarkan
membuat kegiatan belajar mengajar menjadi lancar dan penuh semangat.
C. Perpustakaan SMP Sejati memberikan fasilitas untuk pelaksanaan KBM.
Dengan daya tampung sebanyak 40 siswa, dapat mengakomodasi
pembelajaran kelas dengan jumlah siswa 32. Dua orang pustakawan siap
sedia melayani siswa dengan 1000 koleksi.
D. Perpustakaan SMP Sejati yang terletak di dekat taman sekolah membuat
kegiatan belajar mengajar menjadi penuh semangat karena aura sejuk dari
taman yang ada. Dengan meja kursi warna-warni yang tertata rapi dan
keramahan pustakawan yang melayani, kegiatan belajar menjadi lebih
menyenangkan.

Asesmen Standar Pendidikan Daerah Tahap-2 Paket-2 Bahasa Indonesia 5


(ASPD)
Cermati kedua teks berikut, kemudian kerjakan soalnya!

Teks 1
Kami menuju Harau melalui Payakumbuh. Dari Payakumbuh pukul 06.00 sampai
Harau pukul 07.00. Turun dari angkot kami naik becak motor dengan tarif Rp
40.000 hingga pukul 07.30 karena kami meminta bapak sopir becak motor untuk
pelan-pelan agar kami dapat menikmati permandangan. Kami menuju Lembah
Harau yang memiliki tujuh air terjun (Sarasah Bunta). Sampai di sana kami
memuaskan diri menikmati pemandangan hingga sore menjelang.

Teks 2
Anda dapat memulai perjalanan dari Solok menuju lembah Harau melalui
Batusangka. Dari Batusangka Anda akan menuju Lintau dengan perjalanan
sepanjang 30 km. Dari Lintau Anda harus menempuh perjalanan 35 km menuju
Payakumbuh. Barulah Anda akan merasa lega karena tinggal 10 km lagi menuju
Lembah Harau. Jauhnya perjalanan yang Anda tempuh pastinya terbayar dengan
indahnya pemandangan Lembah Harau.

10. Perbedaan pola penyajian kedua teks tersebut adalah ....


A. teks 1 urutan waktu – teks 2 urutan tempat
B. teks 1 urutan tempat – teks 2 urutan waktu
C. teks 1 urutan peristiwa – teks 2 urutan sebab akibat
D. teks 1 urutan sebab akibat – teks 2 urutan peristiwa

Cermati kedua teks berikut, kemudian kerjakan soalnya!

Teks 1
Kami menuju Harau melalui Payakumbuh. Dari Payakumbuh pukul 06.00 sampai
Harau pukul 07.00. Turun dari angkot kami naik becak motor dengan tarif Rp
40.000 hingga pukul 07.30 karena kami meminta bapak sopir becak motor untuk
pelan-pelan agar kami dapat menikmati permandangan. Kami menuju Lembah
Harau yang memiliki tujuh air terjun (Sarasah Bunta). Sampai di sana kami
memuaskan diri menikmati pemandangan hingga sore menjelang.

Teks 2
Anda dapat memulai perjalanan dari Solok menuju lembah Harau melalui
Batusangka. Dari Batusangka Anda akan menuju Lintau dengan perjalanan
sepanjang 30 km. Dari Lintau Anda harus menempuh perjalanan 35 km menuju
Payakumbuh. Barulah Anda akan merasa lega karena tinggal 10 km lagi menuju
Lembah Harau. Jauhnya perjalanan yang Anda tempuh pastinya terbayar dengan
indahnya pemandangan Lembah Harau.

11. Perbedan penggunaan bahasa kedua teks tersebut adalah ….


A. Teks 1 menggunakan kata tunjuk – teks 2 tidak menggunakan kata tunjuk
B. Teks 1 menggunakan kata sandang – teks 2 tidak menggunakan kata
sandang
C. Teks 1 menggunakan kalimat aktif dan pasif – teks 2 menggunakan kalimat
majemuk
D. Teks 1 menggunakan kata ganti orang pertama-teks 2 menggunakan kata
ganti orang kedua

Asesmen Standar Pendidikan Daerah Tahap-2 Paket-2 Bahasa Indonesia 6


(ASPD)
12. Cermati kalimat berikut!
Salah satu destinasi wisata terkenal di Magelang adalah Dusun Butuh
Kaliangkrik atau terkenal dengan sebutan Nepal van Java.

Variasi yang dapat menggantikan kata destinasi pada kalimat tersebut adalah ....
A. tujuan
B. objek
C. daerah
D. pilihan

13. Cermati kalimat berikut!

Penemuan vaksin menjadi titik terang bagi pandemi di seluruh dunia.

Variasi kalimat tersebut yang tepat adalah ...


A. Pandemi di seluruh dunia menunggu-nunggu penemuan vaksin.
B. Penemuan vaksin diharuskan memberikan harapan baru kepada dunia.
C. Pandemi di seluruh dunia mendapat titik terang dengan ditemukannya vaksin.
D. Vaksin diharapkan menjadi penemuan yang memberi harapan bagi pandemi.

14. Cermati kalimat berikut!


Pada masa pandemi seperti sekarang ini kepedulian kita kepada sesama diuji
dengan kondisi masyarakat yang serba sangat kekurangan.

Kalimat tersebut tidak efektif karena ....


A. penggunaan kata yang berlebih
B. ketidaklengkapan struktur kalimat
C. penggunaan kata tugas yang tidak tepat
D. mengandung kalimat yang bermakna ganda

15. Cermati paragraf berikut!

Pandemi memaksa para siswa untuk belajar melalui media daring. Berbagai
aplikasi pembelajaran dapat digunakan untuk membantu para siswa belajar
sesuai dengan situasi kondisi yang memungkinkan. Pemilihan aplikasi yang tepat
dan terjangkau dapat membuat siswa bersemangat. Dari berbagai aplikasi yang
ada, mobile legend sangat digemari oleh para siswa.

Paragraf tersebut tidak padu karena...


A. Kalimat (1) membahas tentang pandemi, sementara kalimat lainnya
membahas tentang aplikasi pembelajaran.
B. Kalimat (2) membahas tentang aplikasi pembelajaran sementara kalimat
lainnya membahas tentang kesenian Jepang.
C. Kalimat (3) membahas tentang pemilihan aplikasi sementara kalimat lainnya
membahas tentang pandemi
D. Kalimat (4) membahas tentang aplikasi game online, sementara kalimat
lainnya membahas tentang aplikasi pembelajaran.

Asesmen Standar Pendidikan Daerah Tahap-2 Paket-2 Bahasa Indonesia 7


(ASPD)
Cermati teks berikut, kemudian kerjakan soalnya!

Literasi digital berperan mengefektifkan interaksi dan komunikasi selama


proses pembelajaran. Sebagai contoh, kemampuan dalam menggunakan fitur
kamera dan mikrofon pada perangkatnya agar mampu hadir dan terhubung
secara virtual. Lebih jauh, kemampuan menggunakan perangkat lunak untuk
menyajikan teks dan gambar pendukungnya (grafik, ilustrasi, dan sebagainya)
berperan untuk mengoptimalkan kolaborasi dan komunikasi dalam pembelajaran
daring yang dijembatani oleh fitur email, online wordsheet dan spreadsheet, serta
fitur ‘lampirkan file’ yang ada pada berbagai perangkat lunak.
Kompetensi literasi digital berperan dalam kemampuan mengakses
berbagai sumber pembelajaran daring yang berkualitas. Selama masa pandemi,
siswa memiliki keterbatasan dalam mengakses sumber informasi yang ada di
sekolah, sehingga sumber informasi yang mungkin diakses adalah yang berbasis
online. Sumber informasi online yang kaya informasi, menuntut siswa untuk
mampu mengakses informasi yang berkualitas, sebagai suplemen informasi
untuk pembelajaran daring yang diikuti.

16. Ide pokok paragraf kedua pada teks tersebut adalah ....
A. Literasi digital pada masa pandemi
B. Akses informasi berkualitas pada masa pandemi
C. Keefektifan interaksi dan komunikasi pada pembelajaran
D. Peran kompetensi literasi digital pada pembelajaran daring

Cermati teks berikut, kemudian kerjakan soalnya!

Literasi digital berperan mengefektifkan interaksi dan komunikasi selama


proses pembelajaran. Sebagai contoh, kemampuan dalam menggunakan fitur
kamera dan mikrofon pada perangkatnya agar mampu hadir dan terhubung
secara virtual. Lebih jauh, kemampuan menggunakan perangkat lunak untuk
menyajikan teks dan gambar pendukungnya (grafik, ilustrasi, dan sebagainya)
berperan untuk mengoptimalkan kolaborasi dan komunikasi dalam pembelajaran
daring yang dijembatani oleh fitur email, online wordsheet dan spreadsheet, serta
fitur ‘lampirkan file’ yang ada pada berbagai perangkat lunak.
Kompetensi literasi digital berperan dalam kemampuan mengakses
berbagai sumber pembelajaran daring yang berkualitas. Selama masa pandemi,
siswa memiliki keterbatasan dalam mengakses sumber informasi yang ada di
sekolah, sehingga sumber informasi yang mungkin diakses adalah yang berbasis
online. Sumber informasi online yang kaya informasi, menuntut siswa untuk
mampu mengakses informasi yang berkualitas, sebagai suplemen informasi
untuk pembelajaran daring yang diikuti.

17. Ringkasan yang tepat sesuai teks tersebut adalah ...


A. Literasi digital mengefektifkan proses pembelajaran di masa pandemi.
Kemampuan mengakses semua sumber informasi sebagai sumber belajar
menjadi peran dari kompetensi literasi digital.
B. Literasi digital berperan mengefektifkan transaksi selama proses
pembelajaran. Kemampuan mengakses sumber informasi berkualitas sebagai
sumber belajar menjadi peran dari kompetensi literasi digital.

Asesmen Standar Pendidikan Daerah Tahap-2 Paket-2 Bahasa Indonesia 8


(ASPD)
C. Literasi digital berperan mengaktifkan interaksi dan komunikasi proses
pembelajaran. Kemampuan mencari sumber informasi berkualitas sebagai
sumber belajar menjadi peran dari kompetensi literasi digital.
D. Literasi digital berperan mengefektifkan interaksi dan komunikasi selama
proses pembelajaran. Peran kompetensi literasi digital dalam mengakses
berbagai sumber pembelajaran daring berkualitas.

18. Cermati teks berikut!

Teh telang atau teh biru sudah digunakan selama berabad-abad sebagai
penambah daya ingat, nootropik, antistres, anxiolytic, antidepresan, antikonvulsan
dan obat penenang. Teh telang juga bisa membantu menurunkan berat badan
dengan mencegah dan melawan penyakit hati berlemak. Karena teh telang
mengandung antioksidan, teh telang bisa meningkatkan kesehatan mata. Teh
telang telah terbukti efektif melawan depresi, stres dan kecemasan. Ekstrak teh
telang bisa membantu mengurangi kadar glukosa dalam tubuh dan meningkatkan
berat badan. menurunkan kadar kolesterol tinggi yang berisiko menyebabkan
penyakit jantung.

Simpulan isi teks tersebut adalah ...


A. Teh telang bermanfaat mencegah berbagai macam penyakit.
B. Teh telang khusus mengatasi gangguan kejiwaan atau depresi.
C. Teh telang berwarna biru dan berubah menjadi ungu apabila diberi air lemon.
D. Teh telang sebagai alternatif pengobatan alami khusus untuk penderita
kolesterol.

19. Cermati teks berikut!

Ringan, sederhana, dan bermakna adalah tiga kata yang dapat menggambarkan
film Keluarga Cemara. Kurang lebih selama satu jam 50 menit penonton disuguhi
cerita menghibur yang masih menyajikan karakter yang sudah dikenal: Abah,
Emak, Euis, dan Ara. Ceritanya pun masih seperti Keluarga Cemara yang muncul
sejak 1970-an, tentang suatu keluarga kecil yang harus bertahan hidup setelah
tulang punggung keluarga mereka jatuh miskin. Mereka sampai harus pindah
rumah dan anak-anak ikut membantu bekerja. Diawali dengan adegan-adegan
ringan, perlahan kisah Keluarga Cemara mulai menggugah emosi. Beberapa
adegan membuat penonton ikut terenyuh, misalnya ketika Abah dan Emak
berbicara bahwa mereka benar-benar bangkrut. Akting Ringgo dan Nirina Zubir
patut diacungi jempol.

Komentar yang tepat berdasarkan isi teks tersebut adalah teks tersebut ...
A. Tokoh Abah dan Emak yang pernah mengalami bangkrut menjadi
pembelajaran bagi penonton.
B. Penggambaran film Keluarga Cemara yang ringan, sederhana, dan bermakna
dapat menarik penonton.
C. Ulasan tentang tokoh Nirina Zubir sangat bagus hingga penonton tidak perlu
lagi menyimak film secara langsung.
D. Film Keluarga Cemara sangat menghibur sehingga bisa disebut sebagai film
komedi yang bisa membuat penonton terpingkal-pingkal.

Asesmen Standar Pendidikan Daerah Tahap-2 Paket-2 Bahasa Indonesia 9


(ASPD)
20. Cermati data buku berikut!

Judul buku : Sokola Rimba


Pengarang : Butet Manurung
Penerbit : Kompas
Tahun terbit beserta cetakan : Terbit 2013, Cetakan Ketiga, November 2015
Tebal buku : xxviii + 384 hlm
ISBN : 978-979-709-996-1
Keunggulan : bahasa yang digunakan sangat familiar dan mudah dipahami
oleh pembaca.
Kelemahan : penempatan catatan-catatan makna bahasa asing berada di
akhir buku

Teks ulasan yang tepat sesuai dengan data buku tersebut adalah ....

A. Buku karya Butet Manurung tahun 2015 ini menggunakan bahasa yang
sangat familiar dan mudah dipahami pembaca. Namun, buku terbitan Kompas
setebal 384 halaman ini memiliki kelemahan pada penempatan catatan di
akhir buku. Hal itu menyulitkan pembaca untuk mencari makna kata asing
yang ada dalam buku.
B. Buku yang ditulis oleh Butet Manurung pada tahun 2015 ini memiliki
keunggulan yaitu mudahnya memahami bahasa dalam buku dan tidak
memiliki kekurangan sama sekali. Hal itu membuat pembaca sangat
menyukai buku ini karena sangat mudah dibaca dan dipahami. Beberapa
pembaca mengaku mengulang karena suka.
C. Buku yang ditulis oleh Kompas pada tahun 2015 ini memiliki keunggulan
dalam bidang bahasa yang digunakan sangat familiar dan mudah dipahami
oleh pembaca. Namun, buku terbitan Kompas setebal 384 halaman ini
memiliki kelemahan pada penempatan catatan yang berada di akhir buku.
D. Buku karya Butet Manurung pada tahun 2015 ini memiliki keunggulan yang
sulirt ditemukan oleh pembaca. Namun, buku terbitan Kompas setebal 384
halaman ini memiliki kelemahan pada penempatan catatan yang berada di
akhir buku. Hal itu menyulitkan pembaca.

Asesmen Standar Pendidikan Daerah Tahap-2 Paket-2 Bahasa Indonesia 10


(ASPD)
Cermati teks berikut kemudian kerjakan soal !

(1) Petualangan Mata berawal dari ajakan sang mama untuk berlibur ke daerah
Belu. Di daerah terpencil nan hijau itu, kesialan demi kesialan malah
menimpa Mata dan Mama sejak hari pertama.
(2) Dalam usahanya menepis kesialan dengan bimbingan penduduk setempat,
Mata terpisah dari mamanya dan tertawan oleh suku Melus. Suku Melus ini
digambarkan sebagai suku terasing yang tidak tahu Indonesia itu apa, tapi
memiliki kepandaian bahasa dengan bermodalkan sekali dengar. Mereka
mempertahankan diri dan pusakanya dari kepunahan hingga bersembunyi
dari para pemburunya.
(3) Dengan ditemani sahabat baru dari suku Melus bernama Atok, Mata
berusaha kabur dari tanah Melus dan mencari mama. Perjuangan melarikan
diri ini pun penuh halang rintang. Mata dan Atok harus beradu dengan ratu
kupu-kupu dan dewa buaya.
(4) Mata mendapat pertolongan peri. Akhirnya Mata berhasil. Perjuangan Mata
patut diacungi jempol.

21. Bagian klimaks pada kutipan cerita tersebut ditunjukkan nomor ....
A. (1)
B. (2)
C. (3)
D. (4)

Cermati teks berikut kemudian kerjakan soal !

(1) Petualangan Mata berawal dari ajakan sang mama untuk berlibur ke daerah
Belu. Di daerah terpencil nan hijau itu, kesialan demi kesialan malah
menimpa Mata dan Mama sejak hari pertama.
(2) Dalam usahanya menepis kesialan dengan bimbingan penduduk setempat,
Mata terpisah dari mamanya dan tertawan oleh suku Melus. Suku Melus ini
digambarkan sebagai suku terasing yang tidak tahu Indonesia itu apa, tapi
memiliki kepandaian bahasa dengan bermodalkan sekali dengar. Mereka
mempertahankan diri dan pusakanya dari kepunahan hingga bersembunyi
dari para pemburunya.
(3) Dengan ditemani sahabat baru dari suku Melus bernama Atok, Mata
berusaha kabur dari tanah Melus dan mencari mama. Perjuangan melarikan
diri ini pun penuh halang rintang. Mata dan Atok harus beradu dengan ratu
kupu-kupu dan dewa buaya.
(4) Mata mendapat pertolongan peri. Akhirnya Mata berhasil. Perjuangan Mata
patut diacungi jempol.

22. Penyebab konflik pada kutipan cerpen tersebut yang paling tepat adalah ....
A. Ajakan Mama untuk berlibur ke daerah Belu
B. Jahilnya Mata yang mempimpikan liburan di pulau
C. Atok yang menjadi sahabat namun memendam benci.
D. Ratu kupu-kupu dan dewa buaya yang diganggu oleh Mata

Asesmen Standar Pendidikan Daerah Tahap-2 Paket-2 Bahasa Indonesia 11


(ASPD)
Cermati teks berikut kemudian kerjakan soal !
(1) Petualangan Mata berawal dari ajakan sang mama untuk berlibur ke daerah
Belu. Di daerah terpencil nan hijau itu, kesialan demi kesialan malah
menimpa Mata dan Mama sejak hari pertama.
(2) Dalam usahanya menepis kesialan dengan bimbingan penduduk setempat,
Mata terpisah dari mamanya dan tertawan oleh suku Melus. Suku Melus ini
digambarkan sebagai suku terasing yang tidak tahu Indonesia itu apa, tapi
memiliki kepandaian bahasa dengan bermodalkan sekali dengar. Mereka
mempertahankan diri dan pusakanya dari kepunahan hingga bersembunyi
dari para pemburunya.
(3) Dengan ditemani sahabat baru dari suku Melus bernama Atok, Mata
berusaha kabur dari tanah Melus dan mencari mama. Perjuangan melarikan
diri ini pun penuh halang rintang. Mata dan Atok harus beradu dengan ratu
kupu-kupu dan dewa buaya.
(4) Mata mendapat pertolongan peri. Akhirnya Mata berhasil. Perjuangan Mata
patut diacungi jempol.

23. Akibat konflik pada kutipan cerita tersebut yang paling tepat adalah ...
A. Mata mendapatkan pertolongan dari peri
B. Mata mendapatkan sahabat baru bernama Atok
C. Mama bangga pada Mata anak semata wayangnya
D. Mata ditolong oleh Peri dalam menghalau segala gangguan

24. Cermati teks cerita berikut!


Banyak kalangan yang tidak menyetujui eSport menjadi cabang olahraga.
Menariknya, eSport telah masuk dalam ajang Asian Games 2018. Electronic
sport (eSport) sebagai salah satu cabang olahraga yang dipertandingkan.
Ada perbedaan yang cukup besar antara seorang gamer dengan atlet eSports.
Selain mereka kerap melatih fisik dan stamina agar kuat untuk bertanding
layaknya atlet olahraga lain, mereka juga butuh memiliki kemampuan intelektual
dan logika yang juga baik untuk mengatur strategi dalam bermain game untuk
bertanding.
Dalam dunia olahraga eSport bisa masuk dalam kategori olahraga yang
melibatkan motorik halus, layaknya permainan catur dan bridge. Sementara
olahraga pada umumnya merupakan aktivitas olahraga yang banyak melibatkan
motorik kasar.

Tujuan penulisan teks eSport yaitu:


(1) Menunjukkan bahwa eSport masuk dalam kategori olahraga motorik halus.
(2) Menjelaskan bahwa eSport telah menjadi cabang olahraga dalam ajang Asian
Games 2018.
(3) Membandingkan eSport dengan cabang olahraga yang lain untuk
menunjukkan bahwa eSport layak disebut sebagai cabang olahraga.
(4) Meyakinkan bahwa banyak kalangan yang tidak menyetujui eSport menjadi
cabang olahraga.

Tujuan penulisan teks tersebut adalah ...


A. 1 dan 2
B. 3 dan 4
C. 1. 2, dan 3
D. 1, 2, dan 4
Asesmen Standar Pendidikan Daerah Tahap-2 Paket-2 Bahasa Indonesia 12
(ASPD)
25. Cermati kalimat berikut!

Sudah saatnya si Genius berpenampilan rapih, energik, dan jauh dari kumal.

Kata tidak baku pada kalimat tersebut adalah....


A. genius
B. rapih
C. energik
D. kumal

26. Cermati kalimat berikut!


Sebagai pengkerajin anyaman mendong, rasa bangga akan menyelimuti saat
hasil karya dimanfaatkan sebagai benda pakai atau benda seni.

Perbaikan kata bentukan pada kalimat tersebut adalah....


A. terajin
B. perajin
C. pengrajin
D. kerajinan

27. Cermati kalimat berikut!

Sebagai warga negara yang baik ia menyadari [...] pajak harus dibayarkan [...]
akan dimanfaatkan oleh pemerintah untuk membiayai pembangunan.

Konjungsi yang tepat untuk melengkapi kalimat tersebut adalah ....


A. bahwa, dan
B. karena, dan
C. bahwa, tetapi
D. sehingga, dengan

28. Cermati kalimat berikut!

Sudah semua siswa sekolah mendapatkan kuota internet dari Kemendikbud.

Perbaikan kalimat tersebut adalah ...


A. Semua siswa sudah mendapatkan kuota internet dari Kemendikbud.
B. Semua siswa hampir mendapatkan kuota internet dari Kemendikbud.
C. Semua siswa sekolah sudah mendapatkan kuota internet dari Kemendikbud.
D. Hampir semua siswa sekolah mendapatkan kuota internet dari Kemendikbud.

29. Cermati teks berikut!

(1) Daun sambung nyawa, atau yang dikenal juga sebagai daun dewa, dipercaya
memiliki banyak manfaat untuk kesehatan. (2) Beberapa penelitian melaporkan
bahwa ekstrak daun sambung nyawa mengandung berbagai zat aktif kimiawi
yang baik untuk kesehatan, seperti saponin, tannin, terpenoid, flavonoid,
kaempferol-3-O-rutinoside, kaempferol, astragalin, dan rutin. (3) Daun sambung
nyawa banyak tumbuh di negara-negara Asia Tenggara. (4) Selain itu, daun ini
juga diyakini dapat digunakan untuk mengatasi penyakit jantung dan diabetes.
Asesmen Standar Pendidikan Daerah Tahap-2 Paket-2 Bahasa Indonesia 13
(ASPD)
Perbaikan kalimat (3) agar teks tersebut padu adalah ...
A. Banyak yang kebingungan membedakanantara daun sambung nyawa
dengan daun dewa.
B. Daun sambung nyawa sangat diburu untuk obat herbal di se-antero
Yogyakarta dan Jawa Tengah.
C. Daun sambung nyawa belum lama dikenal olah khalayak ramai walaupun
sejak dulu telah tumbuh di desa-desa.
D. Antioksidan dalam daun sambung nyawa dapat melawan radikal bebas yang
dapat merusak sel-sel dan menyebabkan kanker.

30. Cermati kalimat berikut!

Sekertaris itu dituntut untuk segera meyerahkan notulen rapat komisaris


pemegang saham.

Perbaikan penulisan kata yang tidak tepat pada kalimat tersebut adalah....
A. notulen, rapat
B. komisaris, saham
C. sekretaris, notula
D. sekertaris, komisaris

31. Cermati kalimat berikut!

Dr Sutomo lahir di desa Ngepeh Jawa Timur dengan nama asli Subroto.

Perbaikan penggunaan tanda baca pada kaJimat tersebut adalah ....


A. Dr.Sutomo lahir di Desa Ngepeh, Jawa Timur dengan nama asli Subroto.
B. Dr.Sutomo lahir di desa Ngepeh, Jawa Timur, dengan nama asli Subroto.
C. dr.Sutomo lahir di Desa Ngepeh Jawa Timur dengan nama asli Subroto.
D. dr.Sutomo lahir di desa Ngepeh, Jawa Timur, dengan nama asli Subroto.

32. Cermati kalimat berikut!

"Kami sudah memberikan rekomendasi daerah gunung. Sudah kita tentukan


radius 5 kilometer dari puncak gunung Merapi sebagai jarak aman. Masyarakat
supaya berada di jarak aman. Untuk di Semeru 1 kilometer dari puncak, itu yang
harus diwaspadai, kata Kasbani, Kepala PVMBG.

Tanda baca yang tepat untuk melengkapi kalimat tersebut adalah ...
A. "Kami sudah memberikan rekomendasi daerah gunung. Sudah kita tentukan
radius 5 kilometer dari puncak gunung Merapi sebagai jarak aman.
Masyarakat supaya berada di jarak aman. Untuk di Semeru 1 kilometer dari
puncak, itu yang harus diwaspadai," kata Kasbani, Kepala PVMBG.
B. "Kami sudah memberikan rekomendasi daerah gunung.” “Sudah kita tentukan
radius 5 kilometer dari puncak gunung Merapi sebagai jarak aman.”
“ Masyarakat supaya berada di jarak aman.” “ Untuk di Semeru 1 kilometer
dari puncak, itu yang harus diwaspadai," kata Kasbani, Kepala PVMBG.
C. "Kami sudah memberikan rekomendasi daerah gunung. Sudah kita tentukan
radius 5 kilometer dari puncak gunung Merapi sebagai jarak aman.
Masyarakat supaya berada di jarak aman. Untuk di Semeru 1 kilometer dari
puncak, itu yang harus diwaspadai! kata Kasbani, Kepala PVMBG.”

Asesmen Standar Pendidikan Daerah Tahap-2 Paket-2 Bahasa Indonesia 14


(ASPD)
D. "Kami sudah memberikan rekomendasi daerah gunung. Sudah kita tentukan
radius 5 kilometer dari puncak gunung Merapi sebagai jarak aman.
Masyarakat supaya berada di jarak aman. Untuk di Semeru 1 kilometer dari
puncak, itu yang harus diwaspadai?" kata Kasbani, Kepala PVMBG.

33. Cermati kalimat berikut!


Saat inflasi ringan terjadi, harga beras naik dari Rp.10.000,00 menjadi
Rp.10.900,00.
Penggunaan tanda baca pada kalimat tersebut tidak tepat karena ....
A. tanda koma (,) setelah kata terjadi seharusnya tidak perlu
B. tanda koma (,) antara satuan rupiah dengan sen diganti dengan titik
C. tanda titik (.) seharusnya tidak digunakan pada singkatan satuan uang
D. tanda titik (.) tidak digunakan pada akhir kalimat setelah nilai mata uang

34. Cermati kalimat-kalimat berikut!


“Tolonglah kami Tuan,” pinta wanita tua itu di pinggir jalan.
“Aduuuuuh!” keluh sang Pangeran.
“Mengapa masih ada peminta-minta di kota ini Patih?” Sang Pangeran
menyalahkan patih.
Patih menunduk.
[ …. ]

Kalimat yang dapat digunakan untuk melanjutkan kutipan cerita tersebut


adalah ...
A. “Patih, tolong berikan emas permata pada perempuan itu!” perintah Pangeran
diikuti mata berbinar sang Patih.
B. “Bereskan pajak rakyat, jangan sampai perempuan tua itu tidak membayar
pajak juga!” kata Pangeran.
C. “Tolong urus kemiskinan di kota ini Patih, jangan sampai para penduduk
meminta-minta seperti ini, memalukan, huh!”
D. “Patih, tolong kemiskinan seperti ini diberdayakan agar kita mendapatkan
bantuan dari negara lain.” kata Pangeran.

35. Cermati kalimat-kalimat berikut!


(1)Saat membahas mengenai tentang tanaman hias yang sedang melonjak, kita
akan merasa bersemangat.
(2)Kenaikan harga yang fantastis dan sangat tidak masuk logis justru menjadi
daya tarik tanaman hias tertentu.
(3)Entah harga tersebut benar-benar harga jual atau harga penawarannya saja,
hanya pedagang yang bisa menjelaskan.
(4)Peluang bisnis menjanjikan sangat terbuka dengan lonjakan harga tanaman
hias tertentu.
(5)Namun, para pebisnis pemula sering terjerumus oleh spekulasi harga yang
terus melambung.
Kalimat tidak efektif terdapat pada nomor ....
A. (I) dan (2)
B. (2) dan (3)
C. (3) dan (4)
D. (4) dan (5)
Asesmen Standar Pendidikan Daerah Tahap-2 Paket-2 Bahasa Indonesia 15
(ASPD)
36. Cermati teks berikut kemudian kerjakan soal !
Ketika tahu rata-rata masa pengabdian kelima pegawai toko Roti Tan ini, kepada
Pak Joko aku lontarkan satu pertanyaan yang mengganjal, “Bapak nggak bosan
kerja di satu tempat yang sama selama itu?”
Pak Joko malah menatapku bingung, seakan tak mengerti kenapa hal itu
dipertanyakan.
“Ndak, ndak bosan. Namanya pekerjaan, yah, jalankan saja. Lagian kami juga
sudah seperti keluarga di sini.” Ia tertawa ringan.
“Sudah banyak suka duka selama kerja di sini. Terus terang Ibu sedih sekali
Madre dilepas,” kata Bu Sum kepadaku, “tapi kami ingin melepas Madre dengan
bahagia. Moga-moga Madre juga senang di tempat barunya, ya.”

Tokoh utama pada kutipan teks cerita tersebut adalah ....


A. Pak Joko
B. Bu Sum
C. Madre
D. Aku

37. Cermati paragraf berikut!

“Kemarin saya melihat kemunculan ampuk-ampuk. Tepat di daerah Kali Purba,


dekat dari tempat yang mau dituju Mas Lambang. Kalau sudah muncul ampuk-
ampuk berarti bakal ada bahaya.”

Amanat pada kutipan cerita tersebut adalah ...


A. Belajarlah pada fenomena alam
B. Alam yang selalu memberi tanda
C. Tanda-tanda alam hanyalah mitos
D. Fenomena alam yang membahayakan

38. Cermati kutipan cerpen berikut!


(1) Bulan sabit bertengger sehabis adzan berlawanan mulai pudarnya lembayung
di barat. (2) Satu persatu gemintang saut-menyaut dendangkan dzikir. (3)
Kerlipnya memberi isyarat. (4) Lengkungan langit teduh. Meski gelap, kerlipnya
masih seperti kemarin. Indah. Senandung suara menjerambab ulu hati. Bintang-
bintang menari dengan ritus dan ritual ilahiah. Bintang sendiri tidak ragu
berpetualang di negeri dzikir. Tarikan demi tarikan nafasnya dinamis. Dengan
kodrat alamiyahnya.
Bukti latar waktu terjadi pada petang hari terdapat pada kalimat nornor ....
A. (1)
B. (2)
C. (3)
D. (4)

Asesmen Standar Pendidikan Daerah Tahap-2 Paket-2 Bahasa Indonesia 16


(ASPD)
Cermati paragraf berikut!

Teks 1
Aku mengurangi kecepatan mobil, menginjak kopling, mengalihkan gigi tiga ke
gigi dua, lalu mengarahkan setir ke pekarangan rumah. Mesin mobil terdengar
tersendat dan sedikit meraung. Aku tahu, pasti persenelingnya bermasalah lagi.
Aku selalu kesulitan setiap akan memindahkan gigi.

Teks 2
Gus Sabri, siapa yang tidak kenal beliau? Setiap kali nama itu disebut, seperti ada
kekaguman yang tak kan habis diceritakan mengenai daya linuwih putra bungsu
almarhum Kiai Ridwan Sholeh, pendiri pondok pesantren Pringkuning Kesamben.
Di antara kelima saudaranya, Gus Sabri yang paling santer dibicarakan.

39. Perbedaan pola pengembangan kedua teks tersebut adalah ….


A. teks 1 pemunculan masalah, teks 2 menampilkan aksi tokoh
B. teks 1 mendeskripsikan aksi tokoh, teks 2 pemunculan masalah
C. teks 1 menampilkan lokasi kejadian, teks 2 pemunculan masalah
D. teks 1 mendeskripsikan aksi tokoh, teks 2 mendeskripsikan tokoh

Cermati paragraf berikut!

Teks 1
Aku mengurangi kecepatan mobil, menginjak kopling, mengalihkan gigi tiga ke
gigi dua, lalu mengarahkan setir ke pekarangan rumah. Mesin mobil terdengar
tersendat dan sedikit meraung. Aku tahu, pasti persenelingnya bermasalah lagi.
Aku selalu kesulitan setiap akan memindahkan gigi.

Teks 2
Gus Sabri, siapa yang tidak kenal beliau? Setiap kali nama itu disebut, seperti ada
kekaguman yang tak kan habis diceritakan mengenai daya linuwih putra bungsu
almarhum Kiai Ridwan Sholeh, pendiri pondok pesantren Pringkuning Kesamben.
Di antara kelima saudaranya, Gus Sabri yang paling santer dibicarakan.

40. Perbedaan penggunaan bahasa kedua teks tersebut adalah .…


A. teks 1 menggunakan makna kias, teks 2 menggunakan bahasa lugas
B. teks 1 menggunakan dialog antartokoh, teks 2 menggunakan ungkapan
C. teks 1 menggunakan ungkapan, teks 2 menggunakan majas sinekdoke pars
pro toto
D. teks 1 menggunakan kata ganti orang pertama, teks 2 menggunakan kata
ganti orang ketiga

Asesmen Standar Pendidikan Daerah Tahap-2 Paket-2 Bahasa Indonesia 17


(ASPD)
DOKUMEN TPM ASPD PAKET. A
SANGAT RAHASIA

TES PENDALAMAN MATERI


ASESMEN STANDAR PENDIDIKAN DAERAH
TAHAP 1
SMP/MTS KOTA YOGYAKARTA
TAHUN PELAJARAN 2020/2021

Mata Pelajaran : BAHASA INDONESIA


Hari/Tanggal : SENIN/22 FEBRUARI 2021
Waktu : 07.30 – 09.30

PEMERINTAH KOTA YOGYAKARTA


DINAS PENDIDIKAN PEMUDA DAN OLAHRAGA
2021

www.giriwidodo.com
1. Bacalah kutipan teks berikut!
Museum Monumen Jogja Kembali merupakan salah satu destinasi wisata sejarah yang
banyak dikunjungi di Daerah Istimewa Yogyakarta. Museum ini terletak di Jalan Lingkar
Utara, Jongkang, Sariharjo, Ngaglik, Sleman. Masyarakat sekitar sering menyebutnya
Monjali, yaitu singkatan dari Monumen Jogja Kembali. Koleksi museum ini berupa
benda-benda replika, foto, dokumen, dan lain-lain. Replika dapur umum tahun
1945-1949 menjadi salah satu daya tarik pengunjung museum ini.

Makna kata replika pada kutipan teks tersebut adalah ….


A. bangunan
B. buatan
C. lukisan
D. tiruan

2. Bacalah kutipan teks berikut!


Ketika berwisata, masyarakat diimbau untuk selalu waspada dan mematuhi protokol
kesehatan. Salah satunya, masyarakat jangan asal menyentuh fasilitas atau permainan
publik. Tempat yang berpotensi menimbulkan kerumunan sebaiknya dihindari.

Maksud kalimat bercetak miring pada kutipan teks tersebut adalah …


A. Jangan menyentuh fasilitas atau permainan untuk umum.
B. Jangan asal menyentuh sarana atau permainan untuk umum.
C. Sebaiknya tidak menyentuh fasilitas atau permainan untuk umum.
D. Fasilitas atau permainan untuk umum tidak higienis jadi jangan menyentuhnya.

3. Bacalah kutipan teks berikut!


Gempa bumi terjadi karena pergeseran atau gerakan lapisan dasar bumi dan letusan
gunung berapi yang dahsyat. Gempa bumi terjadi begitu cepat dengan dampak yang
sangat besar bagi lingkungan sekitarnya, seperti korban jiwa dan hancurnya bangunan.
Berdasarkan penyebab terjadinya, gempa bumi dapat digolongkan menjadi dua jenis,
yaitu gempa tektonik dan gempa vulkanik. Gempa tektonik terjadi karena lapisan kerak
bumi menjadi lunak sehingga mengalami pergeseran atau pergerakan. Sementara itu,
gempa vulkanik terjadi dikarenakan adanya letusan gunung berapi. Gempa vulkanik
lebih jarang terjadi dibandingkan dengan gempa tektonik.

Pernyataan yang sesuai isi kutipan teks tersebut adalah …


A. Gempa tektonik dapat terjadi karena adanya letusan gunung berapi yang dahsyat.
B. Gempa tektonik biasanya lebih jarang terjadi dibandingkan dengan gempa vulkanik.
C. Ada dua jenis gempa bumi berdasarkan penyebab terjadinya yaitu gempa vulkanik
dan gempa tektonik.
D. Gempa bumi terjadi sangat cepat memiliki dampak bagi lingkungan sekitarnya seperti
tsunami dan longsor.

TPM ASPD Paket. A Bahasa Indonesia 2


Dinas Pendidikan Kota Yogyakarta
www.giriwidodo.com
4. Bacalah kutipan teks berikut!
(1) Kucing termasuk karnivora. (2) Sebagai pemakan daging, kucing suka berburu tikus.
(3) Oleh karena itu, kucing dipelihara untuk memangsa tikus. (4) Akan tetapi,
kenyataannya tidak semua kucing berani memburu tikus.

Kalimat yang menyatakan pertentangan terdapat pada kalimat ….


A. (1)
B. (2)
C. (3)
D. (4)

5. Bacalah paragraf berikut!


Sampah dapat diolah kembali menjadi sesuatu yang lebih bermanfaat. Sampah botol
plastik dapat didaur ulang menjadi kerajinan tangan yang dapat menghasilkan uang.
Seorang bernama Piet Hein Eek mengubah sampah menjadi sebuah karya seni dan
furnitur. Hasil karya seni dan furnitur tersebut kemudian dijual ratusan hingga ribuan
dolar per satu karyanya.

Ide pokok paragraf tersebut adalah ….


A. sampah botol plastik dapat didaur ulang
B. sampah dapat dijadikan sebuah karya seni
C. sampah dapat diolah menjadi lebih bermanfaat
D. sampah dapat diubah menjadi karya seni dan furnitur

6. Bacalah kutipan teks berikut!


Penanaman resapan biopori air hujan perlu diperbanyak saat musim hujan seperti
sekarang. Selain sebagai penampung air hujan dalam tanah, resapan biopori memberi
manfaat bagi lingkungan hidup. Resapan air biopori yang ditanam saat musim hujan
memudahkan tanah menyerap air. Alat ini juga memberi dampak baik bagi lingkungan
hidup karena memanfaatkan sampah organik. Volume sampah rumah tangga dapat
berkurang bila sebagian besar dikumpulkan dan dijadikan alat resapan air. Bahan
organik tersebut akan memberi nutrisi pada tanah. Agar tanah semakin sehat dan hujan
tidak menggenang di permukaan tanah, maka perlu penanaman resapan biopori di
banyak titik.

Simpulan isi kutipan teks tersebut adalah …


A. Penanaman resapan biopori bermanfaat dan berdampak baik bagi lingkungan.
B. Bahan organik diperlukan untuk membuat resapan biopori agar tanah semakin sehat.
C. Resapan biopori perlu diperbanyak saat musim hujan sebagai penampung air hujan.
D. Penanaman resapan biopori dapat mengurangi volume sampah rumah tangga.

TPM ASPD Paket. A Bahasa Indonesia 3


Dinas Pendidikan Kota Yogyakarta
www.giriwidodo.com
7. Bacalah kutipan teks berikut!
Kementerian Pendidikan dan Kebudayaan (Kemendikbud) memberikan wewenang
kepada pemerintah daerah, sekolah, dan orang tua terkait kegiatan belajar mengajar
tatap muka. Bila ketiga pihak tersebut memberi izin, siswa dapat kembali masuk ke
sekolah dengan mematuhi protokol kesehatan.
Kebijakan penghentian PJJ sudah tepat. Hampir dua semester siswa menerima
pembelajaran jarak jauh (PJJ). Pelajaran yang diserap tidak seefektif ketika tatap muka
sehingga sebaiknya dihentikan. Selain itu, PJJ memiliki kendala, antara lain jaringan
yang bermasalah dan perangkat yang kurang mendukung.
Namun, membiarkan siswa masuk sekolah juga akan mendatangkan masalah, yaitu
berisiko memunculkan klaster baru yang besar. Hal ini menyulitkan pelacakan riwayat
dan menjadi beban pemerintah maupun masyarakat untuk melakukan karantina secara
menyeluruh.

Simpulan pendapat pro berdasarkan kutipan teks tersebut adalah …


A. Pelaksanaan pembelajaran tatap muka akan memunculkan klaster baru di sekolah.
B. Kebijakan penghentian PJJ sudah tepat karena dinilai kurang efektif dan mengalami
beberapa kendala.
C. Membiarkan siswa masuk sekolah di masa pandemi akan memunculkan klaster baru
dan menyulitkan karantina menyeluruh.
D. Kemendikbud memberikan kewenangan bagi pemda, sekolah, dan orang tua terkait
pemberian izin siswa dapat masuk sekolah.

8. Bacalah kutipan teks berikut!


(1) Pembelajaran moda daring saat ini dilaksanakan secara intensif, terlebih pada masa
pandemi Covid-19. (2) Guru membuat video pembelajaran, menjelaskan melalui media
live conference, dan melakukan tes secara daring. (3) Sayangnya, kegiatan
pembelajaran daring ini kurang efektif untuk memantau karakter para siswa, khususnya
dari segi kejujuran dan kedisiplinan. (4) Untuk itu, sekolah perlu merumuskan model
pembelajaran yang tepat di masa pandemi.

Pendapat kontra ditandai dengan kalimat ….


A. (1)
B. (2)
C. (3)
D. (4)

TPM ASPD Paket. A Bahasa Indonesia 4


Dinas Pendidikan Kota Yogyakarta
www.giriwidodo.com
9. Bacalah teks berikut!
Air Terjun Kedung Pedut
merupakan sumber mata air yang
berasal dari dalam bumi. Hal ini
membuat Kedung Pedut dijuluki surga
air Kulon Progo lantaran airnya yang
tidak pernah habis meski di musim
kemarau. Air terjun ini berada di Jalan
Kutogiri Gunung Kelir, Kembang,
Jatimulyo, Girimulyo, Kabupaten Kulon
Progo, Daerah Istimewa Yogyakarta.
Rute menuju tempat wisata ini tidak
sulit dan kita bisa melihat
pemandangan yang asri. Selain kedung, terdapat beberapa tempat istirahat yang
terbuat dari bambu sehingga menambah kesan alami. Kawasan wisata ini banyak
menarik wisatawan untuk mengunjunginya.
Daya tarik Air Terjun Kedung Pedut terletak pada warna airnya. Airnya terdiri dari
dua komponen warna, yaitu putih jernih dan hijau tosca. Putih jernih terjadi karena
aliran air deras yang berasal dari air terjun di samping kedung, sedangkan hijau tosca
terbentuk dari pantulan batuan di dasar sungai yang terkena sinar matahari.

Ringkasan teks tersebut adalah …


A. Air Terjun Kedung Pedut merupakan sumber mata air yang berasal dari dalam bumi
sehingga dijadikan wisata alam dengan keindahan warna airnya.
B. Air Terjun Kedung Pedut merupakan sumber mata air di daerah Kulon Progo yang
berasal dari dalam bumi sehingga dijuluki surga air Kulon Progo.
C. Air Terjun Kedung Pedut sebagai kawasan wisata alam di Kulon Progo yang banyak
menarik wisatawan untuk mengunjunginya.
D. Air terjun Kedung Pedut sebagai salah satu wisata alam yang menawarkan keindahan
warna air.

10. Bacalah kedua teks berikut!


Teks 1 Teks 2
Sebagian orang beranggapan bahwa Orang yang mencintai lingkungan
sampah menjadi sesuatu yang tidak tentunya lebih memilih menyelesaikan
dibutuhkan lagi, maka dari itu sampah masalah sampah dengan biodegradasi
harus dimusnahkan dengan cara dibakar. sampah organik. Daun kering, ranting,
Namun, bagi mereka yang mencintai alam, batang pohon, dan sisa-sisa makanan bisa
cara tersebut tidak menyelesaikan dijadikan sebagai pupuk kompos melalui
masalah bahkan memunculkan sebuah proses biodegradasi, sedangkan sampah
bencana baru. Sampah yang dibakar akan nonorganik yang memang tidak bisa
menghasilkan polusi udara berupa bau mengalami pembusukan dilakukan dengan
tidak sedap bahkan asap pembakaran menggunakan kembali, mengurangi, dan
yang dihasilkan dari pembakaran itu dapat mendaur ulang.
menipiskan lapisan ozon.

Perbedaan penggunaan bahasa kedua teks tersebut adalah ....


Teks 1 Teks 2
A kata kajian kata popular
B kata populer kata kajian
C bahasa resmi bahasa sehari-hari
D bahasa sehari-hari bahasa resmi

TPM ASPD Paket. A Bahasa Indonesia 5


Dinas Pendidikan Kota Yogyakarta
www.giriwidodo.com
11. Bacalah kedua teks berikut!
Teks 1 Teks 2
Epidemiolog dari Griffith University Dicky Malam tahun baru, Dinas Pariwisata dan
Budiman menyebut kebijakan Ekonomi Kreatif (Disparekraf) melakukan
pembatasan sosial berskala besar (PSBB) beberapa upaya untuk mencegah adanya
transisi merupakan strategi yang bersifat kerumunan di tempat usaha. Upaya
tambahan atau pelengkap dalam tersebut, yakni melakukan pengawasan
menangani pandemi Covid-19. mandiri, pengawasan secara terpadu atau
Pemerintah Provinsi DKI Jakarta, menurut gabungan, serta pelaporan melalui warga
Dicky, harus melakukan tes dan pelacakan atau media. "Dan mistery guest," tutur
lalu dilanjutkan dengan isolasi dan Bambang. Disparekraf DKI Jakarta tidak
karantina mandiri. "Tes dan pelacakan berencana untuk menggelar acara tahun
adalah strategi utama," ujar Dicky kepada baru, meski melalui virtual. "Dinas
Kompas.com, Selasa (8/12/2020) Pariwisata hingga saat ini tidak ada
rencana menggelar acara malam tahun
baru, walaupun secara virtual," kata
Bambang.

Perbedaan pola penyajian kedua teks tersebut diawali dengan unsur .…


Teks 1 Teks 2
A siapa kapan
B siapa bagaimana
C di mana mengapa
D bagaimana apa

TPM ASPD Paket. A Bahasa Indonesia 6


Dinas Pendidikan Kota Yogyakarta
www.giriwidodo.com
12. Bacalah teks berikut!
Buku Pengantar Teknik dan Manajemen Industri ditulis oleh Sritomo
Wignjosoebroto, seorang dosen senior Jurusan Teknik Industri, Institut Teknologi
Sepuluh Nopember (ITS) di Surabaya. Ia lahir pada 25 Oktober 1946 di Solo, berbagai
jabatan pernah dipegang sebagai akademisi di bidang Teknik Industri dan Manajemen
Industri. Penulis tercatat aktif sebagai anggota maupun pengurus berbagai organisasi
profesi maupun sosial kemasyarakatan, seperti Persatuan Insinyur Indonesia (PII),
Ikatan Sarjana Teknik Industri dan Manajemen Industri (ISTMI), dan lain-lain.
Isi buku ini meliputi berbagai bidang dalam ruang lingkup Teknik Industri. Semua
pembahasan tersebut mencakup disiplin teknik industri mulai dari yang mudah hingga
yang sulit.
Buku Pengantar Teknik dan Manajemen Industri disajikan secara sederhana
disertai contoh-contoh pengaplikasian pada setiap bab. Terdapat tabel, skema, dan
gambar yang membuat penjelasan yang disampaikan lebih mudah dipahami. Selain
itu, disertai soal-soal latihan pada setiap bab yang membuat mahasiswa mandiri
mengerjakan persoalan tersebut. Buku ini lebih fokus merangkum segala macam disiplin
teknik industri menjadi satu buku yang ringkas. Kedalaman isi buku sangat baik, tata
organisasi gagasan cukup baik, gaya penulisan baik, sehingga memberikan sumbangan
praktis maupun teoritis terhadap pembaca. Sayangnya, bab-bab pada buku ini belum
tersusun secara sistematis dan pembahasannya terlalu singkat.
Namun, secara keseluruhan ulasan yang telah dibuat bahwa pengantar teknik dan
manajemen industri baik untuk dibaca karena banyak keunggulan serta kelengkapan isi
buku yang ditawarkan. Buku ini cocok dibaca mahasiswa maupun orang yang ingin lebih
jauh mendalami permasalahan-permasalahan dalam ruang lingkup teknik industri
maupun manajemen produksi industri.

Salah satu keunggulan buku yang terdapat pada teks tersebut adalah ….
A. Buku Pengantar Teknik dan Manajemen Industri terdapat soal-soal di akhir materi
yang tidak diberikan pembahasan.
B. Bab-bab pada buku belum tersusun secara berimbang yang berdasarkan
masing-masing permasalahan yang dibahas, terlalu singkat.
C. Keseluruhan ulasan buku Pengantar Teknik dan Manajemen Industri baik untuk
dibaca karena banyak keunggulan serta kelengkapan isi buku yang ditawarkan.
D. Buku Pengantar Teknik dan Manajemen Industri tersusun atas beberapa mata kuliah
yang secara sederhana dengan contoh-contoh pengaplikasian pada setiap bab.

TPM ASPD Paket. A Bahasa Indonesia 7


Dinas Pendidikan Kota Yogyakarta
www.giriwidodo.com
13. Bacalah kutipan teks berikut!
Kegiatan pembelajaran tatap muka selama masa pandemi butuh kewaspadaan ekstra.
Dikhawatirkan, kasus baru akan bertebaran di klaster sekolah bila tidak menaati
protokol kesehatan dengan ketat. Jumat, 10 November 2020 Menteri Pendidikan dan
Kebudayaan, Nadiem Makarim memberi pengumuman soal pelaksanaan pendidikan
Tahun Ajaran 2020/2021. Ia menyatakan bahwa mulai Januari 2021, izin pembelajaran
tatap muka diserahkan kepada jajaran pemerintah daerah dan orang tua. Dalam
mempersiapkan sekolah tatap muka, seluruh pihak wajib melindungi siswa. Protokol
kesehatan selama pandemi Covid-19 harus diterapkan dengan ketat saat pembelajaran
tatap muka dilaksanakan. Hal tersebut dilakukan untuk memutus mata rantai
penyebaran Covid-19.

Komentar yang tepat berdasarkan kutipan teks tersebut adalah ...


A. Semua orang tua seharusnya mengizinkan anaknya bersekolah sekalipun di masa
pandemi karena pendidikan adalah hal yang paling utama untuk menyiapkan masa
depan mereka.
B. Jika pembelajaran tatap muka dilaksanakan, pemerintah daerah, orang tua, dan
warga sekolah harus dengan ketat menerapkan protokol kesehatan agar kasus baru
tidak menyebar di sekolah.
C. Orang tua tidak perlu khawatir tentang pembelajaran tatap muka di masa pandemi
karena selama kegiatan tersebut dilaksanakan, pemerintah telah menjamin
kesehatan para peserta didik.
D. Pemerintah daerah harus tetap melaksanakan kegiatan pembelajaran jarak jauh
karena jika pembelajaran tatap muka dilaksanakan, penyebaran wabah Covid-19
akan terus meluas di berbagai klaster.

14. Bacalah kutipan cerpen berikut!


“Sudahlah Din, tenangkan hatimu!” kata Santi pada Dinda yang sudah terlanjur
meradang.
“Bagaimana mungkin aku bisa tenang mendengar ejekannya pada adikku
satu-satunya, “ ucap Dinda.
“Memang adikku berkebutuhan khusus, tapi harusnya dia dilindungi bukannya
malah diejek,” lanjut Dinda. Dinda pun menghela napas panjang. Tepukan pelan Santi di
bahu Dinda berulang-ulang membuat Dinda berangsur lebih tenang.

Makna kata meradang pada kutipan cerpen tersebut adalah ….


A. kalut
B. galau
C. geram
D. kecewa

TPM ASPD Paket. A Bahasa Indonesia 8


Dinas Pendidikan Kota Yogyakarta
www.giriwidodo.com
15. Bacalah kutipan fabel berikut!
“Penyu, kenapa telurmu sangat banyak, tetapi hanya beberapa yang berhasil hidup
setelah menetas?” tanya anak kepiting kepada penyu yang baru saja bertelur di pantai.
Sambil tersenyum, penyu menjawab,” Kau tahu, Anak Muda, usia kami bisa
mencapai ratusan tahun. Kalau kami semua hidup, maka lautan ini akan dipenuhi penyu.
Tak ada tempat untuk yang lain.”
“Tempat untukku juga?” anak kepiting tertawa. “Hehehe, ya, untukmu juga,” jawab
penyu dengan ramah.

Makna tersurat dari kutipan fabel tersebut adalah …


A. Anak kepiting yang ingin mengetahui kehidupan penyu di laut luas selama ratusan
tahun.
B. Penyu menjelaskan kepada anak kepiting mengenai jumlah anak penyu yang hidup
setelah menetas hanya sedikit.
C. Tuhan sungguh sangat adil. Binatang yang usia hidupnya sangat lama hanya sedikit
yang bisa hidup setelah menetas.
D. Rantai kehidupan di lautan ini sudah diatur sedemikian rupa oleh Tuhan sehingga
kelangsungan hidup di lautan dapat terjaga.

16. Bacalah kutipan cerpen berikut!


Buurrrrrrrrrr...kaki bapak terpeleset. Bapak berusaha menggapai-gapaikan tangan
kanannya untuk mencari sesuatu yang bisa diraih dan menahannya agar tidak terbawa
arus. Jantungku hampir copot, perasaanku kacau, rasa takutku memuncak, aku tak kuat
menahan tangis. Tangan kiri bapak semakin erat memegangi plastik tepat pada posisi
ikatannya. Aku kebingungan mondar-mandir mencari sesuatu yang bisa diraih bapak. Di
sekelilingku tidak ada apa-apa. Aku semakin putus asa.

Bagian alur kutipan cerpen tersebut adalah .…


A. orientasi
B. komplikasi
C. konflik
D. koda

17. Bacalah kutipan fabel berikut!


Sekawanan kambing hitam telah mengepung persembunyian orang utan. Orang utan
tidak bisa lari ke mana-mana lagi. Akhirnya, orang utan mengangkat tangan. Orang
utan berjanji tidak akan berbuat jahat lagi.

Makna simbol mengangkat tangan pada kutipan fabel tersebut adalah ….


A. berduka cita
B. putus asa
C. menyerah
D. bertobat

TPM ASPD Paket. A Bahasa Indonesia 9


Dinas Pendidikan Kota Yogyakarta
www.giriwidodo.com
18. Bacalah kutipan cerpen berikut!
Risa berjalan sendirian menuju perpustakaan sekolah setelah jam pelajaran selesai.
Namun, Risa sangat kecewa karena perpustakaan sekolah belum selesai direnovasi.
Risa mengharapkan perpustakaan sekolah segera dibuka agar dapat membaca banyak
buku di sana. Berhubung perpustakaan sekolah masih tutup, dengan hati kecewa Risa
bergegas pulang ke rumah. Sesampainya di rumah, Risa berganti pakaian, makan,
kemudian tidur. Sore hari Risa duduk di teras rumahnya sendirian, ia berpikir akan
membuat perpustakaan sendiri. Lagi pula, buku yang dimilikinya cukup banyak.
Risa mencoba membuat perpustakaan dengan beberapa barang bekas. Risa mulai
mencari ruang kosong sebagai perpustakaan. Akhirnya, Risa menemukan satu ruangan
di sudut rumahnya yang tidak terpakai. Risa merapikan ruangan tersebut dan menyulap
kardus-kardus bekas menjadi rak buku. Risa melapisi rak buku itu dengan kertas
bergambar agar rapi dan tampak menarik. Selanjutnya, Risa menata buku yang
dimilikinya. Risa memisahkan buku menjadi beberapa kategori, seperti cerita anak,
dongeng, novel, pengetahuan, pelajaran, dan lain-lain. Risa menyapu dan mengepel
lantai serta menyusun karpet yang tidak terpakai dari gudang. Risa berencana
membuka perpustakaannya besok sore supaya teman-temannya dapat membaca
buku-buku yang dimiliki Risa.

Pesan moral yang terdapat dalam kutipan cerpen tersebut adalah ...
A. Tekad yang kuat disertai usaha optimal akan mewujudkan keinginan.
B. Orang tua bangga terhadap tekad dan usaha yang dilakukan anaknya.
C. Peran serta dan dukungan semua pihak dibutuhkan dalam meraih keinginan.
D. Kesabaran menghadapi permasalahan menunjukkan sikap terpuji dalam hidup.

19. Bacalah kutipan fabel berikut!


Elang Raja menunjuk Langja, putra bungsunya sebagai pengganti.
“Aku sudah terlalu tua untuk memimpin para elang di hutan dan pegunungan ini,”
kata Elang Raja.
“Aku putra sulung, seharusnya ayahanda memilih aku sebagai pengganti,” Langir
tiba-tiba muncul.
Elang Raja tetap pada pendiriannya.
Langir akan memperjuangkan dirinya untuk menjadi pengganti ayahnya. Ia akan
menggunakan segala cara. Akhirnya, ia menemukan cara. Ia meminta ramuan kepada si
Landak. Si Landak memberikan ramuan yang dapat melemahkan seluruh tulang
belulang. Akhirnya, Langja menjadi lumpuh.
“Elang yang tak bisa berjalan dan terbang tak pantas menjadi raja!” kata Langir.
Kemudian ia menobatkan diri sebagai raja.

Penyebab konflik kutipan fabel tersebut yang paling tepat adalah ...
A. Langja tidak dapat berjalan dan terbang seperti elang lainnya.
B. Langir setuju jika adiknya diangkat menggantikan ayah mereka.
C. Langir memberi ramuan yang dapat melemahkan tulang belulang.
D. Elang Raja menunjuk Langja, putra bungsunya sebagai pengganti.

TPM ASPD Paket. A Bahasa Indonesia 10


Dinas Pendidikan Kota Yogyakarta
www.giriwidodo.com
20. Bacalah kedua kutipan cerpen berikut!
Kutipan Cerpen 1 Kutipan Cerpen 2
“Ueenak aja bilang maaf, ndak bisa. Beberapa warga kampung terkesima
Pokoknya harus ganti!” Pardi melotot melihat penampilan Yuyun sekarang. Tak
sambil menunjuk pecahan gerabah yang ada lagi bunyi gemerincing gelang,
berceceran. menemani riuh tawa Yuyun yang ndeso.
“Berapa, Mang?” tanya lelaki itu Yuyun yang sekarang begitu kalem,
tenang sembari membuka dompetnya. bicaranya teratur dan terukur persis guru
“Lima ratus ribu, ya segitu kepribadian. Bahkan saat Yuyun makan
harganya,” Pardi melirik lelaki rapi dengan dan minum, caranya membuka mulut
bau minyak wangi yang tidak akrab di membuat Mimin, teman masa kecilnya
hidungnya. Ia mencium aroma uang pada terbelalak dan kenyang seketika. Yuyun
lelaki berdasi dengan paras macam artis memang berubah menjadi sosok berbeda.
youtube di hp istrinya.

Pola pengembangan cerita berdasarkan kutipan cerpen tersebut diawali dengan ….


Kutipan Cerpen 1 Kutipan Cerpen 2
A orientasi komplikasi
B komplikasi resolusi
C komplikasi orientasi
D resolusi komplikasi

21. Bacalah kedua kutipan fabel berikut!


Kutipan Fabel 1 Kutipan Fabel 2
Pada suatu siang, seekor kancil Pada suatu hari anak kerang
sedang duduk santai di bawah pohon. Ia mengadu pada ibunya sebab sebutir pasir
ingin menghabiskan waktu siangnya tajam memasuki tubuhnya yang merah
dengan menikmati suasana hujan yang dan lembek.
asri dan sejuk. Beberapa waktu “Anakku,” kata sang Ibu sambil
kemudian, perutnya keroncongan. bercucuran air mata, “Tuhan tidak
Kancil yang terkenal cerdik itu memberikan pada kita, bangsa kerang,
merasa sangat lapar. Ia sedang berpikir sebuah tangan pun, sehingga Ibu tidak bisa
bagaimana bisa mendapatkan mentimun menolongmu.” Sang Ibu terdiam sejenak.
yang letaknya berada di seberang sungai. “Sakit sekali, aku tahu anakku, tetapi
Tiba-tiba terdengar suara kecipak keras terimalah itu sebagai takdir alam. Kuatkan
dari dalam sungai. Ternyata itu adalah hatimu. Jangan terlalu lincah lagi. Kerahkan
Buaya. semangatmu melawan rasa ngilu dan nyeri
Kancil yang cerdik itu pun punya ide yang menggigit. Balutlah pasir itu dengan
jitu untuk menghilangkan rasa laparnya. getah perutmu. Hanya itu yang bisa kau
Ia bangkit dari duduknya dan berjalan perbuat,” kata Ibunya dengan sendu dan
cepat ke arah sungai untuk menghampiri lembut.
Buaya. Rupanya dia akan memanfaatkan “Iya, Ibu. Nasihatmu akan aku
si Buaya untuk mendapatkan mentimun. lakukan.” Sahut si Anak Kerang.

Perbedaaan penggunaan bahasa kedua kutipan fabel tersebut adalah ….


Kutipan Fabel 1 Kutipan Fabel 2
A narasi dialog
B makna kias makna lugas
C bahasa baku bahasa sehari-hari
D tidak ada kata sandang ada kata sandang

TPM ASPD Paket. A Bahasa Indonesia 11


Dinas Pendidikan Kota Yogyakarta
www.giriwidodo.com
22. Bacalah kutipan fabel berikut!
(1) Burung kakaktua jambul kuning itu biasa disapa Dija. (2) Ia seekor burung yang
ceroboh sehingga terpisah dari keluarga. (3) Padahal, Dija sudah berulang kali
diingatkan agar berhati-hati karena banyak hewan lain yang memasang jebakan
perangkap. (4) Kini Dija berada di sarang Lim, si burung elang yang siap memangsanya.
(5) Susah payah ia mencoba melepaskan diri, tapi semuanya sia-sia.

Bukti latar tempat pada kutipan fabel tersebut terdapat pada kalimat ....
A. (2)
B. (3)
C. (4)
D. (5)

23. Bacalah kutipan cerpen berikut!


(1) Kebiasaan berbagi tugas kami sepakati karena belum mampu menggaji pembantu.
Semua berjalan lancar tak ada yang keteteran.
(2) Sampai pagi itu Bu Lastri, tetangga kami, ke rumah. Suaminya meninggal sebulan
yang lalu.
(3) “Pak, Bu!” ucapnya sambil menatap kami ragu. Ia tidak segera melanjutkan
kata-katanya. Tiba-tiba ia menunduk dalam, menyembunyikan matanya yang
berkaca-kaca.
(4) Aku dan istriku berpandangan. Kami paham. Selama ini memang kami dekat, saling
membantu jika di antara kami kesulitan.
(5) “Bu,” kata istriku lirih, “Bu Lastri bisa membantu mengantar kue buatan saya ini ke
warung-warung langganan saya. Belum banyak yang bisa didapat, Bu. Tapi kalau
kita tekuni mudah-mudahan usaha ini bisa berkembang,” ujar Bu Harno.
(6) “Alhamdulillah!” seru Bu Lastri lega. Wajahnya seketika berbinar.
“Terimakasih, Pak Harno, Bu Harno!” seru Bu Lastri tak dapat menyembunyikan
bahagianya.

Bukti watak tokoh Bu Harno suka menolong terdapat pada kalimat ....
A. (1)
B. (3)
C. (5)
D. (6)

TPM ASPD Paket. A Bahasa Indonesia 12


Dinas Pendidikan Kota Yogyakarta
www.giriwidodo.com
24. Bacalah kutipan cerpen berikut!
Lea pun ketakutan, tapi guru tersebut berusaha menenangkannya.
“Lea tak apa, Ibu di sini, kamu bisa bantu teman-temanmu? Sekali ini aja Lea
tolonglah,” kata Bu Guru.
Sebenarnya Lea tak mau menolong mereka. Beberapa minggu lalu Lea hampir saja
dikeluarkan dari sekolah karena mereka tiba-tiba berdemo dan menginginkan Lea pergi
dari sekolah.
Lea tak pernah minta untuk berbeda. Lea hanya ingin mereka tahu bahwa dirinya
seorang indigo dan tetaplah manusia.
Lea seketika menyelinap menuju kerumunan teman-temannya yang sedang
menjerit-jerit. Lea pun segera menghentikan langkahnya tepat pada sesosok makhluk
yang selalu dibencinya, yang tak tahu tempat serta hanya menyusahkan.
“Pergi kamu!” ucap Lea saat ada di depan makhluk yang telah menimbulkan
kegaduhan satu sekolah.
Tenaga Lea terserap serta tubuhnya benar-benar lemas, pada akhirnya ia jatuh
pingsan karena kelelahan.
Lea kadang berharap matanya tak terbuka lagi jika melihat mereka yang tak sama
dengannya. Rasanya sudah sangat lelah, tetapi kenyataannya Tuhan masih
memberikan Lea umur panjang.
Hanya saja yang berbeda, saat membuka mata ada beberapa teman di kelasnya
yang telah menunggu Lea sadar dan langsung mengucapkan terima kasih. Hati Lea pun
terasa sangat hangat serta isak tangis tidak lagi tertahankan.

Komentar yang tepat terhadap kutipan teks tersebut adalah …


A. Latar suasana yang dari awal hingga akhir menyedihkan.
B. Sudut pandang orang ketiga membuat cerita lebih menarik.
C. Alur yang digunakan berbelit – belit sehingga membosankan pembaca.
D. Penggambaran watak tokoh melalui tingkah laku membuat cerita kurang masuk akal.

25. Bacalah kalimat berikut!


Jeruk sangatlah beragam dan beberapa [...] dapat bersilangan serta menghasilkan
[…] yang memiliki karakter yang khas.

Istilah yang tepat untuk melengkapi bagian rumpang pada kalimat tersebut adalah .…
A. hibrida antarspesies, klasifikasi
B. klasifikasi, hibrida antarspesies
C. hibrida antarspesies, spesies
D. spesies, hibrida antarspesies

TPM ASPD Paket. A Bahasa Indonesia 13


Dinas Pendidikan Kota Yogyakarta
www.giriwidodo.com
26. Bacalah kalimat-kalimat acak berikut!
Cara Membuat Hand Sanitizer Alami
(1) Siapkan 10 lembar daun sirih dan 3 batang lidah buaya.
(2) Setelah dingin, ambil daging lidah buaya.
(3) Potong kasar daun sirih, tambahkan 200ml air lalu masukkan ke dalam panci dan
rebus hingga mendidih.
(4) Masukkan daging lidah buaya ke dalam blender, tambahkan sedikit air.
(5) Blender hingga hancur kemudian saringlah.
(6) Setelah itu, matikan api dan tunggu hingga dingin.
(7) Masukkan rebusan sirih dan lidah buaya yang sudah diblender ke dalam botol
spray.

Urutan kalimat yang tepat dalam penulisan teks prosedur tersebut adalah .…
A. (1)-(3)-(6)-(2)-(4)-(5)-(7)
B. (1)-(3)-(4)-(5)-(6)-(2)-(7)
C. (1)-(4)-(3)-(6)-(2)-(5)-(7)
D. (1)-(4)-(3)-(5)-(7)-(2)-(6)

27. Bacalah paragraf berikut!


Tiwul adalah makanan tradisional dari Gunungkidul yang terbuat dari olahan singkong.
Selain rasanya yang khas, tiwul dapat dijadikan sebagai makanan pengganti nasi.
Makanan ini sudah terkenal sejak zaman dahulu dan menjadi salah satu warisan kuliner
bagi masyarakat Yogyakarta, terutama Gunungkidul. Tiwul dibuat melalui beberapa
proses. […]. Singkong yang sudah kering tersebut biasa disebut gaplek. Gaplek
ditumbuk hingga halus dan menjadi tepung lalu dikukus hingga matang dan menjadi
tiwul.

Kalimat yang tepat untuk melengkapi paragraf tersebut adalah ...


A. Makanan ini sangat mudah kita temukan di pasar-pasar tradisional.
B. Proses tersebut membutuhkan ketelatenan dan waktu yang cukup lama.
C. Pertama, singkong dikupas dan dicuci bersih kemudian dipotong-potong.
D. Dalam proses pembuatannya, singkong dikupas dan dijemur hingga kering.

TPM ASPD Paket. A Bahasa Indonesia 14


Dinas Pendidikan Kota Yogyakarta
www.giriwidodo.com
28. Bacalah teks berikut!
Biografi Gandhi menceritakan perjalanan hidup Mahatma Gandhi dari kecil hingga
menjadi tokoh yang sangat disegani pemimpin-pemimpin negara di dunia. Cara
hidupnya sangat sederhana, bahkan bisa dikatakan di bawah standar hidup seorang
pemimpin negara.
Gandhi memutuskan untuk menjadi vegetarian murni karena melihat seekor
kambing yang menangis ketika akan disembelih. Ia juga sangat teguh hati. Hal itu
nampak ketika ia sakit keras dan dokter memintanya untuk mengonsumsi kaldu. Gandhi
menolak dan mengatakan bahwa ia lebih baik mati daripada melanggar sumpahnya
sebagai vegetarian.
[…].

Kalimat yang tepat untuk melengkapi teks tersebut adalah ...


A. Biografi Gandhi memiliki keunggulan dan kelemahan. Buku ini ditulis dengan bahasa
yang lugas dan mengalir. Sayangnya, beberapa istilah bahasa India menyulitkan
pembaca.
B. Dari segi bahasa, buku ini ditulis dengan sangat menarik, lugas, dan mudah dipahami.
Selain itu, dilengkapi dengan ilustrasi foto-foto. Buku ini baik dibaca oleh semua
kalangan usia.
C. Buku ini menggunakan bahasa yang sederhana dan sangat mudah dipahami siapa
saja. Sebenarnya buku ini hanya layak dibaca oleh kalangan atas dan pemerhati
kemanusiaan.
D. Buku ini menggunakan beberapa istilah dari bahasa India yang sulit dipahami. Selain
itu, cover buku ini tidak berwarna sehingga tidak menarik. Harganya pun terlalu
mahal. Buku ini mudah didapat karena sudah tersedia di toko-toko buku.

29. Bacalah kalimat berikut!


Badan Amil Zakat (Baznas) Kabupaten Sleman menyerahkan santunan kepada 500
anak yatim atau piatu.

Variasi kata yang tepat untuk mengganti kata menyerahkan pada kalimat tersebut
adalah …
A. mempersembahkan
B. memasrahkan
C. menyodorkan
D. memberikan

30. Bacalah kalimat berikut!


Pemerintah mendahulukan orang-orang yang berada di lini depan untuk mendapatkan
vaksin.

Variasi kalimat tersebut yang tepat adalah ...


A. Orang-orang yang berada di lini depan mendahulukan pemerintah untuk
mendapatkan vaksin.
B. Orang-orang yang berada di lini depan didahulukan untuk mendapatkan vaksin oleh
pemerintah.
C. Pemerintah didahulukan untuk mendapatkan vaksin oleh orang-orang yang berada di
lini depan.
D. Untuk mendapatkan vaksin, orang-orang di lini depan mendahulukan pemerintah.

TPM ASPD Paket. A Bahasa Indonesia 15


Dinas Pendidikan Kota Yogyakarta
www.giriwidodo.com
31. Perhatikan diagram berikut!

Laporan yang sesuai diagram tersebut adalah …


A. Kegiatan ekstrakurikuler paduan suara dan menari paling banyak diminati siswa
dengan jumlah masing-masing 25 peserta. Kemudian diikuti ekstrakurikuler renang
dengan jumlah 20 peserta, futsal dengan jumlah 15 peserta, sedangkan melukis
paling sedikit diminati siswa dengan jumlah 10 peserta.

B. Kegiatan ekstrakurikuler paduan suara dan menari paling banyak diminati siswa
dengan jumlah masing-masing 25 peserta. Kemudian diikuti ekstrakurikuler futsal
dengan jumlah 20 peserta, renang dengan jumlah 15 peserta, sedangkan melukis
paling sedikit diminati siswa dengan jumlah 10 peserta.

C. Kegiatan ekstrakurikuler paduan suara dan melukis paling banyak diminati siswa
dengan jumlah masing-masing 25 peserta. Kemudian diikuti ekstrakurikuler renang
dengan jumlah 20 peserta, futsal dengan jumlah 15 peserta, sedangkan menari
paling sedikit diminati siswa dengan jumlah 10 peserta.

D. Kegiatan ekstrakurikuler paduan suara paling banyak diminati siswa dengan jumlah
25 peserta. Kemudian diikuti ekstrakurikuler renang dengan jumlah 20 peserta, futsal
dengan jumlah 15 peserta, sedangkan melukis paling sedikit diminati siswa dengan
jumlah 10 peserta.

TPM ASPD Paket. A Bahasa Indonesia 16


Dinas Pendidikan Kota Yogyakarta
www.giriwidodo.com
32. Bacalah wawancara berikut!
Ardi : “Jadi, apa alasan Saudara ingin bekerja di sini?”
Neli : “Saya ingin mengembangkan karier saya sebagai editor, Pak.”
Ardi : “Bagus, tetapi Saudara harus siap bekerja di bawah tekanan, mampu
memenuhi target kami. Dalam satu hari harus mengedit dua naskah novel.”
Neli : “Tidak masalah asalkan gajinya di atas UMR.”
Ardi : “Saudara mau mengembangkan karier atau mencari uang?”
Neli : “Bekerja secara profesional.”
Ardi : “Bagus, tetapi kami belum bisa memenuhi harapan Saudara dengan gaji di
atas UMR.”
Neli : “Kalau begitu, saya bukan orang yang tepat berada di sini. Terima kasih.
Selamat pagi!”

Narasi yang sesuai dengan teks wawancara tersebut adalah…


A. Pagi itu Ardi mewawancarai Neli. Neli ingin bekerja secara profesional dengan gaji di
atas UMR. Namun, Ardi belum bisa memenuhinya. Neli mengundurkan diri.
B. Pagi itu Ardi mengajak Neli berbicara di kantornya. Ardi menawari Neli bekerja
sebagai editor. Namun, Neli menolaknya karena harus bekerja profesional dan
gajinya di bawah UMR.
C. Pagi itu Ardi berbicang-bincang dengan Neli. Ardi menawarkan pekerjaan sebagai
editor. Namun, Neli menolaknya karena gajinya masih di bawah UMR dan ia tidak
bisa bekerja di bawah tekanan.
D. Pagi itu Ardi mewawancarai Neli. Neli ingin bekerja sebagai editor secara profesional.
Neli ingin mendapat gaji di atas UMR, tetapi Ardi belum bisa memenuhinya. Neli pun
mengurungkan niat bekerja di tempat itu.

33. Bacalah kalimat berikut!


Masyarakat diharap tetap optimis dan tidak frustasi karena gejolak ekonomi akibat
pandemi Covid-19.

Kata tidak sesuai kaidah pada kalimat tersebut adalah ….


A. pandemi
B. frustasi
C. gejolak
D. optimis

34. Bacalah teks berikut!


(1) Korban gempa tektonik di Kalimantan masih tinggal di tenda-tenda darurat. (2)
Segenap lapisan masyarat dengan berbagai cara membantu meringankan penderitaan
korban, termasuk para siswa. (3) Di sekolah mengadakan pengumpulan dana. (4) Jika
sudah terkumpul, dana tersebut akan ditransfer secepatnya.

Kalimat dalam paragraf tersebut yang tidak sesuai kaidah adalah .…


A. (1)
B. (2)
C. (3)
D. (4)

TPM ASPD Paket. A Bahasa Indonesia 17


Dinas Pendidikan Kota Yogyakarta
www.giriwidodo.com
35. Bacalah kalimat berikut!
Kebijakan pemerintah mengenai pemberlakuan pembatasan kegiatan masyarakat
(PPKM) di beberapa wilayah Jawa-Bali pada 25 Januari 2021 hingga 8 Februari 2021
[…] perekonomian masyarakat.

Kata bentukan yang tepat untuk melengkapi kalimat tersebut adalah ….


A. mempengaruhi
B. memengaruhi
C. berpengaruh
D. dipengaruhi

36. Bacalah kalimat berikut!


Margarin dan mentega serupa tetapi tidak sama. Margarin dibuat dari krim susu atau
lemak hewani, […] mentega terbuat dari lemak nabati.

Konjungsi yang tepat untuk melengkapi bagian yang rumpang adalah ….


A. agar
B. karena
C. sehingga
D. sedangkan

37. Bacalah kalimat berikut!


Pemberian swab menghasilkan antibodi yang dapat melawan virus penyebab infeksi.

Perbaikan kata swab dalam kalimat tersebut adalah .…


A. rapid antibodi
B. swab PCR
C. vaksin
D. rapid

38. Bacalah kalimat berikut!


Untuk mengetahui baik buruk pribadi seseorang, dapat dilihat dari tingkah lakunya
sehari-hari.

Perbaikan kalimat tersebut adalah …


A. Baik buruk pribadi seseorang dapat dilihat dari tingkah lakunya sehari-hari.
B. Baik atau pun buruk pribadi seseorang dapat dilihat dari pribadinya sehari-hari.
C. Untuk mengetahui pribadi seseorang dapat dilihat dari tingkah lakunya sehari-hari.
D. Untuk mengetahui baik buruk pribadi seseorang dapat dilihat dari tingkah lakunya.

TPM ASPD Paket. A Bahasa Indonesia 18


Dinas Pendidikan Kota Yogyakarta
www.giriwidodo.com
39. Bacalah paragraf berikut!
(1) Setiap musim kemarau, pohon-pohan jati menjadi kering. (2) Daunnya
berguguran. (3) Proses itu disebut juga meranggas atau transpirasi. (4) Transpirasi
dibedakan menjadi tiga tipe, yaitu kutikula, stomata, dan lentikuler. (5) Proses tersebut
dapat terjadi apabila cuaca panas. (6) Dengan demikian, pohon jati menggugurkan
daunnya agar cadangan air berfokus ke akar sehingga mampu bertahan hidup sampai
musim hujan tiba.

Perbaikan kalimat nomor (4) yang tepat agar menjadi paragraf padu adalah …
A. Hal ini dilakukan untuk mengurangi penguapan yang terjadi pada pohon jati agar
dapat bertahan hidup.
B. Pohon jati yang gagal meranggas berarti gagal beradaptasi sehingga tidak mampu
bertahan dan akan mati.
C. Transpirasi adalah proses hilangnya air dan karbon dioksida dari jaringan hidup
tanaman yang terletak di atas permukaan tanah.
D. Tumbuhan yang meranggas di musim kemarau antara lain pohon
jati, mahoni, kedondong, randu, dan lain sebagainya.

40. Bacalah kalimat berikut!


Hendrawan tidak sekadar menjadi kuli di perusahaan besar dan ternama, tetapi ia juga
berperan membesarkan nama perusahaan tersebut.

Alasan kesalahan penggunaan kata dalam kalimat tersebut karena ….


A. kata sekadar merupakan kata tidak baku
B. kata kuli memiliki nilai rasa negatif
C. kata kuli memiliki nilai rasa positif
D. kata yang digunakan tidak sejajar

41. Bacalah kalimat berikut!


Untuk mempersingkat waktu, marilah rapat ini segera dimulai.

Alasan ketidakefektifan kalimat tersebut karena ….


A. kalimat tersebut bermakna ganda
B. tanda koma seharusnya diganti dengan konjungsi
C. kata untuk seharusnya digunakan di tengah kalimat
D. kalimat tidak logis karena waktu tidak dapat dipersingkat

42. Bacalah paragraf berikut!


Kecepatan laju pertumbuhan kubah lava Gunung Merapi menurun drastis
pascarentetan aktivitas tinggi pekan lalu. Sebelumnya, kecepatan pertumbuhan kubah
lava gunung ini rata-rata 8.000 meter kubik per hari, sekarang kecepatan
pertumbuhan kubah itu hanya tersisa separuhnya. Letusan eksplosif Merapi dapat
mengakibatkan lontaran material vulkanik mencapai radius 3 km.

Alasan ketidakpaduan paragraf tersebut karena ….


A. terdapat kalimat penjelas yang tidak mendukung gagasan utama
B. seluruh kelimat penjelas tidak mendukung gagasan utama
C. dalam satu paragraf terdapat dua gagasan utama
D. kalimat utama terdapat pada seluruh paragraf

TPM ASPD Paket. A Bahasa Indonesia 19


Dinas Pendidikan Kota Yogyakarta
www.giriwidodo.com
43. Bacalah teks berikut!
(1) Dewi sangat menggemari catur. (2) Olah raga ini tidak membutuhkan fisik yang kuat,
tetapi konsentrasi dan ketelitian. (3) Karena tekun berlatih, Dewi sering menjuarai
pertandingan catur. (4) Saat ini, Dewi memiliki gelar Kandidat Master dan diharapkan
dapat meraih gelar Grand Master pada akhir tahun 2024.

Ejaan yang salah terdapat pada kalimat ….


A. (4)
B. (3)
C. (2)
D. (1)

44. Bacalah kalimat-kalimat berikut!


(1) Rinda sampai di Stasiun Tugu Yogyakarta pada pukul 07:35 WIB.
(2) Anggaran instansi tempat ia bekerja tahun ini mencapai Rp400.000.000,00.
(3) ”Kita harus saling menolong dalam suasana yang serba sulit ini,” kata ibu saya.
(4) Hasil penelitian Muhammad Rendika, M. Hum. sangat bermanfaat dalam kehidupan.

Penggunaan tanda baca yang salah terdapat pada kalimat ….


A. (4)
B. (3)
C. (2)
D. (1)

45. Bacalah kalimat-kalimat berikut!


(1) Penggunaan masker kain satu lapis tidak memenuhi standardisasi kesehatan.
(2) Tubuh akan memberi respon jika terjadi infeksi melalui panas atau demam.
(3) Tidak menjaga jarak atau berkerumun akan beresiko tertular virus Covid-19.
(4) Mencuci tangan dengan sabun dan air mengalir merupakan tindakan prefentif.

Kalimat dengan ejaan yang benar terdapat pada ….


A. (1)
B. (2)
C. (3)
D. (4)

46. Bacalah kalimat-kalimat berikut!


(1) “Wortel memiliki banyak kandungan gizi mulai dari vitamin A, vitamin B, vitamin C,
vitamin E, dan vitamin K.” kata paman.
(2) “Kamu harus berhati-hati dalam perjalanan ke Jakarta nanti malam! Perjalanan kali
ini tidak ditemani ayah.”
(3) Ayah berkata: “Salah satu cara mencegah terpapar virus yang menyebabkan
covid-19 yaitu dengan menjaga daya tahan tubuh.”
(4) “Dengan cara apa kamu akan membeli tanah itu! Harganya sangat tinggi,
sedangkan uang kita tidak cukup untuk membelinya.”

Penggunaan tanda baca yang tepat terdapat pada ….


A. (1)
B. (2)
C. (3)
D. (4)

TPM ASPD Paket. A Bahasa Indonesia 20


Dinas Pendidikan Kota Yogyakarta
www.giriwidodo.com
47. Bacalah kalimat berikut!
Batik merupakan kerajinan bernilai seni tinggi yang menjadi ciri khas bangsa indonesia,
terutama di pulau jawa.

Perbaikan penulisan ejaan pada kalimat tersebut adalah …


A. Batik merupakan kerajinan bernilai seni tinggi yang menjadi ciri khas bangsa
Indonesia, terutama di Pulau Jawa.
B. Batik merupakan kerajinan bernilai seni tinggi yang menjadi ciri khas Bangsa
Indonesia, terutama di Pulau Jawa.
C. Batik merupakan kerajinan bernilai seni tinggi yang menjadi ciri khas bangsa
Indonesia, terutama di pulau Jawa.
D. Batik merupakan kerajinan bernilai seni tinggi yang menjadi ciri khas bangsa
Indonesia, terutama di pulau jawa.

48. Bacalah kalimat berikut!


Ibu membeli: buku, pensil, dan tinta: baju, celana, dan kaus: pisang, apel, dan jeruk.

Perbaikan penggunaan tanda baca kalimat tersebut adalah ...


A. Ibu membeli: buku, pensil, dan tinta, baju, celana, dan kaus, pisang, apel dan jeruk.
B. Ibu membeli buku, pensil, dan tinta: baju, celana, dan kaus: pisang, apel dan jeruk.
C. Ibu membeli ,buku, pensil, dan tinta, baju, celana, dan kaus, pisang, apel, dan jeruk.
D. Ibu membeli buku, pensil, dan tinta; baju, celana, dan kaus; pisang, apel, dan jeruk.

49. Bacalah kalimat berikut!


Pemerintah mengakui kebijakan pemberlakuan pembatasan kegiatan masyarakat
(PPKM) di jawa-bali yang digelar sejak 11 hingga 25 Januari diperpanjang hingga 8
Februari tidak efektif menekan laju penyebaran Covid-19.

Penggunaan ejaan pada kalimat tersebut salah karena ….


A. singkatan menggunakan huruf kapital
B. nama geografi tidak menggunakan huruf kapital
C. awal nama bulan menggunakan huruf kapital
D. awal akronim menggunakan huruf kapital

50. Bacalah kalimat berikut!


Atik mempunyai banyak teman, karena baik hati.

Alasan kesalahan penggunaan tanda baca koma (,) pada kalimat tersebut karena ….
A. hanya digunakan untuk kalimat kompleks
B. tidak digunakan di tengah kalimat majemuk
C. tidak dipakai jika induk kalimat mendahului anak kalimat
D. hanya digunakan apabila anak kalimat mendahului induk kalimat

TPM ASPD Paket. A Bahasa Indonesia 21


Dinas Pendidikan Kota Yogyakarta
www.giriwidodo.com
DOKUMEN TPM ASPD PAKET. B
SANGAT RAHASIA

TES PENDALAMAN MATERI


ASESMEN STANDAR PENDIDIKAN DAERAH
TAHAP 1
SMP/MTS KOTA YOGYAKARTA
TAHUN PELAJARAN 2020/2021

Mata Pelajaran : BAHASA INDONESIA


Hari/Tanggal : SENIN/22 FEBRUARI 2021
Waktu : 07.30 – 09.30

PEMERINTAH KOTA YOGYAKARTA


DINAS PENDIDIKAN PEMUDA DAN OLAHRAGA
2021

www.giriwidodo.com
1. Bacalah kutipan teks berikut!
Museum Gunung Merapi merupakan salah satu museum yang banyak dikunjungi di
Daerah Istimewa Yogyakarta. Letaknya di Pakem, Sleman. Museum ini didirikan pada
tahun 2005. Tujuan didirikannya museum ini untuk mengembangkan pemahaman
umum tentang Gunung Merapi, edukasi, dan konservasi bagi pengunjung. Selain itu,
untuk mengembangkan pemahaman tentang bencana alam, seperti gempa bumi dan
tsunami. Museum Gunung Merapi memiliki koleksi peranti untuk memantau aktivitas
Gunung Merapi, seperti teleskop, seismograf, dan lain-lain.

Makna kata peranti pada kutipan teks tersebut adalah ….


A. alat
B. dasar
C. bahan
D. tujuan

2. Bacalah kutipan teks berikut!


Andy Murray akhirnya mundur dari Australia Open setelah gagal bernegosiasi masalah
karantina mandiri Covid-19. Keputusan ini diambil karena peraih tiga gelar Grand Slam
itu sudah tidak memiliki waktu untuk menjalani karantina 14 hari, seperti yang
diterapkan pemerintah Australia. Sebagaimana diketahui, aturan ketat harus diikuti oleh
para petenis yang ingin tampil di Asutralia Open 2021.

Maksud kalimat bercetak miring pada kutipan teks tersebut adalah …


A. Andy Murray tidak dapat menolak karantina mandiri Covid-19 sehingga harus mundur
dari Australia Open.
B. Andy Murray berusaha menawar masalah karantina mandiri Covid-19 yang diterapkan
pada Australia Open.
C. Setelah gagal tawar-menawar masalah karantina mandiri Covid-19, Andy Murray
mundur dari Australia Open.
D. Karantina mandiri Covid-19 adalah hal yang tidak dapat ditawar oleh para petenis
yang ingin tampil di Australia Open.

TPM ASPD
Paket. B Bahasa Indonesia 2
Dinas Pendidikan Kota Yogyakarta

www.giriwidodo.com
3. Bacalah kutipan teks berikut!
Kemacetan lalu lintas menjadi fenomena umum yang terjadi di Indonesia. Kemacetan
terjadi karena jumlah kendaraan melebihi kapasitas jalan. Pengendara tidak mematuhi
Alat Pemberi Isyarat Lalu Lintas (APILL) juga menjadi penyebab kemacetan. Selain itu,
adanya pedagang kaki lima di trotoar dapat mengurangi kelancaran berkendara.
Masalah lain yang menyebabkan kemacetan yaitu adanya perbaikan jalan.

Pernyataan yang sesuai isi kutipan teks tersebut adalah …


A. Kemacetan lalu lintas di Indonesia menjadi fenomena umum karena banyak ditemui
di kota-kota besar.
B. Kemacetan terjadi karena jumlah kendaraan melebihi kapasitas jalan dan adanya
pembangunan jalan.
C. Kemacetan yang terjadi di Indonesia disebabkan oleh banyak faktor sehingga
pemerintah perlu mengupayakan solusi secepatnya.
D. Jumlah kendaraan melebihi kapasitas jalan, pengendara tidak mematuhi APILL,
adanya pedagang kaki lima di trotoar, dan adanya perbaikan jalan menjadi penyebab
kemacetan.

4. Bacalah teks berikut!


(1) Kambing dan domba adalah dua hewan yang berbeda. (2) Karakter dagingnya pun
tidak sama. (3) Secara umum, daging domba mempunyai tekstur lebih empuk dan rasa
lebih ringan daripada daging kambing. (4) Namun, banyak orang masih keliru
menganggap kedua daging ini sama.

Kalimat yang menyatakan perbandingan terdapat pada ….


A. (1)
B. (2)
C. (3)
D. (4)

5. Bacalah kutipan teks berikut!


Mekanisme kerja tubuh manusia seperti mekanisme mesin kendaraan bermotor. Tubuh
manusia dapat mengubah energi kimiawi yang terkandung dalam bahan-bahan
makanan yang masuk ke tubuh menjadi energi panas dan energi mekanis. Nasi yang
dimakan akan dibakar dalam tubuh sebagimana bensin dibakar dalam silinder mesin
mobil. Sebagian dari energi kimiawi yang terdapat dalam nasi diubah menjadi energi
panas yang membuat tubuh tetap hangat. Sebagian lagi diubah menjadi energi mekanis
yang memungkinkan otot-otot memompa darah.

Ide pokok paragraf tersebut adalah ….


A. proses perubahan energi kimiawi menjadi energi panas dalam tubuh manusia
B. persamaan mekanisme kerja tubuh manusia dan mesin kendaraan bermotor
C. nasi dalam tubuh bisa diubah menjadi energi mekanis dan energi panas
D. mekanisme kinerja mesin dan manusia

TPM ASPD
Paket. B Bahasa Indonesia 3
Dinas Pendidikan Kota Yogyakarta

www.giriwidodo.com
6. Bacalah teks berikut!
Kebersihan lingkungan sangat diperlukan dan wajib bagi lingkungan. Jika lingkungan
kita bersih, kita akan merasa nyaman, tenang, dan konsentrasi dalam melakukan
berbagai hal. Seseorang yang berada dalam keadaan tersebut akan mencapai hasil kerja
yang maksimal. Seseorang yang berkonsentrasi dalam beraktivitas akan menghasilkan
sesuatu yang lebih baik dibandingkan dengan yang tidak berkonsentrasi.
Lingkungan yang tidak terjaga akan menimbulkan berbagai penyakit yang
membahayakan. Penyakit yang mungkin terjadi karena kita tidak menjaga kebersihan
lingkungan, misalnya tipes, demam berdarah, cacingan, sakit kulit, malaria, dan pes.
Penyakit tersebut dapat menyebabkan kematian. Oleh karena itu, kesadaran tentang
kebersihan lingkungan sangat diperlukan.

Simpulan isi teks tersebut yang tepat adalah ...


A. Pentingnya menjaga kebersihan lingkungan agar kita tetap sehat.
B. Kebersihan lingkungan memengaruhi konsentrasi dalam beraktivitas.
C. Lingkungan yang tidak bersih dapat menimbulkan penyakit yang mematikan.
D. Seseorang yang berada dalam keadaan sehat dapat berkonsentrasi secara maksimal.

7. Bacalah teks berikut!


Pola komunikasi pemerintah selama menangani Covid-19 terus menjadi sorotan.
Khususnya, pernyatan-pernyataan dari satgas Covid-19 mengenai masyarakat yang
tidak mematuhi protokol kesehatan. Pernyataan tersebut dianggap menyalahkan dan
menyudutkan masyarakat di tengah pandemi.
Sebenarnya, pola komunikasi semacam itu sah saja dilakukan. Ada berbagai macam
pola komunikasi publik, di antaranya scare communication style yang bertujuan untuk
menakut-nakuti. Pola semacam ini memang memancing emosi. Hal itu bertujuan agar
masyarakat lebih mematuhi protokol kesehatan.
Namun, ada juga pendapat yang mengatakan bahwa pola komunikasi yang
menakuti-nakuti atau menyalahkan masyarakat tidak efektif. Ujaran yang memancing
emosi hanya memunculkan tanggapan negatif dari masyarakat. Alasannya, tidak ada
pihak yang tidak dirugikan karena Covid-19. Masyarakat juga sudah berjuang
semampunya untuk bertahan hidup. Seharusnya semua pihak saling mendukung dan
menguatkan.

Simpulan pendapat pro berdasarkan teks tersebut adalah …


A. Ujaran memancing emosi yang hanya memunculkan tanggapan negatif di masyarakat
harus segera diubah.
B. Pola komunikasi satgas Covid-19 menjadi sorotan karena dianggap menyudutkan
masyarakat di masa pandemi.
C. Pola komunikasi menakut-nakuti boleh dilakukan agar masyarakat lebih
memperhatikan dan mematuhi protokol kesehatan.
D. Cara berkomunikasi menyalahkan masyarakat dianggap tidak efektif karena
masyarakat mengalami banyak kerugian akibat pandemi Covid-19.

TPM ASPD
Paket. B Bahasa Indonesia 4
Dinas Pendidikan Kota Yogyakarta

www.giriwidodo.com
8. Bacalah teks berikut!
(1) Pada zaman sekarang, kekuatan teknologi tidak bisa dibendung. (2) Banyak hal baru
yang dapat kita jumpai pada zaman maju ini, salah satunya adalah gim atau permainan
virtual. (3) Gim atau permainan virtual bisa dijadikan sarana untuk menghibur diri dan
mengistirahatkan pikiran dari rutinitas yang dilakukan sebagian remaja tiap harinya. (4)
Namun, sebagian remaja yang lain justru menjadi malas untuk belajar dan lebih senang
menghabiskan waktu untuk bermain gim setiap saat.

Pendapat kontra ditandai dengan nomor ….


A. (1)
B. (2)
C. (3)
D. (4)

9. Bacalah teks berikut!


Hutan Mangrove Wana Tirta
merupakan salah satu kawasan
konservasi alam untuk pelestarian hutan
mangrove di Kulon Progo. Hutan
mangrove ini terletak di ujung barat daya
Kulon Progo dan dekat dengan
perbatasan Yogyakarta-Jawa Tengah,
tepatnya di Desa Jangkaran, Kecamatan
Temon, Kulon Progo. Kawasan hutan
mangrove memiliki beberapa jenis
tanaman yang bertujuan untuk
melindungi wilayah pesisir dari abrasi.
Hutan Mangrove Wana Tirta kini dikembangkan menjadi kawasan wisata edukasi.
Hutan mangrove ini dibuka untuk umum sehingga dapat mengenalkan serta menambah
pengetahuan para pengunjung tentang pentingnya tanaman mangrove bagi alam dan
lingkungan. Sebagai kawasan yang dikembangkan untuk wisata edukasi, kawasan Hutan
Mangrove Wana Tirta juga dibuat dengan penampilan yang menarik. Salah satunya
membagi dua kawasan berbeda dengan memanfaatkan lokasi dan jenis mangrove yang
ada.

Ringkasan teks tersebut adalah …


A. Hutan Mangrove Wana Tirta dikembangkan sebagai kawasan wisata edukasi yang
dibuat dengan penampilan menarik dan dibuka untuk umum.
B. Hutan Mangrove Wana Tirta sebagai salah satu kawasan konservasi alam di daerah
Kulon Progo yang dijadikan kawasan wisata edukasi.
C. Hutan Mangrove Wana Tirta merupakan salah satu kawasan konservasi alam untuk
pelestarian hutan mangrove di daerah Kulon Progo.
D. Hutan Mangrove Wana Tirta yang berada di daerah Kulon Progo dijadikan kawasan
wisata edukasi yang berbeda.

TPM ASPD
Paket. B Bahasa Indonesia 5
Dinas Pendidikan Kota Yogyakarta

www.giriwidodo.com
10. Bacalah kedua teks berikut!
Teks 1 Teks 2
Pendidikan merupakan senjata ampuh Prof. Dr. Ir. H. Musliar Kasim (Wakil
untuk mengubah dunia karena merupakan Kemendikbud) beranggapan bahwa
sarana bagi kita untuk mempelajari banyak kurikulum 2013 lebih menonjolkan praktik
hal yang bermanfaat. Tingkat pendidikan daripada hafalan. Berdasarkan riset,
di Indonesia tergolong baik. Pemerintah selama ini peserta didik banyak dibebani
telah mengeluarkan dana yang tidak hafalan sehingga dirasa kurang efektif
sedikit demi membangun sarana dan untuk meningkatkan kreativitas.
prasarana sekolah dan memberikan Prof. Dr. Meutia Farida Hatta Swasono,
sekolah gratis. Sekarang tinggal peserta Anggota Dewan Pertimbangan Presiden,
didik yang harus membangkitkan menuturkan bahwa kurikulum 2013 ini
keinginan belajar, berprestasi, dan bertujuan untuk membentuk karakter
bersemangat memberikan yang terbaik generasi yang unggul dan memiliki jiwa
untuk bangsa dan negara. nasionalisme yang tinggi.

Perbedaan penggunaan bahasa kedua teks tersebut adalah ....


Teks 1 Teks 2
A kata populer kata kajian
B istilah daerah istilah asing
C kata kajian kata populer
D bahasa resmi bahasa sehari-hari

11. Bacalah kedua teks berikut!


Teks 1 Teks 2
Pisang merupakan buah yang menjadi Salah satu manfaat pisang yaitu bisa
favorit semua orang untuk dikonsumsi. menangkal radikal bebas. Buah ini bisa
Rasanya yang manis membuat semua langsung dikonsumsi atau diolah menjadi
orang menyukainya. Buah ini bertekstur berbagai macam makanan seperti keripik,
lembut dan baik dikonsumsi bagi mereka kue, atau jus. Manfaat yang terkandung di
yang bermasalah dengan pencernaan. dalam pisang beragam, seperti vitamin dan
Warna buah ini bervariasi dan menarik. mineral. Buah pisang memiliki banyak
manfaat bagi manusia.

Perbedaan pola penyajian kedua teks tersebut adalah …


Teks 1 Teks 2
A urutan waktu urutan tempat
B umum-khusus khusus-umum
C perbandingan pertentangan
D akibat-sebab sebab-akibat

TPM ASPD
Paket. B Bahasa Indonesia 6
Dinas Pendidikan Kota Yogyakarta

www.giriwidodo.com
12. Bacalah teks ulasan berikut!
Novel Karang Setan
Karang Setan adalah novel ke-19 dari seri petualangan Lima Sekawan karya Enid
Blyton. Novel ini menceritakan kisah petualangan Lima Sekawan yang berlibur di sebuah
mercusuar tua milik si Utik yang berada di atas Karang Setan. Ada dongeng yang
menceritakan bahwa terdapat harta karun yang disembunyikan di sana. Lima Sekawan
pun memutuskan untuk menyelidiki sekeliling gua. Lima Sekawan itu adalah Julian, Dick,
George, Anne, dan seekor anjing bernama Timmy. Petualangan mereka kali ini semakin
seru dengan ditemani oleh si Utik dan monyetnya, si Iseng.
Pada bab pertama buku ini, Enid Blyton menceritakan bahwa teman Pak Quentin,
ayah George, akan berkunjung ke Pondok Kirin untuk menyelesaikan pekerjaan. Kedua
sarjana itu orang yang senang kesunyian dalam bekerja. Padahal, di saat yang
bersamaan Lima Sekawan juga akan datang untuk menghabiskan sisa liburan. Tentu
Pondok Kirrin akan sangat ramai oleh anak-anak, ditambah teman Pak Quentin
membawa serta anaknya yang bernama Utik dan juga seekor monyetnya bernama
Iseng.
Untuk ketenteraman bekerja kedua sarjana itu, akhirnya diputuskan Lima Sekawan
ditambah Utik dan Iseng pergi berlibur ke mercusuar milik si Utik. Mercusuar itu dulu
dipakai oleh pencoleng ulung untuk menyesatkan kapal-kapal. Dua keturunannya
sampai sekarang masih mencari tempat persembunyian harta rampasan pencoleng.
Mereka merasa terganggu oleh kedatangan Lima Sekawan dan si Utik. Akibatnya,
mereka dikurung di lonceng mercusuar.
Pada bab-bab berikutnya, pembaca akan dibuat kagum dengan berbagai
petualangan dan tantangan yang dihadapi Lima Sekawan dan Utik di Karang Setan.
Pada setiap bab, mulai awal hingga akhir, buku ini memiliki hubungan yang erat dan
merupakan kesatuan utuh yang saling melengkapi.
Novel disajikan dengan bahasa yang sangat apik dan menarik ini membuat pembaca
seakan-akan ikut berpetualang bersama Lima Sekawan ke Karang Setan. Kekonyolan
Iseng dan Timmy membuat cerita yang ada dalam buku ini semakin seru. Hanya satu
kekurangan buku ini, yaitu penyelesaian yang kurang mengesankan.

Keunggulan buku yang terdapat pada teks tersebut adalah …


A. Setiap seri petualangan Lima Sekawan memiliki cerita dan tempat yang berbeda.
B. Buku ini memberikan gambaran tentang sebuah petualangan dan rintangan.
C. Novel tersebut memiliki keistimewaan tokoh, yaitu Lima Sekawan.
D. Novel disajikan dengan bahasa yang apik dan menarik.

TPM ASPD
Paket. B Bahasa Indonesia 7
Dinas Pendidikan Kota Yogyakarta

www.giriwidodo.com
13. Bacalah teks berikut!
Pemerintah daerah dan orang tua perlu berpikir matang untuk memberi izin sekolah
tatap muka pada anak berkebutuhan khusus di masa pandemi. Sekolah Luar Biasa (SLB)
rentan menjadi klaster baru karena proses pembelajarannya menuntut banyak sentuhan
fisik. Klaster SLB muncul di beberapa daerah seperti Tulungagung, Lumajang,
Kalimantan Barat, Tegal, Cilegon, Sumedang, Pati, Balikpapan, dan Rembang.
Pembelajaran tatap muka untuk SLB perlu ditinjau ulang, mengingat pembelajaran anak
berkebutuhan khusus tidak dapat disamakan dengan sekolah biasa.

Komentar yang tepat berdasarkan teks tersebut adalah ...


A. Sebaiknya pemerintah daerah tidak memberlakukan sistem pembelajaran tatap muka
di Sekolah Luar Biasa (SLB) karena anak berkebutuhan khusus rentan menjadi klaster
baru.
B. Semua sekolah di Tulungagung, Lumajang, Kalimantan Barat, Tegal, Cilegon,
Sumedang, Pati, Balikpapan, dan Rembang tidak menerapkan protokol kesehatan
dengan ketat.
C. Pembelajaran anak berkebutuhan khusus tidak sama dengan sekolah biasa karena
proses pembelajaran di Sekolah Luar Biasa (SLB) menuntut banyak sentuhan fisik.
D. Sekolah Luar Biasa (SLB) rentan menjadi klaster baru sehingga proses belajar harus
menggunakan sistem pembelajaran jarak jauh.

14. Bacalah kutipan cerpen berikut!


Bencana Covid-19 membuat sebagian keluarga mengalami keterpurukan ekonomi,
termasuk keluarga Sandi, teman sekelas Badu.
“Teman-teman, untuk membantu Sandi, kita akan menggalang dana,” kata Hadi,
ketua kelas 9A saat live conference siswa kelas 9A bersama wali kelas. Teman-teman
setuju, kecuali Badu.
“Memangnya kenapa juga kita harus bantu Sandi, mereka hanya kurang kreatif saja
mencari peluang di masa pandemi,” ujar Badu.
Semua teman bergidik mendengar ucapan Badu dan mereka menganggap Badu
anak yang kikir.
“Enak saja, minta-minta, bisa habis uangku,” kata Badu dalam hati.

Makna kata kikir pada kutipan cerpen tersebut adalah …


A. egois
B. pelit
C. hemat
D. tak acuh

TPM ASPD
Paket. B Bahasa Indonesia 8
Dinas Pendidikan Kota Yogyakarta

www.giriwidodo.com
15. Bacalah kutipan fabel berikut!
Kiki Beo berusaha mengibaskan sayap kecilnya. Sekuat tenaga ia berusaha agar
bisa terbang. Akan tetapi, lagi-lagi dia jatuh. Beberapa ekor tupai yang sedang makan
kenari tertawa melihatnya. Kiki hanya terduduk lesu hampir putus asa.
“Ayo, Kiki coba lagi! Lama-lama kamu pasti bisa,” tupai-tupai itu memberinya
semangat. Dengan senyum kecilnya, Kiki mengangguk dan mencoba lagi.

Makna tersurat dari kutipan fabel tersebut adalah …


A. Kiki Beo berlatih terbang, tetapi putus asa ketika ditertawakan para tupai.
B. Jangan pernah putus asa ketika kamu gagal dalam berusaha, seperti Kiki Beo.
C. Tak ada usaha yang sis-sia. Segera bangkitlah kembali apabila kamu gagal dalam
berusaha.
D. Kiki hampir putus asa saat gagal berlatih terbang, tetapi para tupai memberinya
semangat.

16. Perhatikan kutipan fabel berikut!


Miki, si anjing besar berbulu hitam menatap garang seekor kucing bernama Meong.
Dengan sombongnya, Miki mengejek Meong karena bulunya jelek, kelihatan kotor, dan
kecil postur tubuhnya. Meong gemetar dan ketakutan hingga tak berani menatap Miki.
Meong sangat sedih dan berusaha menjauh. Ia berjalan sambil meneteskan air mata.
Hingga beberapa hari ia tidak berani melewati gang itu.
”Mengapa kau menangis?” tanya Gufi, sahabat Meong.
”Miki telah mengejekku, aku juga dibentak-bentak, bahkan aku hampir dipukul,”
kata Meong sesenggukan.

Kutipan fabel tersebut merupakan bagian ....


A. orientasi
B. komplikasi
C. resolusi
D. koda

17. Bacalah kutipan cerpen berikut!


“Dunia ini penuh sandiwara!” gerutu Lia menyesali kebodohannya yang begitu
saja percaya kepada teman-temannya.
Lia menghela napas berusaha menenangkan diri. Lia yakin pertolongan Tuhan pasti
akan datang. Lia hanya memohon kepada Tuhan agar dibukakan pintu hati mereka. Lia
ingin kebenaran segera terungkap.

Makna simbol dunia ini penuh sandiwara melambangkan tingkah laku manusia yang
….
A. seperti selebritis
B. suka bersandiwara
C. penuh kepura-puraan
D. tidak dapat dipercaya

TPM ASPD
Paket. B Bahasa Indonesia 9
Dinas Pendidikan Kota Yogyakarta

www.giriwidodo.com
18. Bacalah kutipan cerpen berikut!
Pak Tarji berjualan bakso keliling di kampungku sejak aku masih kecil. Setiap sore,
aku selalu menunggu gerobaknya lewat depan rumah. Begitu pula tetanggaku, selalu
setia menunggu gerobaknya lewat. Ketika aku duduk di kelas tujuh, dia pernah
menyewa kios kecil pinggir jalan di depan gang masuk kampungku. Pak Tarji berjualan
di kios tersebut dibantu istrinya. Baksonya sangat laris, banyak pembeli dari kampung
lain berdatangan membeli bakso tersebut.
Dua tahun setelah Pak Tarji berjualan bakso di kios, istrinya jatuh sakit. Saat
melayani pembeli, tiba-tiba dia pingsan. Beberapa pembeli langsung berlari
menolongnya. Pada saat itu, Pak Tarji sedang pulang ke rumah mengambil mangkuk
dan sendok. Sesampainya di kios, istrinya belum sadar. Pak Tarji segera membawa
istrinya ke rumah sakit yang tidak jauh dari kios. Ternyata, Bu Tarji mengalami
serangan stroke dan harus dirawat selama dua minggu di rumah sakit. Kehidupan Pak
Tarji berubah sejak istrinya sakit. Dia harus berjualan bakso sambil merawat istrinya
yang lumpuh dan duduk di kursi roda. Siang ini, aku mampir ke kios bakso Pak Tarji.
Aku melihat wajah Pak Tarji begitu lelah. Namun, kedua matanya seolah
mengungkapkan betapa dia bahagia bisa selalu menjaga dan merawat istrinya.

Pesan moral yang terdapat dalam kutipan cerpen tersebut adalah ...
A. Senantiasa sabar dalam menghadapi cobaan hidup.
B. Keputusasaan membuat hidup seseorang menjadi hancur.
C. Setiap kejadian yang dialami seseorang patut dijadikan kenangan.
D. Kesuksesan dapat diraih dengan bantuan dan dukungan banyak pihak.

TPM ASPD
Paket. B Bahasa Indonesia 10
Dinas Pendidikan Kota Yogyakarta

www.giriwidodo.com
19. Bacalah kutipan fabel berikut!
“Kok melamun?” tanya Lalang, si belalang kayu.
“Aku memikirkan penghuni hutan ini,” jawab si Mon, monyet yang nakal sambil
memakan pucuk-pucuk daun.
“Buat apa memikirkan itu, Mon, bukankah sudah ada sang Raja yang memikirkan
kita?” tanya Lalang.
“Apa kamu sudah makan hari ini?” tanya si Mon.
“Aku belum makan, perutku belum terisi apa-apa sejak kemarin,” jawab Lalang
“Itulah, Lang. Sesore ini kita sudah berkeliling hutan mencari makan, tetapi belum
menemukan. Yang bisa kita makan hanya dedaunan. Tak ada buah-buahan sama
sekali,” kata si Mon sedih.
Tiba-tiba pandangan si Mon tertuju ke batas hutan. Ia melihat kawanannya
memasuki pemukiman. Si Mon segera turun dari pohon dan berlari.
“Jangan! Jangan pergi ke pemukiman! Bahaya!” teriak si Mon sambil berlari sekuat
tenaga.
Namun, semua sudah terlambat, kawanan monyet telah menyerbu pemukiman
untuk mencari makanan. Kawanan monyet itu mengamuk dan mencuri makanan
penduduk.

Penyebab konflik kutipan fabel tersebut yang paling tepat adalah ....
A. kekhawatiran si Mon terhadap kawanan monyet di hutan
B. kawanan monyet menyerbu pemukiman penduduk
C. Lalang dan si Mon mencari makanan sampai sore
D. hewan di hutan kesulitan mencari makanan

20. Bacalah dua kutipan cerpen berikut!


Kutipan Cerpen 1 Kutipan Cerpen 2
Aku hanya berpikir apa yang akan Bau tanah basah dan daun gugur sisa
kukatakan pada ibu tentang rencanaku hujan semalam mengajakku berlari
masuk kedokteran di universitas ternama menyusuri kenangan puluhan tahun silam.
gagal total. Memang aku yang telah Di bawah pohon angsana yang
bersikeras untuk melaksanakan rencana ini menggugurkan bunga mungil berwarna
meskipun ibu tidak merestui. Pecah kuning terang layaknya percik mentari
kepalaku memikirkan permasalahan ini. pagi, aku menunggu ayah. Ayah setia
Aku juga harus menanggung malu yang dengan motor tua yang akan
tak tanggung-tanggung. Belum lagi mengantarku sekolah. Tergambar senyum
banyaknya biaya yang sudah dikeluarkan hangat ayah, peluk sayang ibu membuat
ibuku selama mengikuti tes semakin panas mataku.
membebani pikiranku.

Pola pengembangan kutipan cerpen tersebut diawali dengan ….


Cerpen 1 Cerpen 2
A komplikasi resolusi
B orientasi komplikasi
C komplikasi orientasi
D resolusi orientasi

TPM ASPD
Paket. B Bahasa Indonesia 11
Dinas Pendidikan Kota Yogyakarta

www.giriwidodo.com
21. Bacalah kedua kutipan fabel berikut!
Teks 1 Teks 2
Semua penghuni laut tidak suka pada Ayam dan itik bersahabat karib.
Sisi, si singa laut, karena Mereka hidup berdampingan dan tolong-
kesombongannya. Tidak ada yang mau menolong.
berteman dengannya kecuali Jelly, si Ketika burung sedang bertengger di
ubur-ubur. dahan, dia melihat ayam dan itik sedang
“Jelly, kenapa kamu mau berteman mencari makan. Melihat hal itu, muncullah
dengan Sisi?” tanya Pino, si lumba-lumba. kesombongan burung. Dengan gagahnya,
“Kasihan dia tidak punya teman,” dia menertawakan ayam yang mengais
jawab Jelly. tanah dan itik yang mencari makanan
“Tapi bukankah kamu juga sering dengan menyosor.
dihina sama Sisi?” tanya Srimpi, si udang. Kedua binatang itu tidak menggubris
“Iya sih Srimpi, tapi tenang aja aku ejekan burung. Mereka tetap asyik
sudah punya rencana agar Sisi sadar. mencari makanan.
Kalian mau nggak membantuku
menyadarkannya?” tanya Jelly antusias.

Perbedaaan penggunaan bahasa kedua kutipan fabel tersebut adalah ….


Teks 1 Teks 2
A makna kias makna lugas
B dialog narasi
C bahasa baku bahasa sehari-hari
D tidak ada kata sandang kata sandang

22. Bacalah kutipan fabel berikut!


(1) Don adalah badak jawa bercula satu, kulitnya abu-abu dengan sedikit bulu,
memiliki penciuman dan pendengaran sangat tajam. (2) Suatu ketika, Don duduk
termenung karena kubangan yang biasa ia gunakan sudah mengering. (3) Ken, si
burung jalak sahabatnya datang menghibur.
(4) “Sabarlah, Don masih beruntung kita dapat menikmati rumput di sela pohon-
pohon besar ini,” kata Ken.

Bukti latar tempat pada kutipan fabel tersebut terdapat pada kalimat ....
A. (1)
B. (2)
C. (3)
D. (4)

TPM ASPD
Paket. B Bahasa Indonesia 12
Dinas Pendidikan Kota Yogyakarta

www.giriwidodo.com
23. Bacalah kutipan cerpen berikut!
(1) Dodi membereskan meja makan. Dia ingin meninggalkan rumah dalam keadaan
bersih dan rapi. Itu pula yang diajarkan oleh ibunya.
(2) “Kakak!” jerit Putri dari dapur. Suaranya terdengar panik. Bergegas Dodi
menghampirinya. Dilihatnya Putri menggenggam telunjuknya. Di sela-sela jemarinya
cairan merah menetes.
(3) Secepat kilat, Dodi mengambil beberapa benda dari kotak P3K di ruang tengah.
“Sini, Dik, Kakak obati!” kata Dodi. Putri menjulurkan jarinya ragu. Tidak berapa lama
luka itu sudah bersih dan tertutup rapat kain perban. Dodi tersenyum lega melihat
Putri sudah kembali tenang.
(4) “Ibu, seperti pesanmu sebelum engkau pergi agar mengurus Putri. Sudah
kulaksanakan, Bu. Semoga engkau tenang karenanya,” bisiknya dalam hati. Mata Adi
menerawang jauh. Hatinya terbalut rindu yang tak bertepi.

Bukti watak Dodi cekatan terdapat pada pernyataan ....


A. (1)
B. (2)
C. (3)
D. (4)

24. Bacalah kutipan cerpen berikut!


Para pelancong mengunjungi kota kami untuk menyaksikan kepedihan. Mereka
datang untuk menonton kota kami yang hancur. Kemunculan para pelancong itu
membuat kesibukan tersendiri di kota kami. Biasanya kami duduk-duduk di gerbang
kota memandangi para pelancong yang selalu muncul berombongan mengendarai
kuda, keledai, unta, permadani terbang, dan kuda sembrani. Mereka datang dari segala
penjuru dunia. Dari negeri-negeri jauh yang gemerlapan.
Penampilan para pelancong yang selalu riang membuat kami merasa terhibur.
Kami menduga, para pelancong itu sepertinya telah bosan dengan hidup mereka yang
sudah terlampau bahagia. Hidup yang selalu dipenuhi kebahagiaan ternyata bisa
membosankan juga. Mungkin para pelancong itu tak tahu lagi bagaimana caranya
menikmati hidup yang nyaman tenteram tanpa kecemasan di tempat asal mereka.
Karena itulah, mereka ramai-ramai piknik ke kota kami menyaksikan bagaimana
perlahan-lahan kota kami menjadi debu. Kami menyukai cara mereka tertawa saat
mereka begitu gembira membangun tenda-tenda dan mengeluarkan perbekalan.
Mereka berfoto di antara reruntuhan puing kota kami. Kami seperti menyaksikan
rombongan sirkus yang datang untuk menghibur.

Komentar yang tepat terhadap kutipan cerpen tersebut adalah …


A. Sudut pandang orang pertama menggunakan kata kami menambah keunikan cerita.
B. Latar tempat sebaiknya tidak disebutkan secara langsung agar cerita menjadi jelas.
C. Penggambaran watak tokoh sangat jelas sehingga sangat menarik.
D. Alurnya berbelit-belit sehingga sulit dipahami pembaca.

TPM ASPD
Paket. B Bahasa Indonesia 13
Dinas Pendidikan Kota Yogyakarta

www.giriwidodo.com
25. Bacalah kalimat berikut!
Ketela pohon merupakan sumber energi yang kaya […], tetapi sangat minim […].

Istilah yang tepat untuk melengkapi kalimat tersebut adalah …


A. lemak, protein
B. protein, energi
C. karbohidrat, protein
D. lemak, karbohidrat

26. Bacalah teks berikut!


Cara Membuat Hand Sanitizer Spray
(1) Masukkan 2 tetes essential oil agar cairan hand sanitizer wangi.
(2) Masukkan alkohol 70% sebanyak 60 ml dan 70 gram aloe vera ke dalam gelas
takar.
(3) Aduk kedua bahan tersebut selama 15 hingga 20 menit hingga tercampur dengan
baik.
(4) Tuang ke dalam botol spray lalu kocok perlahan agar tercampur dengan baik.
(5) Hand Sanitizer sudah siap digunakan.

Urutan kalimat yang tepat dalam penulisan teks prosedur tersebut adalah ... .
A. (1)-(2)-(3)-(4)-(5)
B. (1)-(3)-(4)-(2)-(5)
C. (2)-(1)-(3)-(4)-(5)
D. (2)-(3)-(1)-(4)-(5)

27. Bacalah paragraf berikut!


Wedang uwuh adalah minuman tradisional dari Imogiri, Bantul yang berkhasiat untuk
menghangatkan badan dan mencegah berbagai penyakit ringan. Selain khasiatnya
yang baik untuk tubuh, wedang uwuh juga memiliki rasa yang nikmat dan khas
sehingga disukai oleh masyarakat. Tampilannya sedikit berantakan karena berisi
ramuan jamu menjadi ciri khas minuman tradisional ini. […] Campuran rempah yang
digunakan untuk bahan membuat wedang uwuh antara lain jahe, cengkeh, serutan
kayu manis, pala, serai, kapulaga, dan daun pala. Lebih nikmat lagi apabila
ditambahkan gula batu. Gula batu dipilih karena memberikan rasa manis, tetapi tidak
mengubah rasa dan aroma asli pada minumannya.

Kalimat yang tepat untuk melengkapi paragraf tersebut adalah ...


A. Rasa wedang uwuh sangat unik dengan perpaduan manis dan sedikit pedas
dari jahenya memberikan cita rasa yang khas pada minuman ini.
B. Wedang uwuh berisi campuran rempah yang sekilas terlihat seperti sampah
dedaunan yang dimasukan ke dalam gelas dan diseduh langsung.
C. Proses penyajiannya cukup sederhana dengan bahan-bahan sesuai porsi kemudian
diseduh dengan air panas dan aduk hingga warnanya merah.
D. Untuk menikmatinya, wedang uwuh diaduk dulu sampai gula batunya mencair dan
menghasilkan manis seperti yang kita inginkan.

TPM ASPD
Paket. B Bahasa Indonesia 14
Dinas Pendidikan Kota Yogyakarta

www.giriwidodo.com
28. Bacalah teks ulasan berikut!
Judul buku: Cinta Oranye
Penulis: Lesta M. Nusa
Penerbit: Bola Persada Media
Tahun terbit: 2020
Buku ini mengisahkan perjalanan hidup seorang penulis yang masih muda. Di usia
17 tahun ia sudah mampu menulis puluhan buku bertaraf internasional. Ia juga sudah
mendapatkan berbagai penghargaan dari dalam dan luar negeri.
Tampilan buku ini sangat menarik dengan kover yang terlihat ceria. Isi buku cukup
mengaduk emosi pembacanya dengan gabungan kisah komedi, cinta, hingga
kehidupan sehari-hari. Novel ini mengunakan bahasa anak muda yang gaul.
[...]
Dibalik segala kelebihan dan kekurangannya, buku ini mengundang minat banyak
orang karena nama besar penulisnya. Sisi humor Lesta M. Nusa dituangkan dalam
buku ini dan mampu menghibur setiap pembacanya.

Paragraf yang tepat untuk melengkapi teks tersebut ...


A. Akhir cerita mengambang sehingga tidak memuaskan pembaca. Menggunakan
banyak kata vulgar yang kadang menimbulkan rasa risih dan tidak nyaman .
B. Alurnya menantang dan banyak kejutan yang membangkitkan rasa penasaran
pembaca untuk terus mengikuti cerita. Menampilkan pergaulan anak-anak berkelas
yang penuh dengan keceriaan.
C. Lesta M. Nusa yang idealis tentu saja tidak bersedia ceritanya diacak-acak hanya
untuk kepentingan sebuah film. Hal itu akan membuat harga dirinya jatuh di mata
penggemarnya.
D. Alur cerita yang runtut memudahkan pembaca untuk memahami isi cerita. Pembaca
seperti dituntun dari satu tahap ke tahap selanjutnya sehingga nyaman mengikuti
alurnya.

29. Bacalah kalimat berikut!


Ketua OSIS memilih anggota-anggotanya yang akan bertugas menggalang dana
kemanusiaan korban bencana alam.

Variasi kata memilih pada kalimat tersebut adalah ...


A. menentukan
B. menekankan
C. menunjukkan
D. mendudukkan

TPM ASPD
Paket. B Bahasa Indonesia 15
Dinas Pendidikan Kota Yogyakarta

www.giriwidodo.com
30. Perhatikan kalimat berikut!
Vaksin Covid-19 sudah bisa digunakan oleh masyarakat dengan syarat-syarat yang
harus dipenuhi.

Variasi kalimat tersebut yang tepat adalah ...


A. Vaksin Covid-19 sudah harus dipenuhi oleh masyarakat dengan syarat-syarat
tertentu.
B. Syarat-syarat yang harus dipenuhi oleh vaksin Covid-19 sudah bisa digunakan oleh
masyarakat.
C. Masyarakat sudah bisa menggunakan vaksin Covid-19 dengan syarat-syarat yang
harus dipenuhi.
D. Vaksin Covid-19 dengan syarat-syarat yang harus dipenuhi sudah bisa digunakan
oleh masyarakat.

TPM ASPD
Paket. B Bahasa Indonesia 16
Dinas Pendidikan Kota Yogyakarta

www.giriwidodo.com
31. Perhatikan diagram berikut!

Laporan yang sesuai diagram tersebut adalah …


A. Jumlah pengunjung Taman Bacaan pada minggu pertama bulan Desember 2020
mengalami kenaikan. Pada hari Senin hanya 15 pengunjung. Pada hari Selasa
bertambah 5 pengunjung, sedangkan Rabu dan Kamis mengalami kenaikan yang
sama, yaitu 10 pengunjung. Puncak kunjungan terjadi pada hari Jumat dengan
jumlah 45 pengunjung.

B. Jumlah pengunjung Taman Bacaan pada minggu pertama bulan Desember 2020
mengalami penurunan. Pada hari Senin hanya 15 pengunjung. Pada hari Selasa
bertambah 5 pengunjung, sedangkan Rabu dan Kamis mengalami penurunan yang
sama, yaitu 10 pengunjung. Puncak kunjungan terjadi pada hari Jumat dengan
jumlah 45 pengunjung.

C. Jumlah pengunjung Taman Bacaan pada minggu pertama bulan Desember 2020
mengalami kenaikan. Pada hari Jumat hanya 15 pengunjung. Pada hari Kamis
bertambah 5 pengunjung, sedangkan Rabu dan Selasa mengalami kenaikan yang
sama, yaitu 10 pengunjung. Puncak kunjungan terjadi pada hari Senin dengan
jumlah 45 pengunjung.

D. Jumlah pengunjung Taman Bacaan pada minggu pertama bulan Desember 2020
mengalami kenaikan. Pada hari Senin hanya 20 pengunjung. Pada hari Selasa
bertambah 5 pengunjung, sedangkan Rabu dan Kamis mengalami kenaikan yang
sama, yaitu 10 pengunjung. Puncak kunjungan terjadi pada hari Jumat dengan
jumlah 45 pengunjung.

TPM ASPD
Paket. B Bahasa Indonesia 17
Dinas Pendidikan Kota Yogyakarta

www.giriwidodo.com
32. Bacalah teks wawancara berikut!
Guru BK : “Apa alasanmu ingin menjadi pengurus OSIS?”
Siswa : “Saya ingin belajar berorganisasi melalui OSIS, Bu.”
Guru BK : “Mengapa memilih OSIS sebagai sarana belajar berorganisasi?”
“Karena OSIS satu-satunya organisasi siswa yang resmi di sekolah kita,
Siswa :
Bu.“
Guru BK : “Apakah ada alasan lain?”
Siswa : “Ada, Bu. Saya ingin jiwa kepemimpinan saya tumbuh dan berkembang
dengan optimal, serta berharap kelak bisa menjadi pemimpin yang
baik.”

Narasi yang sesuai dengan teks wawancara tersebut adalah…


A. Guru BK mewawancarai calon pengurus OSIS. Beliau menanyakan alasan dan tujuan
belajar berorganisasi di OSIS. Dia mengatakan bahwa OSIS merupakan organisasi
terpopuler di sekolah. Dia berharap jiwa kepemimpinannya bisa tumbuh dan
berkembang optimal serta bisa menjadi pemimpin yang baik.

B. Guru BK mewawancarai calon pengurus OSIS. Beliau menanyakan alasan dan tujuan
belajar berorganisasi di OSIS. Dia mengatakan OSIS merupakan satu-satunya
organisasi resmi di sekolah. Dia berharap jiwa kepemimpinannya bisa tumbuh dan
berkembang optimal serta bisa menjadi pemimpin yang baik.

C. Calon pengurus OSIS diwawancarai Guru BK. Beliau menanyakan alasan dan tujuan
belajar berorganisasi di OSIS. Dia mengatakan OSIS merupakan satu-satunya
organisasi resmi di sekolah. Dia berharap jiwa kepemimpinannya bisa tumbuh dan
berkembang optimal serta bisa menjadi pejabat tinggi.

D. Pengurus OSIS diwawancarai Guru BK. Beliau menanyakan alasan dan tujuan belajar
berorganisasi di OSIS. Dia mengatakan bahwa OSIS merupakan organisasi terpopuler
di sekolah. Dia berharap jiwa kepemimpinannya bisa tumbuh dan berkembang
optimal serta bisa menjadi pemimpin yang baik.

33. Bacalah kalimat berikut!


Sesuai dengan diagnosa dokter dan hasil pemeriksaan PCR, pasien itu terinfeksi virus
Covid-19 sehingga harus dikarantina.

Kata tidak sesuai kaidah pada kalimat tersebut adalah ….


A. virus
B. terinfeksi
C. karantina
D. diagnosa

TPM ASPD
Paket. B Bahasa Indonesia 18
Dinas Pendidikan Kota Yogyakarta

www.giriwidodo.com
34. Bacalah paragraf berikut!
(1) Ali selalu serius ketika belajar, baik di rumah maupun di sekolah. (2) Ia memiliki cita-
cita yang begitu luhur. (3) Ia ingin menjadi manusia yang berguna bagi untuk sesama.
(4) Hal ini tidak mengherankan karena ayah Ali seorang pejuang kemanusiaan.

Kalimat tidak efektif terdapat pada ....


A. (1)
B. (2)
C. (3)
D. (4)

35. Bacalah kalimat berikut!


Protokol kesehatan bertujuan untuk mencegah penularan virus korona dan […]
bertambahnya angka kasus infeksi.

Kata bentukan yang tepat untuk melengkapi kalimat tersebut adalah ….


A. peminimalan
B. diminimalkan
C. terminimalkan
D. meminimalkan

36. Bacalah kalimat berikut!


Susu kedelai merupakan susu yang mengandung protein nabati […] susu sapi
mengandung protein hewani.

Konjungsi yang tepat untuk melengkapi bagian yang rumpang adalah …


A. karena
B. supaya
C. sehingga
D. sedangkan

37. Bacalah kalimat berikut!


Warga menyegel pintu gang yang menuju ke wilayah mereka selama PPKM
(pelaksanaan pembatasan kegiatan masyarakat).

Perbaikan kata menyegel dalam kalimat tersebut adalah…


A. mengunci
B. menutup
C. menyekat
D. menghadang

TPM ASPD
Paket. B Bahasa Indonesia 19
Dinas Pendidikan Kota Yogyakarta

www.giriwidodo.com
38. Bacalah kalimat berikut!
Dalam rangka memeriahkan hari kemerdekaan RI, maka banyak warga yang mengecat
pagar berwarna merah putih.

Perbaikan kalimat tersebut adalah …


A. Dalam rangka memeriahkan hari kemerdekaan RI, warga mengecat pagar merah
putih.
B. Dalam rangka memeriahkan hari kemerdekaan RI, maka warga mengecat pagar
berwarna merah putih.
C. Dalam rangka untuk memeriahkan hari kemerdekaan RI, maka banyak mengecat
pagar berwarna merah putih.
D. Dalam rangka untuk memeriahkan hari kemerdekaan RI, banyak warga yang
mengecat pagar berwarna merah putih.

39. Bacalah teks berikut!


(1) Secara umum metamorfosis katak ada empat fase yaitu fase telur, kecebong atau
berudu, katak muda, dan katak dewasa. (2) Fase telur akan berlangsung selama
kurang lebih tiga minggu. (3) Telur yang berhasil menetas akan menjadi kecebong. (4)
Fase kecebong ini akan terjadi selama lima minggu hingga kecebong tersebut menjadi
katak muda. (5) Dari katak muda menjadi katak dewasa butuh waktu tiga minggu. (6)
Katak yang sudah tumbuh dewasa dapat hidup di dua tempat, air dan darat. (7) Alat
pernapasan itulah yang digunakan oleh katak dewasa untuk bernapas di daratan.

Perbaikan kalimat nomor (6) yang tepat agar menjadi paragraf padu adalah …
A. Akibatnya, insang pada katak muda akan menghilang dan si katak akan mengalami
perubahan drastis.
B. Setelah menjadi dewasa, katak bisa hidup di dua alam sehingga katak disebut
hewan amfibi.
C. Perubahan dari katak muda menjadi katak dewasa ditandai dengan terbentuknya
paru-paru.
D. Katak dewasa memiliki paru-paru yang berfungsi sebagai alat pernapasan ketika di
darat.

40. Bacalah kalimat berikut!


Sudah dua minggu Pak Sadri didepak dari kantor tempatnya bekerja karena terbukti
melakukan kesalahan fatal.

Alasan kesalahan penggunaan kata didepak dalam kalimat tersebut karena ….


A. kata tidak baku
B. memiliki nilai rasa negatif
C. memiliki nilai rasa positif
D. bahasa daerah

TPM ASPD
Paket. B Bahasa Indonesia 20
Dinas Pendidikan Kota Yogyakarta

www.giriwidodo.com
41. Perhatikan kalimat berikut!
Hasil bumi daripada petani sebagian besar diekspor ke Eropa dan Amerika Serikat.

Alasan ketidakefektifan kalimat tersebut adalah ….


A. penggunaan kata daripada tidak diperlukan
B. kalimat tersebut tidak mempunyai subjek
C. kalimat tersebut bermakna ganda
D. kesalahan penggunaan konjungsi

42. Bacalah paragraf berikut!


Ada dua tips mengajarkan anak lancar membaca. Pertama, membaca dengan
menggunakan lagu dan sajak. Kedua, menggunakan beragam aplikasi membaca atau
membaca buku pembelajaran. Mengajarkan anak membaca perlu dilakukan melalui
aktivitas yang menyenangkan.

Alasan ketidakpaduan paragraf tersebut karena ….


A. seluruh kalimat merupakan kalimat penjelas
B. kalimat utama terdapat pada seluruh paragraf
C. seluruh kalimat penjelas tidak mendukung gagasan utama
D. terdapat kalimat penjelas yang tidak mendukung gagasan utama

43. Bacalah teks berikut!


(1) Drama Kereta Kencana benar-benar mengaduk emosi penonton. (2) Para aktor
sangat menjiwai perannya sehingga penonton puas menyaksikan pertunjukkan drama
tersebut. (3) Mereka ingin menonton lagi. (4) Bahkan, mereka tidak peduli harga
tiketnya.

Ejaan yang salah terdapat pada kalimat ….


A. (1)
B. (2)
C. (3)
D. (4)

44. Bacalah kalimat-kalimat berikut!


(1) Desi Riantari, M. Hum. terkenal karena keahliannya yang luar biasa.
(2) Ibu membeli bayam, wortel, dan tomat: pisang, apel, dan manggis: piring, gelas,
dan sendok.
(3) Saya pergi ke toko buku bersama paman sampai pukul 13.45.
(4) Pelajari materi bahasa Indonesia pada halaman 36 s.d. 40 dengan cermat!

Penggunaan tanda baca yang salah terdapat pada kalimat ...


A. (1)
B. (2)
C. (3)
D. (4)

TPM ASPD
Paket. B Bahasa Indonesia 21
Dinas Pendidikan Kota Yogyakarta

www.giriwidodo.com
45. Bacalah kalimat-kalimat berikut!
(1) Generasi muda harus berfikir kritis dan memperluas wawasan dengan
memperbanyak literasi.
(2) Intensitas hujan yang cukup tinggi dan udara lembab membuat pengrajin kulit
mengalami kerugian.
(3) Klinik kecantikan itu sudah ditutup karena melakukan praktek tidak berizin.
(4) Keadaan cuaca di Kota Yogyakarta cukup ekstrem sehingga menyebabkan banyak
pohon tumbang.

Kalimat dengan ejaan tepat terdapat pada ….


A. (1)
B. (2)
C. (3)
D. (4)

46. Bacalah kalimat-kalimat berikut!


(1) “Apa yang harus aku lakukan dalam keadaan seperti ini! Seharusnya kamu
membantuku mencari jalan keluar supaya masalah ini segera selesai.
(2) Paman berkata: “Sejak kepergian kakek, rumah menjadi sunyi dan semua
penghuninya tampak bersedih.”
(3) “Kamu harus melaksanakan tugas ini baik-baik! Ini adalah kesempatan yang
berharga untuk masa depanmu,” kata Bu Dewi.
(4) “Obat tradisional itu ternyata manjur untuk menurunkan demam. Padahal, bahan-
bahannya mudah didapatkan.” kata ibu.

Penggunaan tanda baca yang benar terdapat pada kalimat ….


A. (1)
B. (2)
C. (3)
D. (4)

47. Bacalah kalimat berikut!


Kekayaan Budaya Indonesia berupa tari tradisional yang masih dilestarikan hingga saat
ini, yaitu tari melayu.

Perbaikan penulisan ejaan pada kalimat tersebut adalah …


A. Kekayaan budaya Indonesia berupa tari tradisional yang masih dilestarikan hingga
saat ini, yaitu tari Melayu.
B. Kekayaan budaya Indonesia berupa tari tradisional yang masih dilestarikan hingga
saat ini, yaitu tari melayu.
C. Kekayaan Budaya Indonesia berupa tari tradisional yang masih dilestarikan hingga
saat ini, yaitu tari melayu.
D. Kekayaan Budaya Indonesia berupa tari tradisional yang masih dilestarikan hingga
saat ini, yaitu Tari Melayu.

TPM ASPD
Paket. B Bahasa Indonesia 22
Dinas Pendidikan Kota Yogyakarta

www.giriwidodo.com
48. Bacalah kalimat berikut ini!
Agar terhindar dari virus korona; kita perlu memakai masker; menjaga jarak; dan
sering mencuci tangan.

Perbaikan penggunan tanda baca pada kalimat tersebut adalah ....


A. Agar terhindar dari virus korona, kita perlu memakai masker, menjaga jarak, dan
sering mencuci tangan.
B. Agar terhindar dari virus korona; kita perlu memakai masker, menjaga jarak, dan
sering mencuci tangan.
C. Agar terhindar dari virus korona, kita perlu memakai masker; menjaga jarak; dan
sering mencuci tangan.
D. Agar terhindar dari virus korona: kita perlu memakai masker: menjaga jarak: dan
sering mencuci tangan.

49. Bacalah kalimat berikut!


Kepala Dinas Pariwisata Pemuda dan Olahraga kabupaten karanganyar, Titis Sri
Jawoto mengatakan bahwa tempat wisata yang tidak berisiko tinggi penularan Covid-
19 tetap buka selama PPKM diperpanjang hingga 8 Februari 2021.

Penggunaan ejaan pada kalimat tersebut salah karena…


A. huruf awal akronim ditulis kapital
B. singkatan kata ditulis huruf kapital
C. huruf awal nama bulan ditulis kapital
D. huruf awal nama geografi tidak ditulis kapital

50. Bacalah kalimat berikut!


Peserta didik harus banyak mencari informasi, agar memiliki wawasan luas.

Alasan kesalahan penggunaan tanda baca koma (,) pada kalimat tersebut karena …
A. hanya digunakan untuk kalimat kompleks.
B. tidak digunakan di tengah kalimat majemuk.
C. tidak dipakai jika induk kalimat mendahului anak kalimat.
D. digunakan jika anak kalimat mendahului induk kalimat.

TPM ASPD
Paket. B Bahasa Indonesia 23
Dinas Pendidikan Kota Yogyakarta

www.giriwidodo.com
DOKUMEN TPM ASPD PAKET. A
SANGAT RAHASIA

TES PENDALAMAN MATERI


ASESMEN STANDARISASI PENDIDIKAN DAERAH
TAHAP 2
SMP/MTS KOTA YOGYAKARTA
TAHUN PELAJARAN 2020/2021

Mata Pelajaran : BAHASA INDONESIA


Hari/Tanggal : SENIN/15 MARET 2021
Waktu : 07.30 – 09.30

PEMERINTAH KOTA YOGYAKARTA


DINAS PENDIDIKAN PEMUDA DAN OLAHRAGA
2021
1. Bacalah teks berikut!
Pemerintah gencar melaksanakan vaksinasi kepada masyarakat. Vaksinasi
dilakukan dalam dua tahap. Tahap pertama, vaksinasi ditujukan kepada tenaga
kesehatan. Tahap kedua, vaksinasi ditujukan kepada pekerja publik. Sampai
dengan 22 Februari 2021 sebanyak 1,2 juta orang telah mendapatkan vaksin
COVID-19.

Makna kata vaksinasi pada teks tersebut adalah ….


A. menyuntikkan obat ke dalam tubuh
B. menguatkan tubuh dari bibit penyakit yang ada
C. melemahkan bibit penyakit yang ada di dalam tubuh
D. memasukkan bibit penyakit yang sudah dilemahkan ke tubuh

2. Bacalah teks berikut!


Memakai masker telah menjadi salah satu protokol kesehatan yang wajib
dilakukan selama masa pandemi COVID-19. Masker bisa efektif mencegah virus
jika digunakan dengan baik dan benar. Meski demikian, masih banyak
masyarakat yang kurang memahami waktu yang tepat menggunakan masker.
Adapun penggunaan masker yang tepat, yaitu ketika berada di dalam ruangan
yang tidak memiliki ventilasi memadai dan berada di ruang tertutup dengan
banyak orang. Hal tersebut harus dilakukan karena berisiko meningkatkan
penularan COVID-19.

Tujuan penulisan yang terdapat pada teks tersebut adalah ….


A. mengajak masyarakat mengenakan masker setiap waktu
B. membujuk masyarakat untuk mengurangi risiko penularan COVID-19
C. menyadarkan masyarakat mengenai penggunaan masker secara tepat
D. memberi informasi kepada warga tentang fungsi masker di masa pandemi

3. Bacalah teks berikut!


Mengonsumsi buah dapat dilakukan kapan saja, baik sebelum atau sesudah
makan. Apabila Anda tidak memiliki masalah lambung, mengonsumsi buah
sebelum makan akan memberi hasil yang terbaik. Hal ini akan membuat nutrisi
dalam buah lebih banyak terserap oleh tubuh. Untuk orang yang ingin
menurunkan berat badan, makan buah sebaiknya dilakukan sebelum makan
agar perut cepat merasa kenyang.

Pernyataan yang sesuai dengan teks tersebut adalah …


A. Jika ingin menurunkan berat badan, jangan terlalu sering mengonsumsi buah.
B. Semakin sering mengonsumsi buah-buahan, tubuh kita menjadi semakin sehat.
C. Mengonsumsi buah-buahan sesudah makan, baik untuk menurunkan berat badan.
D. Mengonsumsi buah sebelum makan baik jika tidak memiliki masalah lambung.

4. Bacalah paragraf berikut!


Salah satu penyakit penyerta atau komorbid bagi pasien COVID-19 adalah
hipertensi. Hipertensi merupakan suatu keadaan dengan kondisi tekanan darah
terlalu tinggi, yaitu di atas 140/90 mmHg dan dianggap parah jika di atas
180/120 mmHg. Hipertensi dapat memperburuk infeksi COVID-19 sehingga
diperlukan kewaspadaan khusus. Penderita hipertensi yang terkena COVID-19
akan lemah imunitasnya sehingga kondisinya menjadi lebih berat.

TPM ASPD Paket. A Bahasa Indonesia 2


Dinas Pendidikan Kota Yogyakarta
Ide pokok paragraf tersebut adalah ….
A. kondisi tekanan darah yang terlalu tinggi bagi tubuh
B. hipertensi sebagai penyakit penyerta pasien COVID-19
C. diperlukan kewaspadaan khusus bagi penderita hipertensi
D. kondisi pasien COVID-19 yang menderita hipertensi menjadi lebih berat

5. Bacalah teks berikut!


(1) Seiring perpanjangan PPKM (Pemberlakuan Pengetatan Kegiatan
Masyarakarat) berbasis mikro di DIY, Pemda DIY terus menggencarkan protokol
kesehatan di masyarakat dan menggalakkan vaksinasi. (2) Vaksinasi bukan
semata-mata untuk kepentingan perorangan, melainkan juga masyarakat, yakni
mencegah penyebaran koronavirus ke orang lain. (3) Namun, hingga saat ini
masih ada sebagian orang yang menolak vaksin. (4) Pemerintah pun tetap
memberikan pendekatan persuasif dan edukatif ke masyarakat dengan harapan
dapat menumbuhkan kesadaran masyarakat bahwa vaksin menjadi kebutuhan
bersama untuk memerangi koronavirus.

Kalimat yang menyatakan hubungan pertentangan terdapat pada ….


A. (1) dan (2)
B. (2) dan (3)
C. (2) dan (4)
D. (3) dan (4)

6. Cermati poster berikut!

Kalimat yang mewakili isi poster tersebut adalah …


A. Masyarakat diingatkan tidak membiarkan korupsi tumbuh menjadi besar.
B. Masyarakat diajak untuk bersama-sama memberantas tindakan korupsi.
C. Masyarakat diingatkan tentang tahapan tumbuhnya berbagai korupsi.
D. Masyarakat diingatkan untuk tidak melakukan tindak korupsi.

TPM ASPD Paket. A Bahasa Indonesia 3


Dinas Pendidikan Kota Yogyakarta
7. Bacalah teks berikut!
Teks
Segala sesuatu yang dilakukan secara berlebihan dapat berakibat tidak baik,
termasuk olahraga. Olahraga berlebihan justru dapat mengakibatkan masalah.
Definisi olahraga berlebihan pada masing-masing orang berbeda, tergantung
dari kapasitas dan kemampuan masing-masing. Tubuh akan merespons negatif
terhadap olahraga yang dilakukan secara berlebihan atau terlalu keras. Kondisi
ini dikenal dengan istilah sindrom overtraining. Sindrom overtraining dapat
muncul melalui berbagai gejala fisik dan psikologis. Sindrom ini dapat
menyerang siapa saja yang berolahraga dengan durasi tinggi tanpa istirahat
yang cukup.
Kalimat Pernyataan
(1) Masyarakat meminta pemerintah untuk membuat larangan olahraga berat.
(2) Munculnya kekhawatiran dan ketakutan akan merebaknya sindrom
overtraining.
(3) Orang menjadi takut berolahraga berat karena tidak paham ukuran olahraga
yang aman.
(4) Orang menjadi lebih berhati-hati dalam berolahraga agar terhindar dari
sindrom overtraining.

Situasi yang muncul dalam masyarakat berkaitan dengan teks tersebut terdapat pada
kalimat ….
A. (1) dan (2)
B. (1) dan (3)
C. (2) dan (4)
D. (3) dan (4)

8. Bacalah teks berikut!


Vaksinasi COVID-19 harus dilakukan sebanyak dua kali. Hal ini sesuai SK
Dirjen Pencegahan dan Pengendalian Penyakit Nomor HK.02.02/4/1/2021
tentang Petunjuk Teknis Pelaksanaan Vaksinasi dalam Rangka Penanggulangan
Pandemi COVID-19. Pasalnya, pada penyuntikan vaksin COVID-19 yang
pertama, jumlah antibodi yang menetralkan virus masih rendah.
Jika tidak dilakukan penyuntikan vaksin COVID-19 yang kedua, dapat
memicu infeksi tanpa gejala atau asimtomatik. Hal tersebut dapat
memunculkan varian COVID-19 mengalami mutasi lebih resistan terhadap
antibodi yang baru terbentuk.

Ringkasan teks tersebut adalah …


A. Penyuntikan vaksin COVID-19 yang pertama membentuk antibodi penetral virus masih
rendah.
B. Vaksinasi COVID-19 harus dilakukan dua kali karena jika tidak dilakukan dapat memicu
infeksi tanpa gejala.
C. Vaksinasi COVID-19 harus dilakukan dua kali karena jika tidak dilakukan bisa
memunculkan varian baru virus COVID-19.
D. Infeksi tanpa gejala dapat memunculkan varian COVID-19 yang mengalami mutasi
yang lebih resisten terhadap antibodi yang baru terbentuk.

TPM ASPD Paket. A Bahasa Indonesia 4


Dinas Pendidikan Kota Yogyakarta
9. Bacalah teks berikut!
Hutan itu seperti kanopi raksasa yang menciptakan naungan penting bagi
berbagai kehidupan di lantai hutan. Naungan dari pepohonan di hutan juga bisa
mendinginkan suhu bumi yang dapat melawan pemanasan global. Hutan yang
luas memengaruhi pola iklim regional dalam menurunkan hujan. Saat hujan,
hutan akan menjadi spons raksasa yang menyerap dan menyimpan kembali air
ke dalam tanah.

Simpulan teks yang tepat adalah ...


A. Peran hutan bagi kelestarian alam sangat penting.
B. Hutan itu memberikan perlindungan bagi binatang.
C. Hutan menyelamatkan air dengan menyimpannya di dalam tanah.
D. Hasil hutan sangat penting untuk pembangunan bangsa dan negara.

10. Bacalah teks berikut!


(1) Tubuh memang membutuhkan sinar matahari untuk mendapatkan vitamin D
yang diperlukan dalam kehidupan sehari-hari. (2) Menurut penelitian,
kebutuhan vitamin D rata-rata manusia adalah 600 IU per hari. (3) Akan tetapi,
orang berusia di atas 70 tahun membutuhkan vitamin D 800 IU per hari. (4)
Tubuh memproduksi vitamin D secara otomatis ketika kulit terpapar sinar
ultraviolet B (UVB) dari matahari pada kondisi tertentu. (5) Itulah pentingnya
kita mendapatkan sinar matahari maka diimbau untuk berjemur dan tetap
memperhatikan kondisi dan waktu.

Kalimat yang menyatakan fakta adalah …


A. (1) dan (2)
B. (1) dan (3)
C. (2) dan (4)
D. (3) dan (5)

11. Bacalah teks berikut!


Teks 1 Teks 2
Antioksidan adalah zat penting, yang Awalnya bunga telang tumbuh liar.
bisa memproteksi sel sehat di dalam Tingginya mencapai lima meter.
tubuh. Zat antioksidan ini terdapat Namun, tanaman ini kini menjadi
pada bunga telang. Bahkan, bunga tanaman hias karena penampilannya
telang bisa digunakan untuk mencegah sangat cantik. Tak hanya cantik, bunga
atau memperlambat penyakit telang dapat menjadi obat.
degeneratif.

Perbedaan penggunaan bahasa kedua teks tersebut adalah ….


Teks 1 Teks 2
A kata-kata baku kata-kata tidak baku
B kata istilah tertentu kata populer
C ragam bahasa lisan ragam bahasa tulis
D kalimat tidak efektif kalimat efektif

TPM ASPD Paket. A Bahasa Indonesia 5


Dinas Pendidikan Kota Yogyakarta
12. Bacalah kutipan teks berikut!
Teks 1 Teks 2
Pesisir pantai wilayah Indonesia Sore hari, sekitar pukul 16.00 WIB,
bagian selatan Jawa, Bali, dan Nusa Pantai Parangtritis diwarnai suasana
Tenggara dilanda gelombang tinggi menegangkan. Saat itu gelombang
akibat perubahan musim. Di wilayah tinggi pantai selatan mencapai jauh ke
tersebut terjadi potensi hujan yang daratan. Setelah dihantam ombak, satu
minim dan terjadi peningkatan per satu lokasi tempat bermain
kecepatan angin. Di Samudra Hindia wisatawan, termasuk sarana prasarana
terjadi kondisi tekanan tinggi yang yang digunakan untuk duduk-duduk
bertahan sehingga memicu terjadinya menikmati suasana pantai tersapu
gelombang tinggi di perairan selatan ombak. Detik-detik bangunan ambruk
Indonesia. tersapu gelombang sempat diabadikan
oleh warga.
Perbedaan pola penyajian kedua kutipan teks tersebut diawali dengan ….
Teks 1 Teks 2
A peristiwa waktu
B tempat waktu
C pelaku peristiwa
D waktu tempat

13. Bacalah kutipan cerpen berikut!


(1) “Aku dari bekerja membawakan belanjaan ibu-ibu di pasar, Bu,” jelas Saskia
dengan wajah yang polos setelah ditanya ibunya.
(2) “Ya ampun, Nak! Kamu tidak usah bekerja lagi ya, tugasmu hanyalah belajar
biar Ibu dan Ayah yang mencari uang untuk biaya sekolahmu,” respons Ibu
Saskia sembari meneteskan air mata.
(3) “Ibu jangan menangis, Kia tidak apa-apa kok, Kia senang bisa membantu Ibu
dan Ayah,” sembari mengusap air mata sang ibunda.
(4) “Maafkan Ibu dan Ayah ya, Kia. Kami belum bisa membahagiakanmu seperti
anak-anak yang lainnya. Sekarang kamu mandi lalu makan ya, Nak,” sembari
mencium pipi gembil Saskia anak kesayangannya itu.
(5) “Aku tidak apa-apa Ibu, tapi maaf juga ya, Bu, hari ini aku tidak bawa uang.
Tadi, uangnya aku kasih untuk nenek tua yang belum makan,” ucap Kia
sembari memeluk ibundanya.
(6) “Kamu memang anak ibu yang sangat cantik dan baik hati, itu adalah hal
yang mulia, kamu tidak perlu minta maaf sama Ibu. Ibu sayang dan bangga
padamu,” jawab Ibu Saskia sembari memeluk erat anak kesayangannya itu.

Bukti Saskia berwatak murah hati ditunjukkan dengan kalimat ….


A. (1) dan (3)
B. (2) dan (5)
C. (4) dan (6)
D. (5) dan (6)

TPM ASPD Paket. A Bahasa Indonesia 6


Dinas Pendidikan Kota Yogyakarta
14. Bacalah kutipan cerpen berikut!
Kutipan Cerpen
Aku sudah sampai di toko Pak Apung untuk mulai bekerja. Dia memajang piring
mini, cangkir kecil, dan vas bunga berbentuk botol. Seluruhnya bernuansa
monokrom yang terpajang di dalam etalase kaca dan tertata rapi. Toko ini dari
pagi hingga siang tetap terlihat sepi. Aku sejak tadi duduk santai di dekat meja
kasir, mengutak-atik kuku tangan. Tiba-tiba, tak sengaja kusenggol sampel
piring mini di sebelah tangan kiriku. Aku yang ketakutan bergegas membereskan
sampel piring mini itu.
Kalimat Peristiwa
(1) Aku memajang piring mini, cangkir kecil, dan vas bunga berbentuk botol.
(2) Aku menyenggol sampel piring mini yang berada di sebelah tangan kiri.
(3) Aku sejak tadi duduk gelisah di dekat meja kasir menunggu pembeli.
(4) Aku membereskan sampel piring mini yang tak sengaja tersenggol.

Peristiwa yang dialami tokoh sesuai teks tersebut terdapat pada kalimat ….
A. (1) dan (2)
B. (2) dan (3)
C. (2) dan (4)
D. (3) dan (4)

15. Bacalah kutipan cerita fantasi berikut!


Mendekati rumahnya di hutan, Wiz bertemu dengan seorang kakek yang
sedang kebingungan. Wis mendekati kakek dan bertanya.
“Ada apa Kek? Ada yang bisa saya bantu?” tanya Wiz dengan lembut.
“Iya saya butuh bantuan. Saya mau pulang ke desa pinggir hutan. Tetapi
saya lupa jalan pulang, saya tersesat” jawab Kakek.
“Oh jangan khawatir Kek. Kakek makan saja buah belimbing biru ini!” kata
Wiz sambil menyerahkan belimbing terakhir dari dalam tasnya.
Beberapa saat kemudian tampaklah reaksi setelah kakek makan belimbing
biru. Kakek mulai sadar dan tahu arah jalan pulang.
“Terima kasih, saya ingat jalan ke rumah sekarang!” kata kakek senang.

Alasan Wiz memberikan belimbing biru kepada kakek adalah …


A. Wiz mengetahui manfaat belimbing biru untuk mengobati penyakit lupa.
B. Wiz hanya membawa belimbing biru dalam tas, tidak ada benda lain.
C. Mencoba-coba saja siapa tahu bisa menolong orang lain.
D. Wiz mengetahui kalau kakek tinggal di desa tepi hutan.

16. Bacalah kutipan cerpen berikut!


Hatiku berdebar dan sejak kemarin aku sulit tidur. Hari ini adalah hari
bersejarah untukku.
“Lulus atau tidak aku? Berapa nilaiku?” pertanyaan itu terus ada dalam
pikiranku sejak kemarin. Semoga hasil usahaku bisa membuat ayah bangga
padaku.

Latar suasana pada kutipan cerpen tersebut adalah ….


A. mencekam
B. bimbang
C. kacau
D. risau

TPM ASPD Paket. A Bahasa Indonesia 7


Dinas Pendidikan Kota Yogyakarta
17. Bacalah kutipan fabel berikut!
Kiko dan Miko, sepasang anak ikan asyik bermain di sebuah sungai yang
jernih.
“Berenang ke sana, yuk!” ajak Kiko pada Miko.
“Jangan! Di sana banyak perangkap yang dipasang manusia. Jika sudah
masuk, kita tak bisa keluar lagi,” Miko mengingatkan Kiko.
Kiko mengabaikan nasihat Miko. Ia meneruskan berenang dan melihat
sesuatu menyerupai jaring. Penasaran ia masuk dan menemukan makanan
enak. Kiko asyik makan tanpa menyadari bahaya. Tiba-tiba, tubuhnya
terperangkap ketika jaring diangkat oleh dua anak kecil yang mencari ikan di
sungai.

Cara pengarang menggambarkan watak Kiko melalui….


A. tingkah laku tokoh
B. secara langsung
C. ciri fisik tokoh
D. dialog tokoh

18. Bacalah kutipan cerpen berikut!


Sambil mendekap erat buku di dadanya, Prima menghela napas panjang.
Bibirnya tersenyum dan matanya terpejam. Ia sudah mantap dengan pilihannya.
Ia lega karena akhirnya ia berani mengambil keputusan. Tak ada lagi yang bisa
memaksanya mengambil jurusan yang tak diminatinya.

Kutipan cerita tersebut merupakan tahapan alur…


A. pengenalan
B. konflik
C. klimaks
D. peleraian

19. Bacalah kutipan cerpen berikut!


Vara membuka pintu dengan tergesa-gesa. Ia ingin segera masuk kamar. Air
mata membasahi kedua pipinya. Rasa kecewa yang dalam tidak mampu
dibendungnya. Ia tidak ingin siapa pun melihat keadaannya saat ini, termasuk
orang tuanya. Ibu sedang memasak di dapur sehingga tidak menyadari
kedatangan Vara. Namun, ayah yang sedang membaca koran di ruang tengah
tampak mengangkat alis dengan sikap Vara yang beda dari biasanya. Ayah
lantas berdiri dan menemui ibu di dapur. Ayah mengajak ibu menemui Vara di
kamar. Ayah ingin bertanya tentang kejadian yang dialami Vara.

Makna simbol mengangkat alis pada kutipan cerpen tersebut adalah ….


A. ingin tahu
B. khawatir
C. kecewa
D. marah

TPM ASPD Paket. A Bahasa Indonesia 8


Dinas Pendidikan Kota Yogyakarta
20. Bacalah kutipan fabel berikut!
“Hai, aku ikut bermain, ya,” sapa Tata si ikan mungil yang terkenal dengan
duri beracunnya.
“Hah, ada Tata si duri beracun. Awas, teman-teman, ayo kita pergi saja!”
teriak Ipang si ikan cupang kepada teman-temannya.
“Mengapa kamu bicara seperti itu, Pang? Aku juga ingin ikut kalian bermain
bersama. Aku sendirian tidak ada teman. Ayolah izinkan aku ikut bermain
bersama kalian, ya,” ucap Tata sambil memohon.
“Selamat tinggal, duri beracun,” ejek Loli si kuda laut.
Ini semua gara-gara duri beracunku,” Tata mengeluh dan sedih.
Keesokan harinya. Semua binatang laut sudah berkumpul di pesta Rara.
Mereka bermain bersama dengan riang. Tiba-tiba, dari arah timur datanglah
seekor ikan piranha yang ingin menyerang para binatang laut yang ada di pesta.
Dengan sigap dan tanpa rasa dendam, Tata langsung menyelamatkan Ipang,
Rita, Loli, dan binatang laut lainnya. Tata menempelkan duri beracunnya ke
tubuh ikan piranha.

Amanat yang terdapat pada kutipan fabel tersebut adalah …


A. Harus sigap dalam membantu dan menolong sesama makhluk.
B. Jangan membeda-bedakan teman dan menilai makhluk dari fisiknya.
C. Jangan membeda-bedakan teman dan ikhlas ketika menolong sesama.
D. Harus bisa menerima keadaan dan jangan bersedih dengan takdir Tuhan.

21. Bacalah kutipan cerita fantasi berikut!


"Malik, bertahanlah. Aku sudah mengirim sinyal pertolongan ke berbagai
planet. Aku yakin ada yang bersedia membantu kita," kata Meta sambil
mendekap Malik yang lemas.
Tiba-tiba seseorang mendarat di dekat Malik dan Meta. Orang itu seluruh
tubuhnya ditutup dengan besi putih elastis kecuali wajahnya. Wajahnya
menggunakan masker udara yang terhubung dengan tabung oksigen di
punggung. Ia datang karena melihat sinyal yang dikirim Meta.

Sudut pandang yang terdapat dalam kutipan cerita fantasi tersebut adalah ....
A. orang pertama pelaku utama
B. orang pertama pelaku sampingan
C. orang ketiga pengamat/terbatas
D. orang ketiga serba tahu

22. Bacalah teks berikut!


Ardani lunglai. Dilihatnya lagi angka-angka di rapornya. Tak ada yang
berubah. Sembilan dari sebelas mapel jauh di bawah KKM. Kembali ia menatap
kosong jalan pulang. Hatinya resah, gelisah, gundah. Kakinya berat
melangkah. Setitik air bening menetes perlahan.
“Ayah, Ibu, maafkan aku! Sungguh aku menyesal tak menuruti segala
nasihatmu,” rintihnya.
Hari menyenja. Ia tak juga kuasa melangkahkan kakinya.

TPM ASPD Paket. A Bahasa Indonesia 9


Dinas Pendidikan Kota Yogyakarta
Penyebab konflik kutipan teks tersebut yaitu...
A. Ardani tidak berani pulang.
B. Ardani menangisi nilai rapornya.
C. Penyesalan Ardani dengan nilai rapornya.
D. Nilai rapor Ardani banyak yang di bawah KKM.

23. Bacalah kedua kutipan fabel berikut!


Teks 1 Teks 2
Seekor Kijang berjalan keluar dari Kelinci melihat sayur-sayuran
sebuah hutan yang ditumbuhi pohon yang sangat banyak merasa senang.
besar dan rindang. Kijang ke luar Tiba-tiba perasaan senang tersebut
hutan bermaksud untuk mencari berubah menjadi sedih. Apa yang
makan karena sudah berhari-hari tidak terjadi, ternyata datanglah
makan. Kijang tampak bingung beberapa ulat yang menyelinap di balik
memikirkan tempat yang akan dituju, dedaunan tersebut.
akhirnya memutuskan untuk pergi ke “Hai, Kelinci, mengapa wajahmu
sebuah peternakan kambing. tampak murung?” tanya salah satu
Perkiraan Kijang di lokasi peternakan ulat.
itu banyak makanan dan berharap dia “Tidak apa-apa saya hanya
bisa medapatkan makanan dari para heran, kenapa kalian datang ke
kambing. Perjalanan yang cukup jauh kebunku tanpa seizinku?” tanya Kelinci
telah ditempuhnya dengan susah kebingungan.
payah. Ulat tertawa terbahak-bahak dan
bertanya, “Emang ada masalah?”

Perbedaan pola pengembangan cerita tersebut diawali .…


Teks 1 Teks 2
A pengenalan latar pemunculan masalah
B pengenalan latar perumitan masalah
C pengenalan tokoh puncak masalah
D peleraian penyelesaian

24. Bacalah kedua kutipan cerita fantasi berikut!


Teks 1 Teks 2
Peri Ungu melebarkan sayapnya. Ia Peri Ungu terjatuh. Ia didorong oleh
melindungi Liliput dari serangan sinar Peri Hitam. Untung saja, Peri Ungu
X. Komplotan penjahat itu terus segera ditolong oleh tujuh kurcaci.
menyerang. Lalu, Peri Ungu Dibawalah Putri Ungu ke belakang.
membawanya terbang ke angkasa.

Perbedaan penggunaan bahasa kutipan kedua teks tersebut adalah ….


Teks 1 Teks 2
A majas metafora majas hiperbol
B kalimat aktif kalimat pasif
C kata istilah Kata ungkapan
D kata baku kata-kata tidak baku

TPM ASPD Paket. A Bahasa Indonesia 10


Dinas Pendidikan Kota Yogyakarta
25. Bacalah kutipan fabel berikut!
Pada suatu hari Gajah mengadakan perjalanan yang sangat jauh. Ia
berkeliling hutan dan bertemu dengan Harimau yang sedang kesakitan karena
kejatuhan pohon.
“Gajah, Gajah tolong aku,” kata Harimau dengan sedikit menahan sakit.
Mendengar teriakan Harimau, Gajah langsung menolong dengan
mengangkat pohon yang menghimpit tubuh Harimau menggunakan belalainya.
“Terima kasih kawan seandainya kamu tidak segera datang menolongku,
mungkin aku sudah mati tertindih pohon yang sangat besar ini,” kata Harimau.

Komentar yang tepat terhadap watak tokoh Gajah adalah ...


A. Pertolongan yang diterima hendaknya segera dibalas.
B. Menolong orang lain tidak harus menunggu kita mampu.
C. Sifat suka menolong dapat meringankan beban orang lain.
D. Memanfaatkan kemampuan fisik untuk menolong orang lain.

26. Bacalah teks berikut!


(1) Sesuai dengan Undang-Undang No. 41 tahun 1999 tentang Kehutanan,
hutan adalah unit ekosistem dalam bentuk sebidang tanah yang mengandung
sumber daya alam hayati. (2) Pohon-pohon di lingkungan alami tidak dapat
dipisahkan satu sama lain. (3) Sebagai salah satu aspek terpenting dari litosfer
bumi, hutan berfungsi sebagai tempat penyimpanan karbon dioksida, tempat
untuk pertumbuhan hewan, pengaturan aliran hidrogen, dan konservasi tanah.
(4) Berdasarkan kondisi iklim, hutan dapat dibagi menjadi tiga jenis: hutan
tropis, hutan gugur, dan hutan taiga.

Istilah yang tepat untuk menggantikan kata bercetak tebal tersebut adalah ….
A. biosfer, hidrologi
B. biologi, hidroskopi
C. biogas, hidroponik
D. biosolar, hidroskrop

27. Bacalah teks berikut!


Orang yang terinfeksi COVID-19 dan influenza akan mengalami gejala infeksi
saluran pernapasan yang sama, seperti demam, batuk, dan pilek. Walaupun
gejalanya sama, penyebab virusnya berbeda-beda sehingga kita sulit
mengidentifikasi masing-masing penyakit tersebut. […] Bagi setiap orang yang
menderita demam, batuk, dan sulit bernapas sangat direkomendasikan untuk
segera mencari pengobatan. Selain itu, mereka harus menginformasikan
riwayat perjalanan yang telah dilakukan kepada petugas kesehatan.

Kalimat yang tepat untuk melengkapi bagian rumpang teks tersebut adalah …
A. Pemeriksaan medis yang akurat disertai rujukan pemeriksaan laboratorium
diperlukan untuk mengetahui apakah seseorang terinfeksi COVID-19.
B. Sampai saat ini, para ahli masih terus melakukan penyelidikan untuk menentukan
sumber virus, jenis paparan, dan cara penularannya.
C. Seperti penyakit pernapasan lainnya, COVID-19 dapat menyebabkan gejala ringan
termasuk pilek, sakit tenggorokan, batuk, dan demam.
D. Penyakit ini dapat menyebar melalui tetesan kecil (droplet) dari hidung atau mulut
pada saat batuk atau bersin.

TPM ASPD Paket. A Bahasa Indonesia 11


Dinas Pendidikan Kota Yogyakarta
28. Bacalah prosedur berikut!
Memakai Masker Medis yang Benar

(1) Ambil masker dan pastikan tidak ada sobekan atau lubang pada
masker.
(2) Bersihkan tangan dengan cairan antiseptik berbahan dasar alkohol sebelum
menyentuh masker.
(3) Pastikan bagian yang ada pita logamnya berada di sisi atas. Pastikan juga sisi
depan/luar adalah sisi yang berwarna.
(4) Setelah diperoleh masker dalam kondisi baik, pakai dengan benar.
(5) Terakhir, tarik sisi bawah masker yang masih berlipat-lipat sampai menutupi
mulut dan dagu.
(6) Pasang masker di wajah, lalu tekan pita logam sampai menempel sempurna
ke hidung.

Urutan prosedur memakai masker yang benar adalah …


A. (1)-(2)-(4)-(3)-(6)-(5)
B. (1)-(4)-(2)-(6)-(3)-(5)
C. (2)-(1)-(4)-(3)-(6)-(5)
D. (2)-(1)-(3)-(4)-(6)-(5)

29. Bacalah kalimat berikut!


Olahraga yang berlebihan akan memberi dampak buruk bagi tubuh.

Variasi kata dampak pada kalimat tersebut adalah ….


A. cedera
B. akibat
C. sebab
D. kesan

30. Bacalah teks berikut!


(1) Menanam jambu mete dapat dilakukan secara vegetatif dan generatif. (2)
Pembibitan vegetatif dapat dilakukan dengan penyambungan (grafting). (3)
Keunggulan pembibitan secara vegetatif yaitu adalah waktu berbuah lebih cepat
dan produksinya tinggi karena sesuai dengan sifat pohon induknya. (4)
Sementara itu, pembibitan secara generatif adalah perbanyakan tanaman yang
dilakukan dengan melalui cara penyemaian benih mete.

Kalimat yang tidak sesuai dengan kaidah terdapat pada .…


A. (1) dan (2)
B. (1) dan (4)
C. (2) dan (3)
D. (3) dan (4)

31. Bacalah kalimat-kalimat berikut!


(1) Simpulan diperoleh setelah membaca keseluruhan teks dengan seksama.
(2) Motivasi belajar sangat penting untuk mencapai tujuan belajar.
(3) Masyarakat dihimbau agar tidak melakukan kegiatan apa pun di sekitar
Gurung Merapi.
(4) Penyakit COVID-19 lebih berisiko menyerang orang dengan imunitas
lemah.

TPM ASPD Paket. A Bahasa Indonesia 12


Dinas Pendidikan Kota Yogyakarta
Kata tidak baku terdapat dalam kalimat ….
A. (1) dan (2)
B. (1) dan (3)
C. (2) dan (4)
D. (3) dan (4)

32. Bacalah paragraf berikut!


Pengurus Perkumpulan Binaraga Fitnes Indonesia (PBFI) Kota Yogyakarta […]
tiga atlet untuk tampil pada Kejuaraan Daerah (Kejurda) Binagara DIY yang
berlangsung di Star Gym, Turi, Sleman, Minggu (28/2). Meski hanya
mengirimkan tiga atlet saja, Ketua Umum Pengkot PBFI Yogyakarta, Fareza
Febriano, mengatakan target maksimal yang diraih adalah medali emas. Selain
memburu medali emas, ajang tersebut juga dijadikan sarana untuk melihat dan
[…] kekuatan binaragawan dari empat kabupaten lain di DIY.

Kata bentukan yang tepat untuk melengkapi paragraf tersebut adalah ....
A. disiapkan, pemetaan
B. persiapan, pemetaan
C. menyiapkan, memetakan
D. menyiapkan, mempetakan

33. Bacalah kalimat berikut!


Ia diberhentikan dari ketua gugus tugas sehingga tidak dapat bekerja maksimal.

Kesalahan penggunaan kata sehingga dalam kalimat tersebut karena ….


A. menunjukkan akibat
B. menunjukkan sasaran
C. tidak menunjukkan pilihan
D. tidak menunjukkan akibat sebab

34. Bacalah paragraf berikut!


(1) Kebun Binatang Gembiraloka memiliki koleksi satwa terlengkap se-Asia
Tenggara. (2) Berbagai jenis burung dari ukuran terkecil sampai terbesar ada di
sana. (3) Beberapa jenis mamalia juga menghuni kebun binatang itu. (4) Tidak
ketinggalan juga aneka reptil besar dan kecil terdapat di sana. (5) Para pawang
merawat binatang-binatang dengan profesional.

Perbaikan kalimat (5) agar menjadi paragraf padu adalah ...


A. Kebun binatang itu terletak di wilayah timur Kota Yogyakarta.
B. Bermacam-macam ikan di akuarium juga menghuni Gembiraloka.
C. Taman bermain untuk anak-anak tersedia lengkap di Gembiraloka.
D. Kebun Binatang Gembiraloka menyediakan aneka kuliner khas Yogyakarta.

35. Cermati kalimat berikut!


Biaya daftar ulang siswa baru diturunkan saat masa pandemi COVID-19.

Alasan ketidakefektifan kalimat tersebut adalah ….


A. bermakna ganda
B. bentuk kata tidak sejajar
C. bentuk jamak yang diulang
D. terpengaruh bahasa daerah

TPM ASPD Paket. A Bahasa Indonesia 13


Dinas Pendidikan Kota Yogyakarta
36. Bacalah kalimat-kalimat berikut!
(1) “Pangeran Arya harus makan, agar tidak sakit!” kata Ratu.
(2) “Makanlah, Pangeran!” pinta Ratu dengan lembut.
(3) “Lidah ini terasa pahit, Bunda,” jawab Pangeran pelan.
(4) “Jika tidak makan, tubuhmu akan lemah?” bujuk Ratu.

Penggunaan tanda baca yang tepat terdapat pada kalimat .…


A. (1) dan (2)
B. (2) dan (3)
C. (3) dan (4)
D. (4) dan (1)

37. Bacalah kalimat-kalimat berikut!


Tertulis Perbaikan
(1) Pak Gandi pergi ke kebun Pak Gandi pergi ke kebun menanam
menanam sayur-mayur; sawi, sayur-mayur: sawi, bayam, dan
bayam, dan singkong. singkong.
(2) Ibu membeli buku, pensil, dan Ibu membeli buku, pensil, dan tinta;
tinta: baju, celana, dan kaus, baju, celana, dan kaus; pisang, apel,
pisang, apel, dan jeruk. dan jeruk.
(3) Permainan beregu memerlukan: Permainan beregu memerlukan:
kekompakan; kerjasama; dan kekompakan, kerjasama, dan
koordinasi. koordinasi.
(4) Ayah menyelesaikan pekerjaan, Ayah menyelesaikan pekerjaan, ibu
ibu menulis makalah; adik menulis makalah: adik membaca
membaca cerita pendek. cerita pendek.

Perbaikan tanda baca yang tepat terdapat pada kalimat …


A. (1) dan (2)
B. (1) dan (3)
C. (2) dan (3)
D. (3) dan (4)

38. Bacalah kalimat-kalimat berikut!


Tertulis Perbaikan
(1) Para penjual batik Yogyakarta Para penjual Batik Yogyakarta sering
sering mendapatkan keuntungan mendapatkan keuntungan lebih pada
lebih pada libur nasional. libur Nasional.
(2) Jeruk Bali atau yang dikenal Jeruk bali atau yang dikenal dengan
dengan nama latin Citrus nama latin Citrus grandis atau Citrus
grandis atau Citrus maxima merupakan sejenis buah
maxima merupakan sejenis buah sitrus atau jeruk.
sitrus atau jeruk.
(3) Menonton layanan Menonton layanan video Streaming
video streaming yang tampaknya yang tampaknya menyenangkan
menyenangkan justru memiliki justru memiliki banyak efek negatif
banyak efek negatif bagi bagi kesehatan.
kesehatan.
(4) Pameran lukisan digelar di gedung Pameran lukisan digelar di Gedung
bentara budaya Yogyakarta Bentara Budaya Yogyakarta minggu
Minggu depan. depan.

TPM ASPD Paket. A Bahasa Indonesia 14


Dinas Pendidikan Kota Yogyakarta
Perbaikan ejaan yang tepat terdapat pada kalimat ….
A. (1) dan (2)
B. (1) dan (3)
C. (2) dan (4)
D. (3) dan (4)

39. Bacalah kalimat berikut!


Kepala Stasiun Klimatologi BMKG Yogyakarta menyampaikan bahwa hujan es
terjadi di Kawasan Kotabaru, Kota Yogyakarta pada hari Rabu, 3 Maret 2021.

Alasan kesalahan kata pada kalimat tersebut adalah .…


A. kata hari ditulis dengan huruf kecil
B. Stasiun Klimatologi ditulis huruf kapital
C. singkatan kata BMKG ditulis huruf kapital
D. kata kawasan bukan termasuk nama geografi

40. Cermati diagram berikut!

PAKET_A :
15 = PNS (biru tua)
15 = wiraswasta (orange)
25 = karyawan swasta (abu-abu)

Paragraf yang sesuai dengan diagram tersebut adalah …

A. Orang tua siswa kelas IX A memiliki bermacam profesi. Orang tua yang berprofesi sebagai
karyawan swasta berjumlah 15 orang. Orang tua yang berprofesi sebagai PNS berjumlah
20 dan wiraswasta sejumlah 5 orang.

B. Jenis pekerjaan orang tua kelas IX A cukup beragam. Profesi PNS mendominasi jenis
pekerjaan orang tua berjumlah 20 orang. Jenis pekerjaan lain adalah karyawan swasta
sejumlah 15 orang dan wiraswasta sejumlah 5 orang.

C. Jenis pekerjaan orang tua kelas IX A bervariasi. Profesi karyawan swasta mendominasi
jenis pekerjaan orang tua berjumlah 20 orang. Jenis pekerjaan lain adalah PNS berjumlah
15 orang dan wiraswasta berjumlah 5 orang.

D. Jenis pekerjaan orang tua kelas IX A terdiri dari 20 orang berprofesi sebagai karyawan
swasta. Selain itu, terdapat 15 orang tua berprofesi sebagai wiraswasta dan 5 orang tua
berprofesi sebagai PNS.

TPM ASPD Paket. A Bahasa Indonesia 15


Dinas Pendidikan Kota Yogyakarta
DOKUMEN TPM ASPD PAKET. B
SANGAT RAHASIA

TES PENDALAMAN MATERI


ASESMEN STANDARISASI PENDIDIKAN DAERAH
TAHAP 2
SMP/MTS KOTA YOGYAKARTA
TAHUN PELAJARAN 2020/2021

Mata Pelajaran : BAHASA INDONESIA


Hari/Tanggal : SENIN/15 MARET 2021
Waktu : 07.30 – 09.30

PEMERINTAH KOTA YOGYAKARTA


DINAS PENDIDIKAN PEMUDA DAN OLAHRAGA
2021
1. Bacalah teks berikut!
Sudah setahun lebih pandemi koronavirus menjadi mimpi buruk bagi dunia. Pandemi
COVID-19 membuat miliaran orang hidup dengan pembatasan dan aturan ketat demi
mengurangi risiko penularan. Berbicara mengenai akhir pandemi, tentu terkait dengan
berkurangnya kasus secara signifikan hingga level terkendali atau kategori endemi.
Meskipun, indikator untuk menyatakan akhir pandemi koronavirus berbeda-beda di
setiap negara atau kawasan.

Makna kata pandemi pada teks tersebut adalah ….


A. penyakit yang berjangkit di suatu daerah
B. wabah yang berjangkit di daerah tertentu
C. wabah yang berjangkit serempak di mana-mana
D. penyakit yang berjangkit dengan cepat di suatu daerah

2. Bacalah teks berikut!


Berdasarkan penelitian, tidur siang setiap hari selama 15–20 menit dapat membantu
mengurangi stres dan menurunkan risiko penyakit jantung. Selain itu, tidur siang
membuat kewaspadaan dan fokus meningkat. Tidur siang juga meningkatkan
kemampuan otak memproses informasi yang tidak disadari. Tidur siang dapat
membantu proses informasi, meningkatkan waktu reaksi, dan berpotensi
memengaruhi perilaku setelah bangun. Berdasarkan penelitian ini, disimpulkan bahwa
informasi yang dipahami secara tidak sadar akan diproses selama tidur untuk
membantu pengambilan keputusan ketika bangun.

Tujuan penulisan yang terdapat pada teks tersebut adalah ….


A. mengajak masyarakat untuk membiasakan perilaku tidur siang
B. memberikan informasi kepada masyarakat mengenai manfaat tidur siang
C. memengaruhi masyarakat untuk memanfaatkan waktu tidur siang dengan baik
D. menyarankan sebagian orang untuk melakukan tidur siang guna mengurangi stres

3. Bacalah teks berikut!


Daging jangan dijadikan hidangan utama makanan Anda. Lebih baik jadikan biji-bijian
dan sayuran sehat sebagai porsi utama Anda dan daging sebagai lauk. Dengan cara
ini Anda akan kenyang lebih cepat. Dengan mengonsumsi biji-bijian dan sayuran lebih
banyak, vitamin dan serat juga lebih banyak didapat. Cara ini lebih efektif untuk
program diet jangka panjang.

Pernyataan yang sesuai dengan teks tersebut adalah …


A. Daging lebih baik untuk menu diet karena mengandung banyak serat.
B. Mengonsumsi daging akan membuat kita mudah terserang penyakit.
C. Biji-bijian dan sayuran akan membuat Anda lebih cepat kenyang.
D. Biji-bijian lebih baik dijadikan lauk daripada daging.

4. Bacalah paragraf berikut!


Salah satu penyakit penyerta atau komorbid bagi pasien COVID-19 adalah diabetes.
Penyakit ini menduduki peringkat kedua setelah hipertensi. Penderita diabetes yang
terkena COVID-19 akan lemah imunitasnya sehingga kondisinya menjadi lebih berat.
Kadar glukosa darah yang tinggi meracuni pembuluh darah di seluruh tubuh,
termasuk jantung dan pembuluh besar lain.

TPM ASPD
Paket. B Bahasa Indonesia 2
Dinas Pendidikan Kota Yogyakarta
Ide pokok paragraf tersebut adalah ….
A. kadar glukosa yang tinggi bagi penderita diabetes
B. diabetes sebagai penyakit penyerta pasien COVID-19
C. kondisi pasien COVID-19 dengan komorbid diabetes lebih berat
D. penyakit diabetes menduduki peringkat kedua setelah hipertensi

5. Bacalah teks berikut!


(1) Peringatan Hari Peduli Sampah Nasional (HPSN) yang jatuh pada 21 Februari 2021
mengusung tema “Sampah Bahan Baku Ekonomi di Masa Pandemi”. (2) Di masa ini,
sampah justru menjadi suatu musibah tambahan atau collateral damage. (3)
Kementerian Lingkungan Hidup dan Kehutanan mengajak masyarakat mengelola
sampah karena memiliki nilai ekonomis. (4) Pengelolaannya dapat dikembangkan
melalui industri pengumpulan sampah, alat dan mesin pengolah, daur ulang,
komposting dan biogas serta industri sampah menjadi energi alternatif sehingga
bernilai ekonomi.

Kalimat yang menyatakan hubungan sebab akibat terdapat pada ….


A. (1) dan (2)
B. (2) dan (3)
C. (2) dan (4)
D. (3) dan (4)

6. Cermati poster berikut!

Kalimat yang mewakili isi poster tersebut adalah …


A. Masyarakat diberi tahu tentang pentingnya hidup di bumi.
B. Masyarakat diajak untuk hidup berdampingan pohon di bumi.
C. Masyarakat diajak melestarikan lingkungan untuk menjaga kehidupan di bumi.
D. Masyarakat diingatkan bahwa manusia dan pohon memiliki cara hidup yang sama.

TPM ASPD
Paket. B Bahasa Indonesia 3
Dinas Pendidikan Kota Yogyakarta
7. Bacalah teks berikut!
Teks
Tubuh memperoleh vitamin dari makanan dan minuman yang dikonsumsi. Asupan
vitamin dari makanan dan minuman bisa saja tidak mencukupi kebutuhan tubuh
sehingga diperlukan suplemen vitamin. Suplemen vitamin aman jika dikonsumsi
sesuai dosis dan cara yang tepat. Suplemen vitamin yang dikonsumsi berlebihan
dapat membawa dampak bagi tubuh. Vitamin dapat mencapai kadar toksik dan
berdampak buruk bagi kesehatan. Salah satu contohnya apabila seseorang
mengonsumsi terlalu banyak vitamin C atau mineral seng akan merasa mual dan
diare. Dengan demikian, seseorang harus memerhatikan dosis dan petunjuk
penggunaan suplemen vitamin sebelum mengonsumsinya.
Kalimat Pernyataan
(1) Masyarakat menjadi lebih waspada ketika akan mengonsumsi suplemen vitamin.
(2) Masyarakat merasa ketakutan ketika mengonsumsi suplemen vitamin.
(3) Masyarakat akan melakukan konsultasi dokter sebelum mengonsumsi suplemen
vitamin.
(4) Masyarakat meminta pemerintah untuk membuat larangan penjualan suplemen
vitamin.

Situasi yang muncul dalam masyarakat berkaitan dengan teks tersebut terdapat pada
kalimat ….
A. (1) dan (2)
B. (1) dan (3)
C. (2) dan (4)
D. (3) dan (4)

8. Bacalah teks berikut!


Situasi Indonesia saat ini darurat hoaks. Semakin banyak masyarakat
mengonsumsi informasi pada media digital tanpa memeriksa kembali kebenarannya
pada pandemi COVID-19. Hoaks di dunia maya selama pandemi COVID-19 mencapai
1.387 konten.
Indonesia membutuhkan edukasi literasi media untuk perangi hoaks atau berita
bohong. Edukasi literasi media dapat berbentuk formal, nonformal, dan informal.
Edukasi formal terjadi dalam ruang kelas di sekolah. Kemudian, edukasi nonformal
dapat dilakukan dengan target masyarakat umum di luar sekolah. Sementara itu,
edukasi informal dapat dilakukan dalam keluarga atau dilakukan mandiri.

Ringkasan teks tersebut adalah …


A. Situasi Indonesia saat ini berada dalam darurat hoaks sehingga membutuhkan
edukasi literasi media.
B. Edukasi literasi media untuk memerangi hoaks dapat dilakukan dengan edukasi
formal, nonformal, dan informal.
C. Darurat hoaks Indonesia di masa pendemi harus segera diatasi dengan edukasi
formal, nonformal, dan informal.
D. Situasi Indonesia saat ini darurat hoaks karena banyak masyarakat mengonsumsi
informasi media digital tanpa memeriksa kebenarannya.

TPM ASPD
Paket. B Bahasa Indonesia 4
Dinas Pendidikan Kota Yogyakarta
9. Bacalah teks berikut!
Tabulampot adalah teknik pembudidayaan tanaman buah dengan media pot.
Perawatan tabulampot dapat dilakukan secara mandiri, mulai dari pemilihan tanaman,
pemeliharaan, dan pemenuhan nutrisi. Untuk pemenuhan nutrisi tabulampot, dapat
dilakukan dengan pemberian pupuk sesuai ukuran dan jenis tanaman. Adapun pupuk
untuk tabulampot dapat dibeli di kios-kios pertanian. Serangan hama dan penyakitnya
mudah dikontrol dan ditangani. Kebutuhan air dapat dilakukan dengan penyiraman
secukupnya seperti biasa.

Simpulan teks yang tepat adalah …


A. Perawatan tabulampot mudah dilakukan.
B. Pupuk tabulampot dapat dibeli di kios-kios pertanian.
C. Kebutuhan air dipenuhi dengan penyiraman seperti biasa.
D. Serangan hama dan penyakit tabulampot mudah ditangani.

10. Bacalah teks berikut!


(1) Ada dua jenis ultraviolet yang berdampak pada kulit, yakni ultraviolet A (UVA) dan
ultraviolet B (UVB). (2) Menurut penelitian, UVA lebih berbahaya karena mampu
menembus bagian kulit terdalam (dermis). (3) Jika terpapar UVA, dimungkinkan kulit
akan menjadi gelap, keriput, dan berbintik hitam. (4) Sedangkan, UVB mencapai kulit
bagian luar (epidermis) yang mengakibatkan kulit terbakar.(5) Sebaiknya kita
menghindari paparan sinar ultraviolet secara berlebihan.

Kalimat yang menyatakan opini adalah …


A. (1) dan (2)
B. (1) dan (3)
C. (3) dan (4)
D. (3) dan (5)

11. Bacalah kedua teks berikut!


Teks 1 Teks 2
Indonesia merupakan negara pluralis. Dalam negara yang pluralis, seperti
Negara inimemiliki beragam agama, Indonesia dibutuhkan toleransi. Toleransi
suku, ras, dan golongan. Keragaman itu penting banget. Apalagi pada jaman
tersebut membutuhkan sikap toleransi sekarang di mana orang mudah tersulut
untuk menjaga persatuan. Toleransi emosi jika disingggung masalah sara. Oleh
tidak hanya menyangkut satu bidang karena itu, masyarakat harus memupuk
saja, tetapi setiap bidang membutuhkan toleransi agar tidak terjadi benturan.
toleransi.

Perbedaan penggunaan bahasa kedua teks tersebut adalah….


Teks 1 Teks 2
A ragam bahasa lisan ragam bahasa tulis
B terdapat kata-kata baku terdapat kata-kata tidak baku
C kalimat tidak efektif kalimat efektif
D adanya kata teknis adanya kata populer

TPM ASPD
Paket. B Bahasa Indonesia 5
Dinas Pendidikan Kota Yogyakarta
12. Bacalah kutipan teks berikut!
Teks 1 Teks 2
Sore hari, sekitar pukul 16.00 WIB, Pesisir pantai wilayah Indonesia bagian
Pantai Parangtritis diwarnai suasana selatan Jawa, Bali, dan Nusa Tenggara
menegangkan. Saat itu gelombang tinggi dilanda gelombang tinggi akibat
pantai selatan mencapai jauh ke daratan. perubahan musim. Fenomena gelombang
Setelah dihantam ombak, satu per satu tinggi di Indonesia ini disebabkan oleh
lokasi tempat bermain wisatawan, peningkatan kecepatan angin yang terjadi
termasuk sarana prasarana yang di wilayah tersebut. Di samping itu, kondisi
digunakan untuk duduk menikmati tekanan udara yang tinggi di Samudra
suasana pantai rusak. Detik-detik Hindia semakin memperparah kondisi
bangunan ambruk sempat diabadikan perairan selatan Indonesia.
warga.

Perbedaan pola penyajian kedua kutipan teks tersebut diawali dengan ….


Teks 1 Teks 2
A peristiwa waktu
B pelaku peristiwa
C tempat waktu
D waktu tempat

13. Bacalah kutipan cerpen berikut!


(1) “Assalamualaikum,” dengan suara yang lembut Saskia membuka pintu.
(2) “Wa’alaikumussalam, kamu dari mana saja, Kia?” tanya ibunya.
(3) “Tadi aku bekerja membawakan belanjaan ibu-ibu di pasar, Bu,” jelas Saskia
dengan wajah yang polos.
(4) “Ya ampun, Nak! Kamu tidak usah bekerja lagi ya, tugasmu hanyalah belajar, biar
Ibu dan Ayah yang mencari uang untuk biaya sekolahmu,” respons Ibu Saskia
sembari meneteskan air mata karena ia merasa bersalah harus membuat Saskia
ikut memikirkan uang sekolah.
(5) “Ibu jangan menangis, Kia tidak apa-apa kok, Kia senang bisa membantu Ibu dan
Ayah,” sembari mengusap air mata sang ibunda.
(6) “Maafkan Ibu dan Ayah ya, Kia. Kami belum bisa membahagiakanmu seperti anak-
anak yang lainnya. Sekarang kamu mandi lalu makan ya, Nak,” sembari mencium
pipi gembil Saskia anak kesayangannya itu.

Bukti Saskia berwatak pekerja keras ditunjukkan dengan kalimat ….


A. (5) dan (6)
B. (3) dan (4)
C. (2) dan (6)
D. (1) dan (6)

14. Bacalah kutipan cerpen berikut!


Kutipan Cerpen
Kesedihan sepertinya makin akrab pada seorang perempuan bersama bayinya
yang tak punya tempat tinggal. Sudah berhari-hari derita tak bosan mengepung
hidupnya. Baru saja menginjakkan kaki di pekarangan rumah salah seorang warga di
desa sebelah, ia sudah disembur.
"Istri orang perampok itu, ya? Pergi!” begitulah orang-orang mengusirnya.
Kesedihan terlihat di raut wajahnya. Ia tidak bisa mengelak dan pantang menyerah
mencari tempat tinggal sementara.

TPM ASPD
Paket. B Bahasa Indonesia 6
Dinas Pendidikan Kota Yogyakarta
Kalimat Peristiwa
(1) Perempuan itu putus asa dan ingin segera mengakhiri hidupnya.
(2) Perempuan itu baru saja memasuki desanya tetapi diusir warga.
(3) Perempuan itu terus memohon agar diizinkan menetap di desa tersebut.
(4) Perempuan itu pergi meninggalkan desa dengan berlinang air matanya.

Peristiwa yang dialami tokoh sesuai teks tersebut terdapat pada kalimat ….
A. (1) dan (2)
B. (2) dan (3)
C. (2) dan (4)
D. (3) dan (4)

15. Bacalah kutipan cerita fantasi berikut!


Suatu hari, Wiz pergi menggali sumur di desa sebelah hutan Morin. Tiba-tiba matanya
terkena pecahan batu galian. Sangat berbahaya kalau tidak cepat diobati. Wiz
mengambil belimbing kuning dari dalam tasnya kemudian dimakannya. Ajaib, seketika
itu juga sakit mata Wiz kembali pulih.

Alasan Wiz makan buah belimbing kuning adalah…


A. Wiz hanya mempunyai belimbing warna kuning.
B. Wiz mencoba-coba makan belimbing untuk mengobati matanya.
C. Wiz mengetahui khasiat belimbing kuning untuk menyembuhkan mata.
D. Wiz kebingungan karena tidak punya obat untuk menyembuhkan matanya.

16. Bacalah kutipan cerpen berikut!


Suara alarm jam telah membangunkanku di pagi ini. Mentari tersenyum manis
kepadaku. Hari pertamaku masuk SMA. Wajahku yang cerah berseri terpantul di
cermin kamarku. Baju putih abu-abu yang aku nantikan akhirnya bisa kupakai hari ini.
Baju kebanggaan ini aku dapatkan melalui usaha keras tak kenal lelah beberapa
waktu lalu.

Latar suasana pada kutipan cerpen tersebut adalah ….


A. gembira
B. optimis
C. ramai
D. haru

17. Bacalah kutipan fabel berikut!


Kori si Kura-kura sedang bercanda dengan Kodi, seekor kodok hijau yang lincah.
Tiba-tiba datanglah Jeji, jerapah muda dengan leher panjang.
“Hei, Jeji, lama tak bertemu. Kamu semakin tinggi saja,” sapa Kori.
“Hai juga, Kori. Tentu aku semakin tinggi karena tak pernah membawa beban ke
mana-mana,” sahut Jeji enteng.
“Jangan merendahkan teman, Jeji!” Kodi mengingatkan.
“Aku bicara kenyataan. Kori tergencet rumahnya sehingga tak tumbuh lagi
hahaha….” Kori terbahak. Tak lama mendung terlihat di langit dan hujan pun turun.
Jeji panik mencari tempat berteduh, sementara Kodi melompat kegirangan
menyambut hujan. Kori masuk ke dalam rumahnya dan tidur siang.

TPM ASPD
Paket. B Bahasa Indonesia 7
Dinas Pendidikan Kota Yogyakarta
Cara pengarang menggambarkan watak Jeji melalui….
A. tingkah laku tokoh
B. secara langsung
C. ciri fisik tokoh
D. dialog tokoh

18. Bacalah kutipan cerpen berikut!


Menjelang malam Rusti bersiap pulang. Kerja seharian membuat badannya lelah.
Langkahnya gontai menuruni tangga kantornya. Tangan kirinya mendekap map-map
berisi berkas yang belum diselesaikannya. Tangan kanannya menjinjing tas berisi
laptop.

Kutipan cerpen tersebut merupakan tahapan alur …


A. pengenalan
B. konflik
C. klimaks
D. peleraian

19. Bacalah kutipan cerpen berikut!


Aku berusaha sekuat hati mempertahankan pendapatku di depan Paman. Namun,
Paman tetap bersikeras dengan pendapatnya bahwa aku tidak boleh bekerja di luar
kota. Aku tahu Paman sangat khawatir apabila aku hidup sendirian di kota orang.
Paman adalah satu-satunya adik ayahku. Sebelum meninggal dunia, ayah menitipkan
aku kepada paman. Sejak kecil, pamanlah yang membiayai sekolah dan kebutuhanku.
Aku harus membalas kebaikan Paman dengan mencari pekerjaan yang mapan di
Jakarta. Tetap saja, Paman tidak mengizinkanku. Dengan nada tinggi, paman masih
mempertahankan keputusannya. Tiba-tiba Bibi datang. Ia mengerutkan alis berada di
tengah perdebatanku dengan Paman.

Makna simbol mengerutkan alis pada kutipan cerpen tersebut adalah ….


A. sedih
B. marah
C. kecewa
D. bingung

20. Bacalah kutipan fabel berikut!


Kelinci selalu membanggakan kecepatan berlarinya di depan hewan-hewan lain.
"Aku belum pernah terkalahkan dan akan kutunjukkan kecepatanku. Kamu mau
coba?" tanya si Kelinci menantang Kura-kura.
Sang Kura-kura menjawab tenang, "Aku menerima tantanganmu."
"Lelucon yang bagus," kata si Kelinci sinis, "aku bisa menari di sekitarmu sepanjang
jalan."
Kura-kura hanya tersenyum.
Kelinci langsung melesat hampir tak terlihat. Namun, dia terhenti di tengah
perjalanan karena kakinya luka terkena ranting runcing. Dia merasa kesakitan dan tak
kuat melanjutkan perjalanannya. Tak lama kemudian, sang Kura-kura melihat Kelinci.
Dia membantu mengobati luka kaki si Kelinci.

TPM ASPD
Paket. B Bahasa Indonesia 8
Dinas Pendidikan Kota Yogyakarta
Amanat yang terdapat pada kutipan fabel tersebut adalah …
A. Tidak boleh menyombongkan diri dan harus membantu satu sama lain.
B. Tidak boleh takut dan harus berani menunjukkan kehebatan yang dimiliki.
C. Tidak boleh membanggakan dan menyombongkan kelebihan yang dimiliki.
D. Tidak boleh memilih-milih jika akan membantu dan memberikan pertolongan.

21. Bacalah kutipan cerita fantasi berikut!


Walau ragu, Alisa tetap membuka pintu kayu berukir itu. Saat pintu terbuka,
cahaya terang menyinari Alisa. Beberapa saat kemudian, tangannya digandeng oleh
seseorang. Dituntunlah Alisa ke sebuah singgasana. Singgasana yang sangat megah.
Hatinya bertanya-tanya, ini mimpi atau nyata?
"Saya menunggu perintah sang Ratu," kata orang berperawakan gagah dengan
perasaan penasaran.
"Tunggu sebentar, kamu siapa?" tanya Alisa bingung.
"Saya adalah panglima perang di kerajaan ini," jawabnya.
Alisa baru menyadari bahwa ia berada di zaman kerajaan dan menjadi ratu pada
kerajaan tersebut.

Sudut pandang yang terdapat dalam kutipan cerita fantasi tersebut adalah ....
A. orang ketiga serba tahu
B. orang pertama pelaku utama
C. orang ketiga pengamat/terbatas
D. orang pertama pelaku sampingan

22. Bacalah kutipan teks berikut!


“Ma, tolonglah! Izinkan aku masuk SMK!” kata Rio memohon.
“Rio, kamu itu anak Mama satu-satunya. Tidak mungkinlah kamu sekolah di situ.
Itukan sekolah untuk anak-anak yang ekonominya pas-pasan. Jadi, lulus dari situ bisa
kerja, cari duit. Kamu kan tidak harus cari duit, Mama mampu menyekolahkanmu di
sekolah favorit, berapa pun biayanya,” jelas mama Rio panjang lebar.
“Tapi, Ma...,”
“Tidak ada tapi-tapian. Segera kemasi baju-bajumu! Mama sudah siapkan sekolah
terbaik untukmu,” tukas mama Rio sengit.
“Baik, Ma. Sudah saya kemasi dari kemarin, tapi tidak untuk ikut Mama. Lebih baik
saya pergi dari rumah ini,” balas Rio tak kalah sengit sambil menyeret tas kopernya.

Penyebab konflik kutipan teks tersebut yaitu...


A. Rio menerima sekolah pilihan mamanya dengan berat hati.
B. Perbedaan pilihan sekolah antara Rio dengan mamanya.
C. Mama Rio sangat menyayangi anak satu-satunya.
D. Rio ingin melanjutkan sekolah ke SMK.

TPM ASPD
Paket. B Bahasa Indonesia 9
Dinas Pendidikan Kota Yogyakarta
23. Bacalah kedua kutipan fabel berikut!
Teks 1 Teks 2
Matahari mulai terbit dan “Tidak Beo, dia sekarang sudah
burung–burung pun mulai berkicauan. berubah menjadi baik,” jawab Rusa sambil
Ini adalah waktunya sepasang kura- membela Harimau.
kura untuk mencari makanan. Di “Oh, ya? Aku ragu,” jawab Beo tidak
tengah perjalanan, saat mereka sedang percaya.
asyik mengumpulkan makanan, tiba– “Iya, maafkan aku!Aku telah bersalah
tiba mereka bertemu dengan si Kancil. sama kalian.Aku telah menyadari bahwa
Keduanya pun sudah menduga bahwa pertemanan lebih penting daripada saling
mereka pasti akan diolok–olok oleh si memakan,” ujar si Harimau sambil
Kancil. Ternyata dugaan itubenar. memelas.
“Aaaa....kalian jalannya lambat, “Oke, baiklah, janji ya!” pinta Beo
nggak kayak aku nih, cepat,” kata si yang masih ragu.
Kancil. “Janji,” jawab Harimau gembira.
Kura-kura hanya diamdan segera “Teman-teman aku pergi dulu ya,
melanjutkan aktivitas. masih banyak pekerjaan,” ujar Rusa
Sesampainya di rumah, Kura-kura mengalihkan pembicaraan.
berpikiran bahwa bagaimana kalau “Baiklah,” jawab Harimau dan Beo
misalnya ia melepaskan tempurungnya bersamaan.
agar dapat berjalan atau berlari cepat Sebelum berpisah mereka saling
seperti si Kancil. berjabat tangan menandakan persahabatan
yang masih terjalin dengan baik.

Perbedaan pola pengembangan cerita tersebut diawali ….


Teks 1 Teks 2
A puncak masalah peleraian
B perumitan masalah pengenalan latar waktu
C pengenalan latar perumitan masalah
D pengenalan latar penyelesaian

24. Bacalah kedua kutipan cerita fantasi berikut!


Teks 1 Teks 2
Kuda bertanduk itu terjatuh dari atas Peri Ungu menolong kuda bertanduk satu.
bukit. Ia didorong oleh musuhnya. Ia mengobati luka-luka kuda tersebut. Kuda
Kuda itu terperosok ke dalam jurang. itu meringis kesakitan. Namun, Peri Ungu
Untung saja, segera ditolong oleh Peri tidak memedulikannya.
Ungu.

Perbedaan penggunaan bahasa kutipan kedua teks tersebut adalah….


Teks 1 Teks 2
A kata baku kata-kata tidak baku
B kata istilah kata ungkapan
C majas metafora majas hiperbol
D kalimat pasif kalimat aktif

TPM ASPD
Paket. B Bahasa Indonesia 10
Dinas Pendidikan Kota Yogyakarta
25. Bacalah kutipan berikut!
Di antara sekian banyak nyamuk, ada satu nyamuk yang pemberani.
“Semua hewan harus tunduk kepadaku!” seru si Nyamuk.
Mendengar tantangan si Nyamuk, sang Singa marah.
Sementara itu, laba-laba sangat senang karena berhasil mendapatkan mangsa.
Seekor nyamuk akan menjadi santapannya yang lezat.

Komentar yang tepat terhadap watak tokoh si Nyamuk adalah ...


A. Si Nyamuk tidak mau tahu dengan tokoh lain.
B. Nyamuk takut kepada laba-laba karena mau disantap.
C. Si Nyamuk memiliki sifat yang perlu dihindari karena sombong.
D. Bila bisa mengalahkan sang Singa, si Nyamuk merasa ditakuti tokoh lain.

26. Bacalah teks berikut!


(1) Kecerdasan identik dengan kemampuan akademik yang mumpuni. (2) Padahal,
ada banyak bidang kecerdasan di luar akademis yang dimiliki individu. (3) Tokoh
pendidikan sekaligus psikolog yang mencetuskan teori kecerdasan majemuk, Howard
Gardner berpendapat, ada sembilan tipe kecerdasan pada manusia yang mungkin
dikuasai bila diasah dengan baik. (4) Salah satunya, yaitu kecerdasan pragmatik
yangditunjukkan dengan kemampuan menggunakan kata-kata dengan kreatif dan
menguasai bahasa, baik secara lisan maupun tulisan.

Istilah yang tepat untuk menggantikan kata bercetak tebal tersebut adalah….
A. praktis, efektif
B. linguistik, efektif
C. akademik, efisien
D. statistik, sistematik

27. Bacalah teks berikut!


Gerhana bulan terjadi saat sebagian atau keseluruhan penampang bulan tertutup oleh
bayangan bumi. Hal itu terjadi bila bumi berada di antara matahari dan bulan pada
satu garis lurus yang sama sehingga sinar matahari tidak dapat mencapai bulan
karena terhalang bumi. […] Hal ini disebabkan masih adanya sinar matahari yang
dibelokkan ke arah bulan oleh atmosfer bumi. Kebanyakan sinar yang dibelokkan ini
memiliki spektrum cahaya merah. Itulah sebabnya, pada saat gerhana, bulan akan
tampak berwarna gelap, bisa berwarna merah tembaga, jingga, atau cokelat.

Kalimat yang tepat untuk melengkapi bagian rumpang teks tersebut adalah…
A. Saat terjadi gerhana bulan, bulan sama sekali tidak tampak.
B. Pada peristiwa gerhana bulan, seringkali bulan masih terlihat.
C. Bulan adalah satu-satunya benda langit terdekat dengan bumi.
D. Bulan merupakan satelit bumi karena bergerak mengelilingi bumi.

28. Cermati prosedur berikut!


Mencuci Tangan yang Benar
(1) Tuangkan sabun secukupnya di telapak tangan
(2) Gosoklah telapak tangan yang sudah diberi sabun dengan merata
(3) Basahilah telapak tangan yang masih kering dengan air mengalir
(4) Bilas dengan menggunakan air mengalir
(5) Setelah bagian telapak selesai, mulailah menggosok sela-sela jari
(6) Lalu gosok punggung jari satu per satu secara bergantian

TPM ASPD
Paket. B Bahasa Indonesia 11
Dinas Pendidikan Kota Yogyakarta
Urutan kalimat yang tepat dalam penulisan teks perosedur tersebut adalah …
A. (1)-(3)-(2)-(6)-(5)-(4)
B. (1)-(3)-(6)-(5)-(2)-(4)
C. (3)-(1)-(2)-(5)-(6)-(4)
D. (3)-(1)-(6)-(5)-(2)-(4)

29. Bacalah kalimat berikut!


Terlalu banyak mengonsumsi kafein dapat mendatangkan indikasi tertentu bagi tubuh.

Variasi kata indikasi pada kalimat tersebut adalah ….


A. gejala
B. kronis
C. faktor
D. rentan

30. Bacalah teks berikut!


(1) Semangka dapat ditanam di dataran rendah hingga dataran tinggi. (2) Tanah
berkapur yang banyak mengandung bahan organik dan iklim kering cocok untuk
menanam semangka. (3) Tanaman semangka menyukai lokasi yang banyak sekali
sinar matahari. (4) Tanaman semangka juga memerlukan penyiraman yang intensif
terus-menerus.

Kalimat yang tidak sesuai dengan kaidah terdapat pada ….


A. (1) dan (2)
B. (1) dan (4)
C. (2) dan (3)
D. (3) dan (4)

31. Bacalah kalimat-kalimat berikut!


(1) Hembusan angin dari dalam tanah milik Pak Hendra diduga semburan gas.
(2) Fotosintesis merupakan proses pembentukan zat makanan pada tumbuhan.
(3) Wawancara dapat dijadikan sebagai metoda pengumpulan data dalam penelitian.
(4) Fondasi rumah merupakan bagian dari konstruksi bangunan yang sangat penting.

Kata tidak baku terdapat dalam kalimat ….


A. (1) dan (2)
B. (1) dan (3)
C. (2) dan (4)
D. (3) dan (4)

32. Bacalah paragraf berikut!


Presiden masih terus […] pemberian vaksinasi virus COVID-19 kepada masyakarat.
Kini vaksinasi tersebut ditujukan untuk tenaga pendidik dan kependidikan. Presiden
[…] pada bulan Juni 2021, lima juta tenaga pendidik dan kependidikan sudah
mendapatkan vaksinasi. Harapannya vaksinasi yang telah dimulai di Jakarta dapat
menjalar ke provinsi lain di Indonesia.

Kata bentukan yang tepat untuk melengkapi paragraf tersebut adalah....


A. bersosialisasi, ditargetkan
B. disosialisasikan, penargetan
C. mensosialisasikan, mentargetkan
D. menyosialisasikan, menargetkan

TPM ASPD
Paket. B Bahasa Indonesia 12
Dinas Pendidikan Kota Yogyakarta
33. Bacalah kalimat berikut!
Tujuan pemberian vaksin daripada masyarakat untuk memunculkan imunitas tubuh.

Kesalahan penggunaan kata daripada dalam kalimat tersebut karena ….


A. menunjukkan perbandingan
B. tidak menunjukkan perlawanan
C. tidak menunjukkan arah sasaran
D. menunjukkan keterangan tempat

34. Bacalah paragraf berikut!


(1) Kita dapat memanfaatkan lahan sempit untuk berkebun dengan teknik vertikultura.
(2) Teknik vertikultura adalah menanam tanaman dengan cara vertikal atau ke atas.
(3) Tanaman berakar serabut cocok ditanam di pot. (4) Bahan untuk membuat rak bisa
dari kayu, besi, atau pipa paralon. (5) Bahan paralon dapat dilubangi dan diletakkan
berdiri seperti menara.

Perbaikan kalimat (3) agar menjadi paragraf padu adalah ...


A. Teknik ini dilakukan dengan membuat rak-rak bertingkat menyerupai tangga.
B. Tanaman buah dan obat bisa dibudidayakan dengan teknik vertikultura.
C. Tanaman buah perlu perawatan khusus untuk hasil maksimal.
D. Rak-rak yang sudah jadi cocok ditanami tanaman hias.

35. Bacalah kalimat berikut!


Mobil pelukis yang sudah tua itu dijual mahal.

Ketidakefektifan kalimat tersebut karena ….


A. ketidaktepatan bentuk kata
B. kemubaziran kata depan
C. pengaruh bahasa daerah
D. bermakna ganda

36. Bacalah kalimat-kalimat berikut!


(1) “Gunakan ilmu meringankan tubuh,agar cepat sampai!” kata Pangeran Arya.
(2) “Selamat datang, Pangeran Arya!” sambut Paman Lembu Sora.
(3) “Apakah Kisanak yang bernama Lembu Sora?” tanya Pangeran Arya.
(4) “Benar sekali, Pangeran. Silakan masuk!”, sahut Paman Lembu Sora ramah.

Penggunaan tanda baca yang tepat terdapat pada kalimat .…


A. (1) dan (2)
B. (2) dan (3)
C. (3) dan (4)
D. (4) dan (1)

TPM ASPD
Paket. B Bahasa Indonesia 13
Dinas Pendidikan Kota Yogyakarta
37. Bacalah kalimat-kalimat berikut!
Tertulis Perbaikan
(1) ‘Jangan mengubah susunan meja “Jangan mengubah susunan meja
kursi!’ larang Pak David. kursi!” larang Pak David.
(2) Meskipun sudah diingatkan, mereka Meskipun sudah diingatkan; mereka
tetap saja berangkat mendaki gunung. tetap saja berangkat mendaki gunung.
(3) Para siswaberkumpul di aula pada Para siswa berkumpul di aula pada
Jumat; 5 Maret 2021 untuk berdoa Jumat: 5 Maret 2021 untuk berdoa
bersama. bersama.
(4) Bulan depan ada pertemuan kepala Bulan depan ada pertemuan kepala
desa se: Indonesia di Jakarta. desa se-Indonesia di Jakarta.

Perbaikan tanda baca yang tepat terdapat pada kalimat ....


A. (3) dan (4)
B. (2) dan (3)
C. (1) dan (4)
D. (1) dan (2)

38. Bacalah kalimat-kalimat berikut!


Tertulis Perbaikan
(1) Melonjaknya harga gula pasir ternyata Melonjaknya harga gula pasir ternyata
juga diikuti gula merah atau biasa juga diikuti gula merah atau biasa
dikenal sebagai gula jawa. dikenal sebagai gula Jawa.
(2) Tari gambyong merupakan salah satu Tari Gambyong merupakan salah satu
tarian tradisional Jawa Tengah yang tarian tradisional Jawa Tengah yang
dimainkan oleh beberapa penari wanita dimainkan oleh beberapa penari wanita
dengan indah dan anggun. dengan indah dan anggun.
(3) Anggrek atau yang dikenal dengan Anggrek atau yang dikenal dengan
nama latin orchidaceae merupakan nama latin Orchidaceae merupakan
suku tumbuhan berbunga yang suku tumbuhan berbunga yang
memiliki banyak jenis. memiliki banyak jenis.
(4) Pertunjukkan drama di Gedung Pertunjukkan drama digedung
Pamungkas itu terlihat sangat meriah. Pamungkas itu terlihat sangat meriah.

Perbaikan ejaan yang tepat terdapat pada kalimat ….


A. (1) dan (3)
B. (2) dan (3)
C. (2) dan (4)
D. (3) dan (4)

39. Bacalah kalimat berikut!


Pak Rahman menjabat kepala SMP Jaya yang terletak di jalan pandawa 15
Yogyakarta.

Kesalahan penulisan ejaan pada kalimat tersebut karena ….


A. jalan pandawa merupakan nama geografi
B. kepala SMP merupakan jabatan
C. Pak Rahman bukan nama diri
D. SMP merupakan singkatan

TPM ASPD
Paket. B Bahasa Indonesia 14
Dinas Pendidikan Kota Yogyakarta
40. Cermati diagram berikut!

PAKET_B :
10 = cipta cerpen (biru tua)
15 = cipta puisi (orange)
25 = baca puisi (abu-abu)

Paragraf yang sesuai dengan diagram tersebut adalah …


A. Dalam rangka memperingati bulan bahasa, SMP Kebangsaan mengadakan tiga jenis
lomba. Jenis lomba yang paling banyak diminati adalah lomba baca puisi dengan
peserta berjumlah 25. Lomba cipta puisi diikuti 15 peserta dan lomba cipta cerpen
diikuti 10 peserta.

B. Dalam rangka memperingati bulan bahasa, SMP Kebangsaan mengadakan beragam


perlomba. Jenis lomba yang paling banyak diminati adalah lomba cipta cerpen
dengan peserta berjumlah 25. Lomba cipta puisi diikuti 15 peserta dan lomba baca
puisi diikuti 10 peserta.

C. SMP Kebangsaan mengadakan berbagai lomba dalam rangka memperingati bulan


bahasa. Lomba ini diikuti banyak peserta. Terdapat 25 peserta mengikuti lomba cipta
puisi, 15 peserta mengikuti lomba cipta cerpen, dan 10 peserta mengikuti lomba baca
puisi. Para peserta antusias mengikuti lomba.

D. SMP Kebangsaan mengadakan berbagai lomba dalam rangka memperingati bulan


bahasa. Perlombaan berlangsung meriah dan sukses. Lomba ini diikuti banyak
peserta. Terdapat 15 peserta mengikuti lomba cipta puisi, 10 peserta mengikuti cipta
cerpen, dan 15 peserta mengikuti baca puisi. Semua peserta gembira.

TPM ASPD
Paket. B Bahasa Indonesia 15
Dinas Pendidikan Kota Yogyakarta
BAHASA INDONESIA
SELAMAT MENGERJAKAN
1. Cermatilah teks berikut!
Asam lambung merupakan zat penting dalam proses pencernaan. Kadar asam
lambung yang berlebihan atau terlalu sedikit dapat menimbulkan berbagai masalah
kesehatan, seperti nyeri perut, diare, dan kembung. Jika dibiarkan berlarut-larut tanpa
penanganan, kondisi tersebut dapat berakibat fatal.

Makna kata fatal pada teks tersebut adalah ….


A. celaka
B. mematikan
C. menerima nasib
D. tidak dapat diperbaiki

2. Cermatilah teks berikut!


Masyarakat memang tidak perlu melakukan persiapan khusus sebelum menerima
vaksin COVID-19. Akan tetapi, badan sehat dan tubuh sedang dalam kondisi bugar
adalah hal penting sebelum divaksin. Sebaiknya badan dalam kondisi bagus. Jangan
terlalu capek agar badan fit. Berpikir positif juga penting agar hati tenang dan
bahagia. Semua itu akan membuat nyaman saat menerima vaksin.

Tujuan penulisan teks tersebut adalah ….


A. menyampaikan informasi tentang prosedur penerimaan vaksin COVID-19
B. memberi tahu dampak tubuh sehat dan bugar setelah penerimaan vaksin COVID-19
C. meyakinkan orang bahwa penerima vaksin COVID-19 tidak perlu melakukan
persiapan khusus
D. mengimbau calon penerima vaksin COVID-19 agar menjaga asupan makan.

3. Cermatilah teks berikut!


Salah satu permainan tradisional yang banyak digemari di Indonesia adalah layang-
layang. Menurut Encyclopedia Britannica, layang-layang pertama kali dipopulerkan
di Cina sekitar 3.000 tahun yang lalu. Saat itu bahan-bahan yang ideal untuk
membuat layang-layang sudah tersedia, seperti kain sutra untuk bahan layar, sutra
berkekuatan tarik tinggi untuk terbang, dan bambu tangguh untuk kerangka yang kuat
dan ringan. Layang-layang Cina yang paling awal diketahui berbentuk persegi
panjang dan datar.

Pernyataan yang sesuai dengan teks tersebut adalah …


A. Layang-layang berbahan sutra banyak ditemukan di Indonesia.
B. Layang-layang dipopulerkan pertama kali di Encyclopedia Britannica.
C. Layang-layang sudah dipopulerkan di Cina sejak 3.000 tahun yang lalu.
D. Layang-layang adalah permainan tradisional yang paling digemari di Cina.
4. Cermatilah teks berikut!
Budi daya ikan nila hitam memiliki prospek yang sangat besar. Ikan ini mudah
dibudidayakan, baik dalam kolam semen, kolam tanah, atau keramba. Saat ini para
peternak hanya mampu memenuhi 75% kebutuhan pasar lokal. Untuk
menanggulangi hal ini, Dinas Perikanan dan Kelautan melakukan sosialisasi budi daya
nila hitam.

Ide pokok paragraf tersebut adalah ….


A. kebutuhan nila hitam
B. keunggulan nila hitam
C. prospek budi daya nila hitam
D. sosialisasi budi daya nila hitam

5. Cermatilah teks berikut!


(1) Penyakit paru obstruktif kronik (PPOK) menduduki peringkat ketiga kematian di
dunia. (2) Asap rokok menjadi penyebab utama PPOK karena mengandung bahan
kimia berbahaya. (3) Penyakit ini menghalangi aliran udara ke paru-paru sehingga
membuat seseorang sulit bernapas. (4) Sayangnya, belum ada obat untuk penyakit
ini.

Kalimat yang menyatakan hubungan sebab-akibat pada teks tersebut ditandai nomor ....
A. (1) dan (2)
B. B. (1) dan (3)
C. C. (2) dan (3)
D. (2) dan (4)

6. Cermatilah poster berikut!

Kalimat yang mewakili isi poster tersebut adalah ...


A. Penyalahgunaan narkoba dapat menimbulkan beberapa pemikiran bagi kaum milenial.
B. Kaum milenial perlu memahami pentingnya menghindari beberapa akibat narkoba.
C. Kita harus selalu memikirkan akibat penyalahgunaan narkoba pada kaum milenial.
D. Agar terhindar dari penyalahgunaan narkoba, kaum milenial perlu memikirkan
akibatnya.
7. Cermatilah teks dan pernyataan berikut!
Teks
Pandemi COVID-19 berdampak terhadap industri pariwisata di seluruh dunia.
Permintaan dari wisatawan domestik maupun mancanegara turun drastis. Penurunan
ini disebabkan oleh pemberlakuan pembatasan perjalanan dari berbagai negara.
Pembatasan tersebut dimaksudkan untuk membendung penyebaran virus. Indonesia,
sebagai salah satu negara pilihan tujuan wisata, juga tidak luput dari imbas ini.

Pernyataan
(1) Sektor wisata di Indonesia mengalami kelesuan.
(2) Indonesia tidak terpengaruh oleh pandemi COVID-19.
(3) Tingkat penyebaran virus COVID-19 di Indonesia menurun.
(4) Pelaku bisnis wisata di Indonesia akan beralih ke bidang kerajinan.

Situasi yang kemungkinan muncul dalam masyarakat berkaitan dengan isi teks tersebut
ditunjukkan kalimat pernyataan nomor ....
A. (1) dan (2)
B. (1) dan (3)
C. (2) dan (3)
D. (3) dan (4)

8. Cermatilah teks berikut!


Budaya sambatan merupakan sistem gotong royong yang masih sering dilakukan
di pedesaan. Budaya ini diwariskan turun-temurun dari generasi ke generasi.
Sambatan biasa dilaksanakan ketika ada warga tertimpa musibah atau sedang
melakukan pekerjaan besar, seperti membangun rumah, hajatan, dan panen, Kegiatan
ini dilaksanakan karena adanya rasa kekeluargaan.
Setiap warga yang mengikuti sambatan tidak mendapatkan bayaran sama sekali.
Tenaga yang mereka keluarkan hanya akan dibalas dengan pemberian konsumsi atau
makan bersama setelah kegiatan itu selesai. Hal ini untuk meningkatkan kekeluargaan
dan rasa solidaritas antarwarga.

Ringkasan isi teks tersebut yang benar adalah ...


A. Budaya sambatan yang diwariskan turun temurun dari generasi ke generasi
dilaksanakan pada pekerjaan besar.
B. Budaya sambatan yang masih sering dilakukan di pedesaan diikuti oleh warga tanpa
mendapatkan bayaran.
C. Kegiatan sambatan yang masih sering dilakukan di pedesaan selalu diakhiri dengan
makan bersama.
D. Budaya sambatan masih sering dilakukan oleh warga pedesaan karena ada rasa
kekeluargaan
9. Cermatilah teks berikut!
Buah pisang mengandung kalium dan serat bahan pangan yang dibutuhkan tubuh.
Kalium dapat mengurangi kadar natrium dalam tubuh yang menjadi salah satu
penyebab tekanan darah tinggi. Serat bahan pangan pada pisang mampu mengikat
lemak yang dapat mencegah pembentukan plak. Plak dapat menyempitkan dan
bahkan menyumbat pembuluh darah sehingga tekanan darah meningkat.

Simpulan isi teks tersebut adalah ...


A. Pisang memiliki kandungan kalium dan serat bahan pangan.
B. Kadar natrium dalam darah dapat menurunkan tekanan darah.
C. Kalium pada pisang dapat mengurangi kadar natrium dalam tubuh.
D. Mengonsumsi pisang secara rutin dapat menstabilkan tekanan darah.

10. Cermatilah teks berikut!


(1) Di tengah situasi bertarung melawan dan mengatasi pandemi COVID-19, awal
2021 ini, negara kita diterjang bencana alam bertubi-tubi, seperti gempa bumi, banjir,
dan tanah longsor. (2) Kita seakan-akan tidak diberi waktu bernapas. (3) Masyarakat
menghadapi dua ancaman sekaligus, yakni COVID-19 dan bencana alam. (4) Untuk
mengatasi hal tersebut, dibutuhkan strategi yang tepat dan menyeluruh agar
masyarakat mempunyai daya tahan kuat.

Kalimat yang menyatakan fakta dalam teks tersebut ditunjukkan nomor ...

A. (1) dan (2)


B. (1) dan (3)
C. (2) dan (4)
D. (3) dan (4)
11. dua teks berikut!
Teks 1 Teks 2
Burnout atau merasa gagal dan lesu Menurut Direktur Guardian Indonesia,
merupakan salah satu kondisi psikologis Naresh Kalani, kesehatan mental sama
yang sering dialami oleh seseorang. pentingnya dengan kesehatan phisik.
Berkaca pada masa pandemi COVID-19, Guardian merasa terdorong untuk
merasa gagal lazim dialami seseorang menyediakan layanan conseling
karena beraktivitas secara daring tanpa phsikologi geratis karena adanya
banyak keluar ruangan. Akibatnya, berbagai gangguan tersebut. Layanan itu
muncul kejenuhan dan gangguan dimaksudkan untuk membantu masarakat
kecemasan. yang membutuhkan.

Perbedaan penggunaan bahasa kedua teks tersebut adalah ...


Teks 1 Teks 2
A kalimat pasif kalimat aktif
B kalimat majemuk kalimat tunggal
C kata baku kata tidak baku
D kata asing kata daerah

12. Cermatilah dua teks berikut!


Teks 1 Teks 2
Pelaksanaan vaksinasi COVID-19 sudah Pemberian vaksinasi terhadap pengidap
masuk pada tahap kedua bagi lansia dan penyakit jantung, penyakit ginjal kronis,
petugas pelayanan publik. Vaksinasi serta pengidap penyakit hati atau liver
diprioritaskan di Pulau Jawa dan Bali. tidak dapat dilakukan. Ada aturan khusus
Untuk keperluan tersebut, pemerintah yang harus dipatuhi oleh petugas dalam
telah menyiapkan 7 juta vaksin untuk memberikan vaksin kepada penderita
didistribusikan ke tiap provinsi. Dalam penyakit tersebut. Oleh karena itu,
waktu dekat, pendistribusian vaksin pemberian vaksin terhadap penderita
tersebut segera sampai di 34 provinsi. penyakit tersebut diperbolehkan jika
keadaan pasien baik dan terkontrol.
Perbedaan pola penyajian kedua teks tersebut adalah ....
Teks 1 Teks 2
A umum-khusus sebab-akibat
B khusus-umum umum-khusus
C sebab-akibat umum-khusus
D akibat-sebab sebab-akibat

13. Cermatilah teks berikut!


(1) Andhini mengambil karung untuk mulai memungut rezeki, tetapi niatnya
diurungkan. (2) Pandangannya tertuju pada wanita paruh baya yang nampaknya
terlihat kebingungan. Entah apa yang sedang ia cari. (3) Karena merasa kasihan, dia
bangkit dan menghampiri ibu paruh baya tersebut. (4) “Maaf, Bu. Apa yang sedang
Ibu lakukan?” tanyanya dengan lembut.
(5) Ibu itu pun menoleh ke arahnya, melihatnya dengan tatapan menyelidik.
“Kau mau apa?” tanyanya ketus.
Andhini tersenyum mendengar jawabannya. Lalu, ia mendekat.
(6) “Maaf, Bu. Saya hanya ingin membantu Ibu. Itu pun kalau berkenan.”

Bukti watak tokoh Andhini suka menolong pada kutipan cerita tersebut terdapat pada
nomor ....
A. (1) dan (2)
B. (2) dan (5)
C. (3) dan (4)
D. (3) dan (6)

14. Cermatilah kutipan cerpen berikut!


Cerpen
Lusi yang duduk di sebelahku terdengar terisak pelan. Kami berdebar tak sabar
menunggu reaksi Mbah Joyo.
"Jika Mbah Joyo tak mau menerima, terus setiap hari makan apa?" tanya Lusi
dengan suara gemetar. Aku juga menyimpan pertanyaan yang sama seperti Lusi.
"Mboten usah dikasih berkat. Nanti enggak ada yang makan, loh," tolak Mbah Joyo
saat tangan Lusi menyerahkan kantong keresek hitam, menunggu diterima.
Peristiwa
(1) Lusi duduk sambil menangis.
(2) Lusi kecewa karena hadiahnya ditolak.
(3) Mbah Joyo menolak pemberian Lusi..
(4) Mbah Joyo tidak berselera makan pemberian orang lain.

Peristiwa yang terdapat pada kutipan cerpen tersebut ditunjukkan nomor ....
A. (1) dan (2)
B. (1) dan (3)
C. (2) dan (3)
D. (3) dan (4)

15. kutipan cerpen berikut!


Bu Ridwan hanya bisa bersabar. Gurauan anaknya secara tidak langsung telah
menyinggung perasaannya. Tapi tindak mungkin ia memarahi anaknya di depan banyak
orang. Ia tahu bagaimana harus bersikap. Baginya, pikiran anaknya tentang hidup ini
masih sebatas jangkauan pikiran sesaat karena belum tahu ucapan tersebut layak
diucapkan atau tidak. Justru, kalau ia memarahinya, ia takut anaknya akan dijauhi oleh
teman-temannya.
Sambil tersenyum, Bu Ridwan mempersilakan teman-teman anaknya menikmati
makanan yang sudah disiapkannya.

Alasan Bu Ridwan tidak memarahi anaknya dalam kutipan cerpen tersebut adalah ...
A. Anaknya masih belum dewasa.
B. Bu Ridwan mempunyai sikap keibuan.
C. Bu Ridwan adalah seorang ibu yang penyabar.
D. Bu Ridwan takut anaknya dijauhi oleh temannya.

16. Cermatilah kutipan cerpen berikut!


Suara bel nyaring dan panjang adalah alunan kebahagian bagi sebagian siswa. Mereka
akan langsung mengemas barang-barangnya. Ini adalah waktu yang tepat untuk pulang
setelah seharian menuntut ilmu. Berbeda dengan mereka, aku dan teman-temanku,
lebih memilih berlatih dan menyalurkan hobi di ruang latihan. Gelak tawa mewarnai
suasana latihan hari ini.. Sebelum matahari tenggelam, kami pantang untuk pulang.

Latar suasana kutipan cerpen tersebut adalah ....


A. senang
B. bangga
C. bosan
D. jenuh

17. Cermatilah kutipan cerpen berikut!


“Jadi berangkat sekarang, Tang?” Pak Zulham bertanya sembari menyamankan diri
di amben beralas tikar rumbia.
“Tentu, Paman, mumpung belum gelap.” Lintang meraih parang dan keranjang lalu
mengenakan alas kaki tipis yang berjejak telapak kakinya.
Ia bergegas menyusuri hutan bakau dan jajaran pohon nipah. Tangan mungil
dengan telapak kasar karena kapalan begitu terampil menebas daun nipah. Ia tetap
waspada menajamkan indera agar tak lengah pada reptil yang tiba-tiba muncul. Bukan
tak mungkin seekor buaya lapar mengintai dari sela-sela bakau atau seekor ular terusik
karena kehadirannya.

Pengarang menggambarkan watak tokoh Lintang melalui ….


A. secara langsung
B. dialog tokoh
C. tingkah laku
D. ciri fisik

18.
“Kata siapa Kodok selalu pandai bernyanyi?” Kadal melengos sambil melempar
sindiran.
“Tentu saja bangsa kodok pandai bernyanyi. Kami tak sepertimu, seumur hidupku
belum pernah mendengar senandungmu,” ejek Kodok pada Kadal. Kadal semakin
geram mendengar ejekan Kodok.
“Sebentar lagi hujan turun, aku semakin ingin bernyanyi,” kata Kodok dengan
senyum meremehkan. Merasa direndahkan, Kadal semakin tersinggung.

Kutipan fabel tersebut merupakan tahapan alur bagian ….


A. pengenalan
B. perumitan
C. peleraian
D. penyelesaian
19. Cermatilah kutipan cerpen berikut!
“Ayo, Rio, sini duduk dekat Eyang Sastro,” ajak Eyang Sastro pada Rio.
“Hari ini mau cerita apalagi, Eyang?” tanya Rio bersemangat.
Rio selalu antusias mendengarkan sambil memandang wajah dan kulit Eyang Sastro
yang penuh kerut karena dimakan usia. Eyang Sastro sering bercerita banyak hal pada
Rio sejak Rio kecil.

Makna dimakan usia pada kutipan cerpen tersebut adalah ….


A. tua
B. lama
C. bekas
D. dewasa

20.
Teks
Kucing Hitam tampak mengintip dari jendela. Aroma ikan goreng menggodanya
untuk datang. Tak lama kemudian pemilik rumah keluar dari dapur. Kucing Hitam
mengendap-endap menuju meja dan segera menyambar ikan goreng yang disajikan
di atas piring. Rupanya, ia berhasil membawa ikan goreng itu. Ketika ia akan melahap
ikan goreng, Kucing Kuning mendatanginya.
“Bolehkah aku meminta sedikit?” tanya Kucing Kuning.
“Tentu saja tidak! Aku bersusah payah mengambilnya dan kau datang meminta
bagian.”
“Baiklah, aku tak akan meminta. Tapi, hei, alangkah tak sopannya dirimu, makan
tanpa berdoa dan mencuci tangan. Kucing Hitam merasa malu. Ia meletakkan ikan
goreng dan mulai memejamkan mata untuk berdoa. Seketika Kucing Kuning
menyambar dan membawa lari ikan goreng milik Kucing Hitam.
Amanat
(1) Berbagilah kepada orang lain!
(2) Jangan meminta milik orang lain!
(3) Jangan mengambil milik orang lain!
(4) Berdoalah untuk kebaikan orang lain!!

Amanat yang terkandung dalam kutipan fabel tersebut adalah ….


A. (1) dan (3)
B. (2) dan (3)
C. (3) dan (4)
D. (4) dan (1)
21. Cermatilah kutipan cerpen berikut!
Sungguh, Riri begitu merindukan senyum Ayah. Sudah setengah tahun Riri tak
berjumpa dengan ayahnya. Profesi Ayah sebagai pelaut membuat Riri tak bebas
melepas rasa kangen.
Kadang, jika rasa rindu tak terbendung Riri kerap main ke pantai bersama Bunda.
Mereka akan memandang laut lepas dan mengirim salam melalui angin laut seraya
berkabar bahwa langit biru masih menaungi mereka sehingga ayah tak perlu khawatir.
Rangkaian doa selalu Riri sematkan untuk ayah, meski hanya dalam hati. Ayah akan
baik-baik saja di mana pun berada.

Sudut pandang penceritaan cerpen tersebut adalah ….


A. orang pertama pelaku sampingan
B. orang pertama pelaku utama
C. orang ketiga serba tahu
D. orang ketiga pengamat
22.
“Sudah tutup mata belum?” tanya sang Tikus pada Harimau.
“Kenapa harus tutup mata?” Harimau kebingungan sambil menggenggam sang
Tikus erat-erat.
“Tentu saja kau harus tutup mata. Jika tidak, tubuhku gagal membesar dan sama
sekali tak akan membuatmu kenyang.
Harimau mengikuti keinginan sang Tikus. Ia menutup mata dengan kedua
tangannya dan berharap tubuh tikus membesar guna memuaskan rasa laparnya. Sang
Tikus segera mengambil kesempatan meloloskan diri. Begitu menyadari tipuan sang
Tikus, Harimau mengaum marah karena merasa dibodohi.

Simpulan akibat konflik berdasarkan fabel tersebut adalah … A.


Tikus berhasil meloloskan diri dari Harimau.
B. Tikus menipu Harimau dan melarikan diri.
C. Harimau menutup mata untuk berdoa.
D. Harimau marah karena ditipu tikus.
23. Cermatilah dua kutipan cerita fantasi berikut!
Teks 1 Teks 2
“Mari kita coba sekali lagi!” ajak Bayu. Senja mulai merapat. Warna
“Bagaimana jika kita gagal? Lalu kekuningan emas menyebar di langit
adikku tak dapat keluar lagi dari lorong mengantarkan rasa hangat di dada Sheryl.
itu?” Anggi bertanya dengan suara Saat Sheryl akan menutup pintu pagar
hampir menangis. rumahnya, ia melihat sesuatu. Tampak
“Jika kita tidak mencoba, bagaimana sebuah titik hitam di langit yang mulai
kita tahu. Bukankah lebih baik berusaha kemerahan. Apakah itu? Sheryl bertanya.
meski kita belum tahu hasilnya?” Bayu Makin lama tampak semakin jelas, seperti
berusaha meyakinkan Anggi mahluk berekor terbang di udara.

Perbedaan pola pengembangan kedua kutipan cerita tersebut diawali ….


Teks 1 Teks 2
A deskripsi fisik tokoh pemuculan masalah
B pengenalan latar pengenalan tokoh
C pemunculan masalah deskripsi fisik tokoh
D pemunculan masalah pengenalan latar
24.Cermatilah kutipan cerpen berikut!
Teks 1 Teks 2
Jabat tangannya bagai roti dalam “Bu, boleh Niko berangkat sekarang?”
keranjang yang baru dikeluarkan dari Niko menutup ransel yang berisi
oven. Hangat dan lembut. Walau tak perlengkapan berkemah.
jumpa sepuluh tahun, ia tetaplah teman “Semua sudah lengkap? Jangan lupa
istimewa. Seperti siang ini, meski panas obat-obatan. O, ya, Ibu sudah buatkan
sang surya mendidihkan ubun-ubun, serundeng. Ini buah tangan untuk guru
senyum Shinta sahabatku terasa sejuk. pembimbingmu!” Ibu menyerahkan
Hal-hal yang kurang menyenangkan bungkusan kepada buah hatinya.
selalu ditanggapi Shinta dengan positif. “Terima kasih, Bu. Niko berangkat, ya.”
“Bagaimana kabarmu?” tanyaku Niko mengambil bungkusan menyalami
antusias. tangan ibu. Niko bergegas memanggil ojek
daring.

Perbedaan penggunaan bahasa kedua cerpen tersebut adalah ….


Teks 1 Teks 2
A kalimat bermajas ungkapan
B adanya ungkapan tidak ada ungkapan
C kalimat lugas kalimat bermajas
D kalimat langsung kalimat tidak langsung

25. Cermatilah kutipan cerpen berikut!


“Ibu, kapan aku akan dibelikan laptop baru? Laptop yang lama sudah ketinggalan
zaman, Bu,” kata Mira pagi itu saat sarapan.
“Masih bisa dipakai, kan, Mira? Apakah ada aplikasi yang tidak bisa diinstal di dalam
laptop yang selama ini kamu pakai?” tanya Ibu.
“Tidak ada. Mau ganti saja, Bu. Apalagi baru ada diskon. Lumayan, Bu,” bujuk Mira.
“Kalau masih bisa digunakan, kan tidak harus segera ganti laptop baru, Nak,” tutur
Ibu dengan lembut.
“Ah, Ibu,” kata Mira merajuk.

Komentar yang tepat terhadap isi kutipan cerpen tersebut adalah ...
A. Upaya Ibu dalam memberikan pemahaman kepada Mira merupakan tindakan
bijaksana.
B. Sudah sewajarnya, Mira meminta laptop baru karena laptopnya ketinggalan zaman.
C. Ibu seharusnya membelikan Mira laptop selagi ada diskon sehingga harganya murah.
D. Ibu seharusnya memenuhi permintaan Mira karena laptop merupakan sarana belajar.
26.Cermatilah teks berikut!
Perubahan iklim merupakan kapasitas menyimpangnya kondisi lingkungan,
khususnya cuaca di muka bumi secara global dan berkepanjangan. Penyebab utama
perubahan iklim tersebut adalah aktivitas manusia yang kurang bijak. Penggunaan
bahan bakar fosil, penggundulan hutan, dan pembuangan gas industri adalah contoh
praktis pemicu pemanasan global. Padahal, dampak yang ditimbulkan sangat beragam,
seperti krisis pangan, kekeringan, wabah penyakit baru, gangguan ekologi, dan
masalah lainnya.

Perbaikan penggunaan istilah pada teks tersebut adalah ....


Tertulis Perbaikan
A kapasitas, praktis fenomena, konkret
B aktivitas, krisis kreativitas, kritis
C kapasitas, krisis fenomena, kritis
D aktivitas, praktis kreativitas, konkret

27. Cermatilah teks berikut!


Penyakit jantung koroner merupakan penyebab utama morbiditas (kesakitan) dan
mortalitas (kematian) di seluruh dunia, baik di negara berkembang maupun negara
maju. Penyakit jantung koroner menjadi penyebab utama lebih dari 1/3 kematian
orang berusia di atas 35 tahun. [...]. Namun, penelitian menunjukkan bahwa terdapat
2%-6% penyakit jantung koroner terjadi pada usia muda (usia<45 tahun).

Kalimat yang tepat untuk melengkapi teks tersebut adalah ...


A. Penyakit jantung koroner dapat dicegah dengan pola makan yang sehat.
B. Pola makan seseorang berpengaruh terhadap penyakit jantung koroner.
C. Agar terhindar dari penyakit jantung koroner, pola makan harus dijaga.
D. Umumnya, penyakit jantung koroner diderita oleh pasien usia lanjut.

28. Cermatilah teks prosedur berikut!


Cara Membuat Hand Sanitizer dari Daun Lidah Buaya (1)
Berikutnya, tuang 150 ml alkohol berkadar 60% ke dalam gelas takar!
(2) Campurkan gel lidah buaya dengan alkohol hingga menyatu!
(3) Kupas daun lidah buaya, ambil gelnya!
(4) Hand sanitizer siap digunakan!
(5) Haluskan gel dengan sendok lalu tuang ke dalam gelas takar sebanyak 50 ml!
(6) Diamkan campuran selama beberapa menit!
Urutan teks prosedur yang tepat adalah ….
A. (3)-(5)-(1)-(6)-(2)-(4)
B. (3)-(5)-(1)-(2)-(6)-(4)
C. (3)-(5)-(2)-(6)-(1)-(4)
D. (3)-(1)-(5)-(1)-(2)-(4)
29. Cermatilah kalimat berikut!
Para ahli memiliki pandangan bahwa perubahan iklim secara global ditandai dengan
peningkatan suhu global bumi, baik suhu di darat, laut, maupun di udara.

Variasi kata pandangan pada kalimat tersebut adalah ….


A. asumsi
B. dugaan
C. prakiraan
D. pendapat

30. Cermatilah teks berikut!


(1) Kondisi kesehatan, kebugaran, dan kesegaran tubuh akan memengaruhi aktifitas
kegiatan siswa sehari-hari. (2) Tubuh yang sehat dan bugar membuat siswa
berkonsentrasi belajar. (3) Selain itu, olahraga rutin perlu dilakukan. (4) Siswa
hendaknya juga mengonsumsi makanan dan minuman yang sangat banyak sekali
mengandung nutrisi dan gizi yang diperlukan tubuh.

Kalimat tidak efektif dalam paragraf tersebut ditandai nomor ….


A. (1) dan (2)
B. (1) dan (4)
C. (2) dan (3)
D. (3) dan (4)

31. Cermatilah kalimat-kalimat berikut!


(1) Seorang manajer harus mampu memberikan suri teladan kepada para karyawannya.
(2) Prosentase keberhasilan pelaksanaan program vaksinasi COVID-19 oleh Kemenkes
sangat tinggi.
(3) Festival Andong 2021 diselenggarakan dengan tujuan
mengenalkan transportasi tradisionil.
(4) Masa pancaroba biasa ditandai dengan hujan yang sangat deras disertai petir, guntur,
dan angin kencang.

Kata tidak baku terdapat pada kalimat nomor ….


A. (1) dan (2)
B. (1) dan (4)
C. (2) dan (3)
D. (3) dan (4)
32. Cermatilah teks berikut!
Jambu biji merupakan salah satu buah dengan kandungan vitamin C yang tinggi.
Bahkan, kandungan vitamin C dalam jambu biji [...] jeruk. Vitamin C ini berpengaruh
terhadap peningkatan sistem kekebalan tubuh dan [...] risiko terkena infeksi. Selain
itu, vitamin C juga penting untuk menjaga kesehatan tulang, gigi, gusi, dan pembuluh
darah.

Kata bentukan yang tepat untuk melengkapi teks tersebut adalah ....
A. kelebihan, mengurangkan
B. melebihi, mengurangkan
C. kelebihan, pengurangan
D. melebihi, pengurangan

34. Cermatilah kalimat berikut!


Bahan baku rumah itu terbuat daripada kayu-kayu pilihan.

Penggunaan daripada pada kalimat tersebut salah karena tidak menunjukkan ….


A. perbandingan
B. kata depan
C. tempat
D. asal

35. Cermatilah teks berikut!


(1) Warga kaget dengan datangnya banjir bandang di kampung mereka. (2) Mereka
masih terlelap saat air tiba-tiba masuk ke rumah dan mengepung dari segala penjuru.
(3) Air memang sangat dibutuhkan dalam kehidupan manusia. (4) Warga pun panik
sehingga tidak sempat menyelamatkan harta bendanya.

Perbaikan kalimat (3) agar paragraf menjadi padu adalah …


A. Warga mengundang berbagai pihak untuk menyingkirkan material.
B. Air datang sangat cepat, menghanyutkan harta benda milik warga.
C. Air itu sangat keruh dan tidak sehat untuk kesehatan warga.
D. Pemerintah setempat segera memberikan pertolongan.

36. Cermatilah kalimat berikut!


Siswa yang membawa handphone harap dimatikan.
Kalimat tersebut tidak efektif karena ….
A. bermakna ganda
B. tidak memiliki objek
C. tidak terdapat predikat
D. terpengaruh bahasa daerah
36.Cermatilah kalimat-kalimat berikut!
(1) Ibu membeli lauk pauk: tahu, tempe, dan ayam goreng.
(2) Varian baru koronavirus, diduga telah masuk ke Indonesia.
(3) Meskipun hujan turun deras, dia tetap pergi belajar bersama.
(4) Hujan turun disertai angin, sehingga beberapa pohon tumbang.

Kalimat dengan tanda baca yang tepat terdapat pada nomor ….


A. (1) dan (2)
B. (1) dan (3)
C. (2) dan (4)
D. (3) dan (4)

37. Cermatilah penggunaan tanda baca pada kalimat berikut!


Kepala SMP Nusantara Susetyo Santoso,M.Pd.,mengatakan bahwa sukses tidak ditunggu
tetapi diburu.

Perbaikan penggunaan tanda baca yang tepat pada kalimat tersebut adalah ...
A. Kepala SMP Nusantara, Susetyo Santoso,M.Pd.mengatakan bahwa sukses tidak
ditunggu tetapi diburu.
B. Kepala SMP Nusantara, Susetyo Santoso,M.Pd.,mengatakan bahwa sukses tidak
ditunggu, tetapi diburu.
C. Kepala SMP Nusantara Susetyo Santoso,M.Pd.,mengatakan bahwa sukses tidak
ditunggu, tetapi diburu.
D. Kepala SMP Nusantara Susetyo Santoso M,Pd.mengatakan bahwa sukses tidak
ditunggu tetapi diburu.
38. Cermatilah kalimat berikut!
Meskipun masih muda, kedatangan bapak bagas di desa sukamaju di sambut warga
dengan penuh hormat.

Perbaikan penulisan ejaan yang tepat pada kalimat tersebut adalah ...
A. Meskipun masih muda, kedatangan Bapak Bagas di desa Sukamaju disambut warga
dengan penuh hormat.
B. Meskipun masih muda, kedatangan bapak Bagas di desa Sukamaju disambut warga
dengan penuh hormat.
C. Meskipun masih muda, kedatangan Bapak Bagas di Desa Sukamaju disambut warga
dengan penuh hormat.
D. Meskipun masih muda, kedatangan Bapak Bagas di Desa sukamaju di sambut warga
dengan penuh hormat.
39.Cermatilah kalimat berikut!
Sejak pandemi COVID-19, pengunjung pasar beringharjo berkurang.

Penulisan pasar beringharjo seharusnya diawali huruf kapital karena merupakan nama ....
A. geografi
B. jenis benda
C. badan atau lembaga
D. dokumen resmi negara

40. Cermatilah tabel berikut!

Tabel Kandungan Kalori Makanan Pokok Golongan A

Nama Makanan Berat (gr) Kalori


nasi putih 100 175
jagung rebus 250 90,2
kentang rebus 200 166
singkong rebus 100 146
talas rebus 100 98

Pernyataan yang sesuai dengan tabel tersebut adalah ....


A. Kentang rebus aman dimakan karena lebih sedikit kalori daripada talas rebus.
B. Kandungan kalori kentang rebus lebih banyak daripada singkong rebus.
C. Kandungan kalori terendah terdapat pada jagung rebus.
D. Nasi putih mengandung gizi makanan terbanyak.

Anda mungkin juga menyukai